• Shuffle
    Toggle On
    Toggle Off
  • Alphabetize
    Toggle On
    Toggle Off
  • Front First
    Toggle On
    Toggle Off
  • Both Sides
    Toggle On
    Toggle Off
  • Read
    Toggle On
    Toggle Off
Reading...
Front

Card Range To Study

through

image

Play button

image

Play button

image

Progress

1/273

Click to flip

Use LEFT and RIGHT arrow keys to navigate between flashcards;

Use UP and DOWN arrow keys to flip the card;

H to show hint;

A reads text to speech;

273 Cards in this Set

  • Front
  • Back
1. Which one of the following drugs inhibits platelet function for the life of the platelet?
A) Aspirin
B) Ibuprofen
C) Dipyridamole (Persantine)
D) Ticlopidine (Ticlid)
E) Warfarin (Coumadin)
Item 1
ANSWER: A
A number of drugs inhibit platelet function, but aspirin is the only effective drug that interferes with
platelet aggregation for the life of the platelet. It does this by permanently acetylating the platelet enzyme
cyclooxygenase, thus inhibiting prostaglandin synthesis. This phenomenon is clinically helpful when an
antithrombotic effect is desired, but it may require that necessary surgical procedures be delayed. The
effect of a single aspirin on bleeding times can persist for up to 5 days. Other NSAIDs (i.e.,
indomethacin, sulfinpyrazone) also inhibit platelet activity, but their effect on prostaglandin synthesis is
reversible. The anti-platelet effect of dipyridamole is less well understood. Warfarin is a biochemical
antagonist of prothrombin and vitamin K-dependent coagulation factors, and therefore has no significant
effect on platelet activity.
2. An otherwise healthy 10-year-old female presents with a papulovesicular eruption on one leg.
It extends from the lateral buttock, down the posterolateral thigh, to the lateral calf. It is mildly
painful. The patient’s immunizations are up to date, including varicella and MMR. Her family
has a pet cat at home, and another child at her school was sent home with a rash earlier in the
week.
Which one of the following is the most likely diagnosis?
A) Contact dermatitis
B) Herpes zoster dermatitis
C) Tinea corporis
D) Scabies
Item 2
ANSWER: B
Herpes zoster can occur from either a wild strain or a vaccine strain of varicella-zoster virus in vaccinated
children, but the incidence is low. All cases are mild and uncomplicated.
3. According to the U.S. Preventive Services Task Force, which one of the following is true
regarding prostate cancer screening?
A) Most men between the ages of 50 and 75 should be screened for prostate cancer
B) Screening for prostate cancer is inappropriate at any age
C) The harms of screening for prostate cancer after the age of 75 outweigh the benefits
D) A digital rectal examination has greater sensitivity than prostate-specific antigen (PSA)
testing for detecting prostate cancer
E) Health outcomes are improved by monitoring PSA velocity and doubling time
Item 3
ANSWER: C
The U.S. Preventive Services Task Force (USPSTF) concluded with moderate certainty that the harm from
screening for prostate cancer in men 75 or older outweighs the benefits. Potential harms include urinary
incontinence, erectile dysfunction, bowel dysfunction, and death, and the USPSTF found no direct
evidence of benefits from screening. The USPSTF also concluded that current evidence is insufficient to
assess the benefits versus harms of screening in men younger than age 75. Most major U.S. medical
organizations, including the American Academy of Family Physicians, recommend individualizing
decisions after discussions with the patient, and agree that the best candidates for screening are men age
50 or older with a life expectancy 10 years. Prostate-specific antigen (PSA) is more sensitive than the
digital rectal examination. Utilizing PSA velocity and doubling time has not been shown to improve health
outcomes.
5. You are treating an 18-year-old white male college freshman for allergic rhinitis. It is
September, and he tells you that he has severe symptoms every autumn that impair his academic
performance. He has a strongly positive family history of atopic dermatitis.
Which one of the following medications is considered optimal treatment for this condition?
A) Intranasal glucocorticoids
B) Intranasal cromolyn sodium
C) Intranasal decongestants
D) Intranasal antihistamines
Item 5
ANSWER: A
Topical intranasal glucocorticoids are currently believed to be the most efficacious medications for the
treatment of allergic rhinitis. They are far superior to oral preparations in terms of safety. Cromolyn
sodium is also an effective topical agent for allergic rhinitis; however, it is more effective if started prior
to the season of peak symptoms. Because of the high risk of rhinitis medicamentosa with chronic use of
topical decongestants, these agents have limited usefulness in the treatment of allergic rhinitis.
Azelastine, an intranasal antihistamine, is effective for controlling symptoms but can cause somnolence
and a bitter taste. Oral antihistamines are not as useful for congestion as for sneezing, pruritus, and
rhinorrhea. Overall, they are not as effective as topical glucocorticoids.
6. You are treating a 53-year-old female for a deep-vein thrombosis in her left leg. The use of
compression stockings for this problem has been shown to
A) increase the risk of pulmonary embolism
B) increase the level of pain
C) increase complications if used prior to completion of a course of anticoagulation therapy
D) decrease the risk of post-thrombotic syndrome
Item 6
ANSWER: D
Post-thrombotic syndrome (PTS) is a complication of acute deep-vein thrombosis (DVT), and is
characterized by chronic pain, swelling, and skin changes in the affected limb. Within 5 years of
experiencing a DVT, one in three patients will develop PTS. A Cochrane review identified three
randomized, controlled trials examining the use of compression therapy in patients diagnosed with a new
DVT. The use of elastic compression stockings was associated with a highly statistically significant
reduction in the incidence of PTS, with an odds ratio of 0.31 (confidence interval of 0.20–0.48). A
separate trial cited in the Cochrane review documented no increased incidence of pulmonary embolism,
and a reduction in pain and swelling in the treatment group.
7. Which one of the following is true concerning the use of dexamethasone to treat acute
laryngotracheitis (croup)?
A) A single dose is adequate for treatment
B) It commonly leads to a secondary bacterial infection due to immunosuppression
C) It increases the need for hospitalization
D) It is indicated only for patients with severe croup
Item 7
ANSWER: A
Treatment with corticosteroids is now routinely recommended for acute laryngotracheitis (croup). A single
dose of dexamethasone, either orally or intramuscularly, is appropriate. Prolonged courses of
corticosteroids provide no additional benefit and may lead to secondary bacterial or fungal infections.
Secondary infections rarely occur with single-dose treatment. Corticosteroid therapy shortens emergency
department stays and decreases the need for return visits and hospitalizations. It is indicated for patients
with croup of any severity.
8. In the development of clinical guidelines, which one of the following is rated as the strongest and
highest-quality evidence?
A) Evidence from randomized, placebo-controlled studies
B) Evidence from nonrandomized, double-blind, placebo-controlled studies
C) Evidence from nonrandomized, double-blind, crossover, placebo-washout–controlled
studies
D) Evidence obtained from well-designed cohort or case-control analytical studies from
more than one center or research group
E) Evidence based on reports of expert committees or opinions of respected authorities in
the appropriate specialty area
Item 8
ANSWER: A
Randomized, controlled studies yield stronger evidence than other types of studies, especially case-control
or cohort studies, because randomization provides the greatest safeguard against unanticipated study bias.
Evidence obtained from randomized, controlled studies is considered level 1 (strongest) by the U.S.
Preventive Services Task Force. Evidence obtained from nonrandomized, controlled studies is considered
level 2a; well-designed case-control and cohort studies are considered level 2b; and reports of expert
committees or respected authorities are considered level 3 (weakest).
9. A 55-year-old male who had a recent episode of atrial fibrillation that converted in the
emergency department is asymptomatic and currently in sinus rhythm. He is in good health
otherwise and has no history of hypertension, diabetes mellitus, heart failure, transient ischemic
attack, or stroke.
Which one of the following would be best for preventing a stroke in this patient?
A) Aspirin
B) Clopidogrel (Plavix), 75 mg daily
C) Warfarin (Coumadin), with a goal INR of 1.5–2.5
D) Warfarin, with a goal INR of 2.0–3.0
E) Warfarin, with a goal INR of 2.5–3.5
Item 9
ANSWER: A
2 The absolute rate of stroke depends on age and comorbid conditions. The stroke risk index CHADS , used
to quantify risk of stroke for patients who have atrial fibrillation and to aid in the selection of
antithrombotic therapy, is a mnemonic for individual stroke risk factors: C (congestive heart failure), H
(hypertension), A (age 75), D (diabetes mellitus), and S (secondary prevention for prior ischemic stroke
or transient attack—most experts include patients with a systemic embolic event). Each of these clinical
parameters is assigned one point, except for secondary prevention, which is assigned 2 points. Patients
are considered to be at low risk with a score of 0, at intermediate risk with a score of 1 or 2, and at high
risk with a score 3. Experts typically prefer treatment with aspirin rather than warfarin when the risk
2 of stroke is low. The patient in this question has a CHADS score of 0, which is low risk. Treatment with
aspirin is therefore appropriate.
10. Which one of the following serum proteins is typically DECREASED in a hospitalized patient
with sepsis?
A) Complement C3
B) Ferritin
C) C-reactive protein (CRP)
D) Albumin
E) Fibrinogen
Item 10
ANSWER: D
The acute phase response refers to the multiple physiologic changes that occur with tissue injury. The
synthesis of acute-phase proteins by hepatocytes is altered, leading to decreased serum levels of several
of these proteins, including albumin and transferrin. Serum levels rise for other proteins, such as
ceruloplasmin, complement proteins, haptoglobin, fibrinogen, and C-reactive protein. Serum levels of
ferritin may be extremely high in certain conditions, but are also influenced by total-body iron stores.
11. A 25-year-old female is concerned about recurrent psychological and physical symptoms that
occur during the luteal phase of her menstrual cycle and resolve by the end of menstruation. She
wants help managing these symptoms, but does not want to take additional estrogen or
progesterone.
Which one of the following management strategies is supported by the best clinical evidence?
A) Cognitive-behavioral therapy
B) Spironolactone during the luteal phase
C) Bright light therapy during the luteal phase
D) Evening primrose oil started 2–4 days prior to the luteal phase
E) Black cohosh
Item 11
ANSWER: B
Randomized, controlled trials found that luteal-phase spironolactone improved psychological and physical
symptoms of premenstrual syndrome over 2–6 months compared with placebo. Based on existing
evidence, the effectiveness is unknown for cognitive-behavioral therapy, bright light therapy, evening
primrose oil, and black cohosh.
12. A 55-year-old male is brought to the emergency department with a complaint of pain in the right
eye and reduced vision of about 10 minutes’ duration. His eye was injured while he was hitting
a metal stake with a sledge hammer. He was not wearing safety goggles. On examination you
note a subconjunctival hemorrhage completely surrounding the cornea. The iris is irregular.
Which one of the following is contraindicated prior to emergency transfer to an ophthalmologist?
A) Administering an analgesic
B) Attempting tonometry
C) A visual acuity test
D) Use of an eye shield
E) Administering an antiemetic
Item 12
ANSWER: B
The injury and findings described raise the possibility of globe rupture due to a fragment of steel
penetrating through the cornea and pupil and into the globe. Relief of pain with an analgesic is appropriate
before transfer. Because of a risk of extruding intraocular fluid, tonometry should not be attempted if
globe rupture is suspected. A rapid assessment of gross visual acuity (e.g., counting fingers, seeing light
versus dark) may be performed. An eye shield should be placed over the affected eye to avoid putting
pressure on the eye during transport to the ophthalmologist. Because the Valsalva effect from vomiting
may lead to extrusion of intraocular contents, an antiemetic would be appropriate before transfer as well.
14. Which one of the following injection sites for insulin administration is best for preventing
hypoglycemia in a 14-year-old male with diabetes mellitus who wishes to participate in track and
field running events?
A) Arm
B) Abdomen
C) Hip
D) Calf
E) Thigh
Item 14
ANSWER: B
The use of a nonexercised injection site for insulin administration, such as the abdomen, may reduce the
risk of exercise-induced hypoglycemia. If the leg is used as an injection site, exercise may accelerate
insulin absorption, resulting in increased levels of plasma insulin. However, leg exercise has no effect on
insulin disappearance from the arm and may actually reduce the rate of insulin disappearance from
abdominal injection sites. Compared with leg injection, arm or abdominal injection reduces the
hypoglycemic effect of exercise by approximately 60% and 90%, respectively.
15. Which one of the following is the best diagnostic test for vitamin D deficiency?
A) Ionized calcium
B) Serum phosphorus
C) 24-hour urine for calcium
D) 1,25-hydroxyvitamin D
E) 25-hydroxyvitamin D
Item 15
ANSWER: E
Undiagnosed vitamin D deficiency is not uncommon, and 25-hydroxyvitamin D is the barometer for
vitamin D status. Although there is no consensus on optimal levels of 25-hydroxyvitamin D as measured
in serum, vitamin D deficiency is defined by most experts as a 25-hydroxyvitamin D level of <20 ng/mL
(50 nmol/L).
16. A 69-year-old male presents with a 2-week history of fever, fatigue, weight loss, and mild
diarrhea. He is found to have a mildly tender mass in the left lower quadrant of the abdomen.
The most likely diagnosis is
A) Crohn’s disease
B) ulcerative colitis
C) celiac disease
D) diverticulitis
E) lymphoma
Item 16
ANSWER: D
Diverticulitis commonly affects the left lower quadrant in the elderly and may present as an abscess.
Crohn’s disease primarily affects the distal small intestine (regional enteritis), most typically in a young
person, and usually in the second or third decade of life. Ulcerative colitis usually presents with a longer
history and does not typically present with a mass. A 2-week history of a palpable mass is not a typical
presentation for lymphoma. Celiac disease does not cause a palpable left lower quadrant mass.
17. A 35-year-old white gravida 2 para 1 sees you for her initial prenatal visit. Since delivering her
first child 10 years ago, she has developed type 2 diabetes mellitus. She has kept her disease
1c under excellent control by taking metformin (Glucophage). A recent hemoglobin A level was
6.5%.
You should now treat her diabetes with
A) metformin
B) acarbose (Precose)
C) pioglitazone (Actos)
D) human insulin
Item 17
ANSWER: D
The safety of most oral hypoglycemics in pregnancy has not been established with regard to their
teratogenic potential. However, all oral agents cross the placenta (in contrast to insulin), leading to the
potential for severe neonatal hypoglycemia. For these reasons, plus the requirement for exquisitely tight
glucose control to reduce fetal macrosomia and organ dysgenesis, the American Diabetes Association
advocates the use of human insulin for pregnant women. Insulin requirements generally increase
throughout gestation, but the precise dosage is unimportant as long as it is sufficient to maintain glucose
control.
18. A 4-year-old male is brought to your office for evaluation of fever, coryza, and cough. On
examination, the child appears mildly ill but in no respiratory distress. His temperature is
37.4°C (99.3°F) and other vital signs are within the normal range. An HEENT examination
is significant only for light yellow rhinorrhea and reddened nasal mucous membranes. Lung
auscultation reveals good air flow with a few coarse upper airway sounds. While performing
the examination you note multiple red welts and superficial abrasions scattered on the chest and
upper back. When you question the parents, they tell you the marks are where “the sickness is
leaving his body,” and were produced by rubbing the skin with a coin.
This traditional healing custom is practiced principally by people from which geographic region?
A) Sub-Saharan Africa
B) Southeast Asia
C) The Middle East
D) Caribbean islands
E) Andean South America
Item 18
ANSWER: B
Coin rubbing is a traditional healing custom practiced primarily in east Asian countries such as Cambodia,
Korea, China, and Vietnam. The belief is that one’s illness must be drawn out of the body, and the red
marks produced by rubbing the skin with a coin are evidence of the body’s “release” of the illness. These
marks may be confused with abuse, trauma from some other source, or an unusual manifestation of the
illness itself.
19. A 12-year-old white male asthmatic has an acute episode of wheezing. You diagnose an acute
2 asthma attack and prescribe an inhaled $ -adrenergic agonist. After 2 hours of treatment, he
continues to experience wheezing and shortness of breath.
Which one of the following is the most appropriate addition to acute outpatient management?
A) Oral theophylline (Theo-Dur)
B) Oral corticosteroids
2 C) An oral $ -adrenergic agonist
D) Inhaled cromolyn (Intal)
E) Inhaled corticosteroids
Item 19
ANSWER: B
2 The treatment of choice for occasional acute symptoms of asthma is an inhaled  -adrenergic agonist such
as albuterol, terbutaline, or pirbuterol. If symptoms do not respond to -agonists, they should be treated
with a short course of systemic corticosteroids.
Theophylline has limited usefulness for treatment of acute symptoms in patients with intermittent asthma;
it is a less potent bronchodilator than subcutaneous or inhaled adrenergic drugs, and therapeutic serum
concentrations can cause transient adverse effects such as nausea and central nervous system stimulation
in patients who have not been taking the drug continuously.
Cromolyn can decrease airway hyperreactivity, but has no bronchodilating activity and is useful only for
prophylaxis. Inhaled corticosteroids should be used to suppress the symptoms of chronic persistent
2 asthma. Oral  -selective agonists are less effective and have a slower onset of action than the same drugs
given by inhalation.
19. A 12-year-old white male asthmatic has an acute episode of wheezing. You diagnose an acute
2 asthma attack and prescribe an inhaled $ -adrenergic agonist. After 2 hours of treatment, he
continues to experience wheezing and shortness of breath.
Which one of the following is the most appropriate addition to acute outpatient management?
A) Oral theophylline (Theo-Dur)
B) Oral corticosteroids
2 C) An oral $ -adrenergic agonist
D) Inhaled cromolyn (Intal)
E) Inhaled corticosteroids
Item 20
ANSWER: E
Panic disorder typically presents with the symptoms described, in late adolescence or early adulthood. The
attacks are sporadic and last 10–60 minutes. Generalized anxiety disorder is more common, and common
symptoms include restlessness, fatigue, muscle tension, irritability, difficulty concentrating, and sleep
disturbance. Patients with mitral valve prolapse usually have an abnormal cardiac examination.
Pheochromocytoma is associated with headache and hypertension, and usually occurs in thin patients.
Paroxysmal supraventricular tachycardia is usually not associated with gastrointestinal distress or
paresthesias.
21. A 45-year-old female with no family history of breast cancer presents for a routine examination.
She has no constitutional symptoms or obvious breast abnormalities, and a clinical breast
examination is negative.
U.S. Preventive Services Task Force recommendations advise that until 50 years of age the
patient should
A) perform monthly breast self-examinations
B) have mammography every 1–2 years
C) have scintimammography every 1–2 years
D) have breast ultrasonography annually
E) have a breast MRI every 1–2 years
Item 21
ANSWER: B
Previous recommendations for breast cancer screening included self-examination and clinical examination
on a regular basis. However, self-examination has been shown to be potentially harmful because it leads
to an increase in biopsies but does not decrease mortality; it is therefore no longer recommended.
Mammography is recommended every 1–2 years in women 40–50 years old, with digital mammography
being more sensitive. Ultrasonography and scintimammography are not recommended for screening, but
may be used in the evaluation of a palpable breast lesion. MRI is also not recommended as a routine
screening tool, but may be appropriate in high-risk women.
22. Which one of the following is most associated with falls in older adults?
A) Diphenhydramine (Benadryl)
B) Atorvastatin (Lipitor)
C) Metformin (Glucophage)
D) Memantine (Namenda)
E) Theophylline (Theo-24)
Item 22
ANSWER: A
Certain classes of medications are frequently associated with falls in older adults. These classes include
benzodiazepines, antidepressants, antipsychotics, antiepileptics, anticholinergics, sedative hypnotics,
muscle relaxants, and cardiovascular medications. Diphenhydramine is one of the anticholinergic
medications associated with falls in older adults. The other drugs listed are not in the higher-risk groups
of medications.
23. A 78-year-old white male is scheduled to undergo CT with contrast. His current diagnoses
include type 2 diabetes mellitus, heart failure, anemia of chronic disease, and renal insufficiency.
Evidence supports the use of which one of the following to reduce the risk of contrast-induced
nephropathy in this patient?
A) Intravenous furosemide
B) Ascorbic acid
C) Calcium antagonists
D) Isotonic bicarbonate infusion
E) High osmolar contrast media
Item 23
ANSWER: D
Prospective randomized trials examining the risk for contrast-induced nephropathy have identified
significant differences between contrast agents due to their physiochemical properties. Low-osmolar or
iso-osmolar contrast media should be used to prevent contrast-induced nephropathy in at-risk patients. The
volume of contrast medium should be as low as possible. Evidence also supports hydration before the
8
procedure, preferably with isotonic saline or isotonic sodium bicarbonate solution. There is limited
evidence that any pharmacologic intervention will prevent contrast-induced nephropathy.
24. A 70-year-old white female complains of two episodes of urinary incontinence. On both
occasions she was unable to reach a bathroom in time to prevent loss of urine. The first episode
occurred when she was in her car and the second while she was in a shopping mall. She is
reluctant to go out because of this problem.
The most likely cause of her problem is
A) overflow incontinence
B) stress incontinence
C) urge incontinence
D) functional incontinence
Item 24
ANSWER: C
At least 10 million Americans suffer from urinary incontinence. In the neurologically intact individual the
most common subtypes are stress incontinence, which occurs with coughing or lifting; urge incontinence,
which occurs when patients sense the urge to void but are unable to inhibit leakage long enough to reach
the toilet; and overflow incontinence, which occurs when the bladder cannot empty normally and becomes
overdistended. The term functional incontinence is applied to those cases where lower urinary tract
function is intact but other factors such as immobility and severe cognitive impairment lead to incontinence.
This patient has mild urge incontinence. The first approach to this problem should be behavioral. In a
mild case such as this, a cure can be expected, with success rates of 30%–90% in published studies. For
more severe cases, various pharmacologic agents, including anticholinergics, are useful. Failure of these
modalities should lead to urodynamic testing and consideration of surgery.
25. A 40-year-old obese African-American male presents with a history of excessive daytime
drowsiness. He readily falls asleep when reading or watching television. He admits to nearly
crashing his car twice in the past month because he briefly fell asleep behind the wheel. Most
frightening to the patient have been episodes characterized by sudden loss of muscle tone, lasting
about 1 minute, associated with laughing. An overnight sleep study shows decreased sleep
latency and no evidence of obstructive sleep apnea.
Appropriate treatment includes which one of the following?
A) Methylphenidate (Ritalin)
B) Zolpidem (Ambien) at bedtime
C) Carbidopa/levodopa (Sinemet)
D) Weight reduction
E) Avoidance of daytime napping
tem 25
ANSWER: A
The clinical history and laboratory findings presented are consistent with a diagnosis of narcolepsy.
Methylphenidate and other stimulant drugs remain the pharmacologic agents of choice in managing this
disorder. Since there is no evidence of obstructive sleep apnea, weight reduction would not be expected
to address his sleep problem. In general, sedatives, hypnotics, and alcohol should be avoided. Periodic
daytime naps may help to reduce symptoms.
26. A 42-year-old female presents to the emergency department with pleuritic chest pain. Her
probability of pulmonary embolism is determined to be low.
Which one of the following should be ordered to further evaluate this patient?
A) Brain natriuretic peptide (BNP)
B) CT pulmonary angiography
C) ELISA-based D-dimer
D) A cardiac troponin level
E) A ventilation-perfusion lung scan
Item 26
ANSWER: C
Patients who have a low or moderate pretest probability of pulmonary embolism should have d-dimer
testing as the next step in establishing a diagnosis.
27. A 34-year-old white male is brought to the emergency department following an automobile
accident in which he was the only occupant of the vehicle. He lost control of the vehicle and
hit a utility pole. He was knocked unconscious initially, but he is now awake and combative.
You note a strong smell of alcohol. He has a frontal hematoma approximately 3 cm in diameter
and an actively bleeding 4-cm laceration of the occiput. He will not permit you to examine him
further and he prepares to leave the emergency department.
You should
A) detain him in the emergency department
B) make him sign out against medical advice
C) tell him that he cannot return if he leaves
D) tell him that if he leaves he can return later
Item 27
ANSWER: A
Two of the most important ethical principles in medicine are respect for autonomy and beneficence.
Respect for autonomy means regarding patients as rightfully self-governing in matters of choice and action.
To make an autonomous decision, the patient must be mentally sound, have knowledge and understanding
of the facts, and be free of coercion. Beneficence means that physicians are motivated solely by what is
good for the patient. There are often ethical conflicts between these two principles. This particular patient
is clearly in need of further emergency treatment, but he refuses. He has had a significant head injury,
is combative and possibly intoxicated, and therefore cannot be considered mentally sound. The physician
should detain him for his own good and provide the appropriate care. Threatening the patient, having him
sign out against medical advice, or encouraging him to return later is not appropriate because his mentation
is impaired.
28. A 17-year-old soccer player presents for a preparticipation examination. His family history is
significant for the sudden death of his 12-year-old sister while playing basketball, and for his
mother and maternal grandmother having recurrent syncopal episodes. His medical history and
examination are completely normal.
Prior to approving his participation in sports, which one of the following is recommended?
A) A resting EKG
B) A stress EKG
C) An echocardiogram
D) Pulmonary function testing
E) No further evaluation
Item 28
ANSWER: A
A family history of sudden death and recurrent syncope is highly suspicious for genetic long-QT syndrome.
It is best diagnosed with a resting EKG that shows a QTc >460 msec in females and >440 msec in males.
This syndrome especially places young people at risk for sudden death. Management may include
-blockers, an implantable cardioverter-defibrillator, and no participation in competitive sports.
29. Which one of the following dietary supplements has the best evidence of efficacy in the treatment
of osteoarthritis of the knee?
A) Methylsulfonylmethane (MSM)
B) Glucosamine sulfate
C) Harpagophytum procumbens (devil’s claw)
D) Curcuma longa (turmeric)
E) Zingiber officinale (ginger)
Item 29
ANSWER: B
Glucosamine sulfate may be used to reduce symptoms and possibly slow disease progression in patients
with osteoarthritis of the knee (SOR B). Methylsulfonylmethane, devil’s claw, turmeric, and ginger are
not recommended because of insufficient evidence of their effectiveness.
30. A 24-year-old female has a history of mood swings over the past several months, which have
created marital and financial problems, in addition to jeopardizing her career as a television news
reporter.
You have made a diagnosis of bipolar disorder, and she has finally accepted the need for
treatment. However, she insists that you choose a drug that “won’t make me fat.”
Which one of the following would be best for addressing her concerns?
A) Aripiprazole (Abilify)
B) Olanzapine (Zyprexa)
C) Quetiapine (Seroquel)
D) Risperidone (Risperdal)
Item 30
ANSWER: A
All of the atypical antipsychotics are associated with some degree of weight gain. Of the choices listed,
aripiprazole is associated with the least amount of weight gain, generally less than 1 kilogram. The other
agents listed are likely to cause considerably more weight gain.
31. A 75-year-old white female presents with back pain of several months’ duration, which is
worsened by movement. A physical examination is unremarkable except for mild pallor. She
takes furosemide (Lasix) for hypertension.
Laboratory Findings
Hemoglobin.. . . . . . . . . . . . . . . . . . . . . 10.0 g/dL (N 12.0–16.0)
Serum creatinine.. . . . . . . . . . . . . . . . . . 2.0 mg/dL (N 0.6–1.5)
BUN. . . . . . . . . . . . . . . . . . . . . . . . . . 40 mg/dL (N 8–25)
Serum uric acid. . . . . . . . . . . . . . . . . . . 8.0 mg/dL (N 3.0–7.0)
Serum calcium. . . . . . . . . . . . . . . . . . . . 12.0 mg/dL (N 8.5–10.5)
Total serum protein. . . . . . . . . . . . . . . . . 9.8 g/dL (N 6.0–8.4)
Globulin. . . . . . . . . . . . . . . . . . . . . . . . 6.1 g/dL (N 2.3–3.5)
Albumin. . . . . . . . . . . . . . . . . . . . . . . . 3.7 g/dL (N 3.5–5.0)
Serum IgG.. . . . . . . . . . . . . . . . . . . . . . 3700 mg/dL (N 639–1349)
Urine. . . . . . . . . . . . . . . . . . . . . . . . . . positive for Bence-Jones protein
Which one of the following would be most appropriate at this point?
A) Repeat the physical examination and laboratory evaluation every 6 months
B) Discontinue the diuretic and repeat the laboratory evaluation in 1 month
C) Obtain a bone scan
D) Obtain a bone marrow examination
E) Begin therapy with tamoxifen (Soltamox), 20 mg daily
Item 31
ANSWER: D
This patient has typical symptoms and laboratory findings of multiple myeloma, which accounts for 1%
of all malignant diseases and has a mean age at diagnosis of 61 years. The diagnosis is confirmed by a
bone marrow examination showing >10% plasma cells in the marrow. The serum level of monoclonal
immunoglobulin is typically >3 g/dL. A bone scan is inferior to conventional radiography and should not
be used. Tamoxifen is indicated for the treatment of breast cancer, which is unlikely given the physical
and laboratory findings in this case.
32. A 26-year-old gravida 1 para 0 presents for a prenatal examination. She has two cats and
expresses concern about toxoplasmosis.
Which one of the following would be most appropriate for this patient?
A) Recommend that she avoid directly handling the cats’ litter box
B) Immunize the patient against toxoplasmosis
C) Prophylactically treat the cats with antibiotics
D) Screen the patient’s urine for Toxoplasma antigens
E) Screen the patient’s serum for Toxoplasma antibody
Item 32
ANSWER: A
There is no immunization against toxoplasmosis, and the use of antibiotics is limited to cases in which there
is known maternal infection with the protozoa. Screening pregnant women for seroconversion (not with
urine antigen testing) is controversial, and recommendations by various professional groups differ.
Currently, the American College of Obstetrics and Gynecology does not recommend routine screening
except in patients who are known to be HIV positive. However, because the infection is thought to be
passed primarily from undercooked meat or through infected animal feces, it is universally recommended
that pregnant women avoid direct contact with cats’ litter boxes. If avoidance is not possible, wearing
gloves when handling a litter box is recommended.
33. A 20-year-old nonsmoker presents to your office with a sudden onset of chest pain. You order
a chest radiograph, which shows a small (<15%) pneumothorax. He is in no respiratory distress
and vital signs are normal. Pulse oximetry shows a saturation of 98% on room air.
Which one of the following would be most appropriate initially?
A) CT of the affected lung
B) Analgesics and a follow-up visit in 48 hours
C) Chest tube insertion
D) Hospital admission and a repeat chest film in 24 hours
Item 33
ANSWER: B
Practice guidelines state that a patient without apparent lung disease who develops a spontaneous “small”
pneumothorax (<15% of lung volume) can be managed as an outpatient with analgesics and follow-up
within 72 hours. CT of the lung is needed in complicated cases, including patients with known lung
disease or recurrent pneumothoraces. A chest tube is required only when the pneumothorax involves
>15% of lung volume.
34. In which one of the following patients can a diagnosis of osteoporosis be made?
A) A 58-year-old female who fractured her wrist when she slipped and fell on her
outstretched hand onto a carpeted floor
B) A 62-year-old female who sustained a pelvic fracture in a motor vehicle accident
C) A 52-year-old female with a T-score of +2.5 on bone mineral density (BMD) testing
of her hip
D) A 67-year-old female with a T-score of –1.7 on BMD testing of her spine
E) A 72-year-old female with a T-score of –2.0 on BMD testing of her spine
Item 34
ANSWER: A
Osteoporosis is defined as a fragility or low-impact fracture, or as a spine or hip bone mineral density
(BMD) 2.5 standard deviations below the mean for young, healthy women. A fracture of the radius
caused by a fall from a standing position would be considered a low-impact fracture. A fracture resulting
from a motor vehicle accident would be considered a high-impact fracture, which is not diagnostic for
11
osteoporosis. A T-score of –2.5 or less is considered osteoporosis, a T-score between –1.0 and –2.5 is
considered osteopenia, and a T-score of –1.0 or higher is considered normal.
35. At a routine visit, a 40-year-old female asks about beginning an exercise regimen. She has a
family history of heart disease and hypertension. She currently has no medical problems, but
she is sedentary.
Which one of the following would be the most appropriate recommendation for this patient?
A) Fast walking for 30 minutes on 5 or more days per week
B) Jogging for 30 minutes every other day
C) Weight training once weekly
D) An exercise stress test prior to beginning exercise
E) A baseline EKG and rhythm strip
Item 35
ANSWER: A
This patient would benefit from exercise to prevent or delay the onset of heart disease and hypertension,
and to manage her weight. Exercise stress testing is not specifically indicated for this patient. Current
recommendations are for healthy adults to engage in 30 minutes of accumulated moderate-intensity physical
activity on 5 or more days per week.
36. A 56-year-old male with a history of nephrolithiasis presents with a complaint of right flank
pain. Further evaluation reveals a right ureteral calculus 4 mm in diameter. Laboratory tests
reveal a serum calcium level of 12.1 mg/dL (N 8.5–10.5), a normal albumin level, and normal
kidney and liver function tests. The patient takes no chronic medications.
Which one of the following is most likely to reveal the cause of this patient’s elevated calcium?
A) A 24-hour urine calcium level
B) A repeat serum calcium level in 4–6 weeks
C) A serum 25-hydroxyvitamin D level
D) A serum calcitonin level
E) A serum intact parathyroid hormone level
Item 36
ANSWER: E
Primary hyperparathyroidism and malignancy are the most common causes of hypercalcemia, accounting
for about 90% of cases. An intact parathyroid hormone (PTH) level should be obtained initially, as the
results will indicate what kind of additional evaluation is needed. Vitamin D and urine calcium studies are
useful in evaluating hypercalcemia, but a PTH level should be obtained first. It would not be appropriate
to wait for a repeat calcium level in 4–6 weeks, because this patient has nephrolithiasis and a calcium level
12 mg/dL, indicating a possible need for surgery or perhaps a malignancy. Calcitonin levels generally
are not necessary in the evaluation of hypercalcemia.
37. You are caring for a 70-year-old male with widespread metastatic prostate cancer. Surgery,
radiation, and hormonal therapy have failed to stop the cancer, and the goal of his care is now
symptom relief. He is being cared for through a local hospice. Over the past few days he has
been experiencing respiratory distress. His oxygen saturation is 94% on room air and his lungs
are clear to auscultation. His respiratory rate is 16/min.
Which one of the following would be best at this point?
A) Morphine
B) Oxygen
C) Albuterol (Proventil, Ventolin)
D) Haloperidol
Item 37
ANSWER: A
Dyspnea is a frequent and distressing symptom in terminally ill patients. In the absence of hypoxia, oxygen
is not likely to be helpful. Opiates are the mainstay of symptomatic treatment.
38. A 72-year-old white female is admitted to the hospital with her first episode of acute heart
failure. She has a history of hypertension treated with a thiazide diuretic. An echocardiogram
reveals no evidence of valvular disease and no segmental wall motion abnormalities. Left
ventricular hypertrophy is described, and the ejection fraction is 55%. Her pulse rate is 72
beats/min.
The most likely cause of her heart failure is
A) systolic dysfunction
B) diastolic dysfunction
C) hypertrophic cardiomyopathy
D) high-output failure
Item 38
ANSWER: B
Diastolic dysfunction is now recognized as an important cause of heart failure. It is due to left ventricular
hypertrophy as a response to chronic systolic hypertension. The ventricle becomes stiff and unable to relax
or fill adequately, thus limiting its forward output. The typical patient is an elderly person who has systolic
hypertension, left ventricular hypertrophy, and a normal ejection fraction (50%–55%).
39. A new drug treatment is shown to reduce the incidence of a complication of a disease by 50%.
If the usual incidence of this complication were 1% per year, how many patients with this
disease would have to be treated with this medication for 1 year to prevent one occurrence of
this complication?
A) 20
B) 50
C) 100
D) 200
E) 500
Item 39
ANSWER: D
Considering relative risk reduction without also considering the absolute rate can distort the importance
of a therapy. A useful way to assess the importance of a therapy is to determine the number needed to
treat to benefit one patient. To calculate this number, the percentage of absolute risk reduction of a
particular therapy is divided into 100. In the case in question, the absolute risk reduction is 0.5%
(0.5×.01). Thus, the number-needed-to-treat for the example cited is 200 (100/0.5).
40. You would recommend pneumococcal vaccine for which one of the following?
A) A 20-year-old male who smokes 1 pack of cigarettes daily
B) A 52-year-old male with type 2 diabetes mellitus who received pneumococcal vaccine
6 years ago
C) A 60-year-old male who is a long-term resident of a nursing home because of a previous
stroke, and who received pneumococcal vaccine at age 54
D) A 62-year-old male with chronic renal failure who received pneumococcal vaccine at
age 50 and age 55
E) A 71-year-old male with no medical problems who received pneumococcal vaccine at
age 65
Item 40
ANSWER: A
In October 2008 the Advisory Committee on Immunization Practices (ACIP) of the Centers for Disease
Control and Prevention recommended adding cigarette smoking to the list of high-risk conditions that are
indications for the 23-valent pneumococcal polysaccharide vaccine. All persons between the ages of 19
and 64 who smoke should receive this vaccine. One-time revaccination after 5 years is recommended for
persons with chronic renal failure, asplenia (functional or anatomic), or other immunocompromising
conditions.
The patient with chronic renal failure in this question has already received two immunizations. The
diabetic patient and the nursing-home resident have both received one immunization and should not receive
a second dose until age 65. The 71-year-old has already been immunized after age 65, and a repeat
immunization is not recommended.
41. A 64-year-old white male presents to the emergency department with a 48-hour history of left
lower quadrant pain. After a thorough history and a physical examination you conclude that the
patient has diverticulitis. The patient is allergic to metronidazole (Flagyl).
You recommend a clear-liquid diet, a follow-up visit with his primary care physician in 48
hours, and treatment with
A) amoxicillin
B) amoxicillin/clavulanate (Augmentin)
C) ciprofloxacin (Cipro)
D) doxycycline
E) azithromycin (Zithromax)
Item 41
ANSWER: B
An accepted regimen for outpatient treatment of diverticulitis is amoxicillin/clavulanate, 875 mg every 12
hours. The other regimens are not optimal treatments because they do not include anaerobic coverage.
42. An 86-year-old mildly demented male nursing-home resident rarely leaves the facility. He has
frequent fecal incontinence that is disturbing to both him and his family. He has diet-controlled
diabetes mellitus and hypertension, and a history of transurethral resection of the prostate
(TURP) for benign prostatic hypertrophy. An examination is remarkable only for an empty
rectum and no focal neurologic findings.
Which one of the following is the most likely cause of this patient’s fecal incontinence?
A) Decreased rectal sensation secondary to diabetes mellitus
B) Decreased rectal storage capacity
C) Internal sphincter weakness
D) Puborectalis weakness
E) Overflow
Item 42
ANSWER: E
Overflow incontinence is common in the institutionalized elderly, and is often due to constipating
medications. Reduced storage capacity is usually seen with inflammatory bowel disease. Mild diabetes
mellitus does not cause decreased rectal sensation, and puborectalis and internal sphincter weakness are
uncommon in males, as they usually result from vaginal delivery.
43. A previously healthy 18-month-old male is brought to your office with a 2-day history of cough
and fever. On examination the child has a temperature of 38.3°C (100.9°F), a respiratory rate
of 30/min, and mild retractions and mild wheezes bilaterally. Oxygen saturation is 90%.
The most appropriate initial management would be
A) azithromycin (Zithromax)
B) a short course of corticosteroids
C) aerosolized racemic epinephrine every 4 hours
D) postural drainage
E) a single treatment with aerosolized albuterol, continued only if there is a positive
response
Item 43
ANSWER: E
For patients with bronchiolitis, evidence supports a trial of an inhaled bronchodilator, albuterol, or
epinephrine, with treatment continued only if the initial dose proves beneficial. There is no evidence to
support the use of antibiotics unless another associated infection is present (e.g., otitis media). Neither
corticosteroids nor postural drainage has been found to be helpful.
44. A 13-year-old male presents with a 3-week history of left lower thigh and knee pain. There is
no history of a specific injury, and his past medical history is negative. He has had no fevers,
night sweats, or weight loss, and the pain does not awaken him at night. He tried out for the
basketball team but had to quit because of the pain, which was worse when he tried to run.
Which one of the following physical examination findings would be pathognomonic for slipped
capital femoral epiphysis?
A) Excessive forward passive motion of the tibia with the knee flexed
B) Lateral displacement of the patella with active knee flexion
C) Limited internal rotation of the flexed hip
D) Reduced hip abduction with the hip flexed
E) Inability to extend the hip past the neutral position
Item 44
ANSWER: C
Slipped capital femoral epiphysis (SCFE) typically occurs in young adolescents during the growth spurt.
Physical activity, obesity, and male gender are predisposing factors for the development of this condition,
in which the femoral head is displaced posteriorly through the growth plate. There is pain with physical
activity, most commonly in the upper thigh anteriorly, but one-third of patients present with referred lower
thigh or knee pain, which can make accurate and timely diagnosis more difficult.
The hallmark of SCFE on examination is limited internal rotation of the hip. Specific to SCFE is the even
greater limitation of internal rotation when the hip is flexed to 90°. No other pediatric condition has this
physical finding, which makes the maneuver very useful in children with lower extremity pain. Orthopedic
consultation is advised if SCFE is suspected.
Hip extension and abduction are also limited in SCFE, but these findings are nonspecific. The knee
findings in this patient are not associated with SCFE.
45. A 74-year-old male presents with a 4-day history of diarrhea that he had initially thought was
“a 24-hour virus.” He states that the onset of his illness included nausea, one episode of
vomiting, and profuse diarrhea. He has felt feverish and has been having abdominal cramps.
He does not recall eating anything unusual and has not traveled recently.
On examination he appears uncomfortable, but in no real distress. His oral temperature is
37.1°C (98.8°F), blood pressure 134/82 mm Hg, and pulse rate 100 beats/min. He has lost 4
kg (9 lb) since his last visit 2 months earlier. His abdomen is soft, with hyperactive bowel
sounds and mild diffuse tenderness on palpation. A CBC and basic metabolic profile are
normal.
Which one of the following is the most likely cause of this patient's illness?
A) Norwalk-like virus (Norovirus)
B) Shigella
C) Campylobacter
D) Escherichia coli O157:H7
E) Staphyloccocus aureus
Item 45
ANSWER: C
Campylobacter jejuni is one of the most common causes of bacterial foodborne illnesses, estimated to
affect 1 million Americans annually. Undercooked or improperly handled chicken is most often implicated
as the source; surveys have demonstrated that between 20% and 100% of all retail chicken sold in the
United States is contaminated. The infection is generally isolated and sporadic, occurs more frequently
14
at the extremes of age, is most common during the summer months, and affects males disproportionately.
Symptoms typically begin 2–5 days following exposure. Diarrhea is the predominant symptom, with a
lesser degree of nausea and vomiting. Up to 10 days is required for full recovery.
While Escherichia coli O157:H7 and Shigella may cause a similar illness, both generally present with
bloody diarrhea. E. coli O157:H7 is most often transmitted in contaminated undercooked beef, and
Shigella is usually spread in a fecal-oral pattern or via contaminated water. The peripheral WBC count
is typically increased substantially in shigellosis. Staphylococcus aureus produces an enterotoxin in food
that causes the onset of nausea, vomiting, and diarrhea within hours of ingestion and clears within 24–48
hours. Norovirus is a very common cause of acute viral gastroenteritis, usually with more vomiting than
diarrhea. It spreads person to person, and patients usually recover within 24 hours.
46. A 50-year-old female with significant findings of rheumatoid arthritis presents for a preoperative
evaluation for planned replacement of the metacarpophalangeal joints of her right hand under
general anesthesia. She generally enjoys good health and has had ongoing medical care for her
illness.
Of the following, which one would be most important for preoperative assessment of this
patient’s surgical risk?
A) Resting pulse rate
B) Resting oxygen saturation
C) Erythrocyte sedimentation rate
D) Rheumatoid factor titer
E) Cervical spine imaging
Item 46
ANSWER: E
While all of the options listed may have some value in evaluating the preoperative status of a patient with
long-standing rheumatoid arthritis, imaging of the patient’s cervical spine to detect atlantoaxial subluxation
would be most important for preventing a catastrophic spinal cord injury during intubation. In many cases,
cervical fusion must be performed before other elective procedures can be contemplated. Although
rheumatoid arthritis may influence oxygen saturation and the erythrocyte sedimentation rate, these tests
would not alert the surgical team to the possibility of significant operative morbidity and mortality. Resting
pulse rate and rheumatoid factor are unlikely to be significant factors in this preoperative scenario.
47. A 68-year-old white male with diabetes mellitus is hospitalized after suffering a right middle
cerebral artery stroke. A nurse in the intensive-care unit calls to advise you that his blood
pressure is 200/110 mm Hg.
You should
A) continue monitoring the patient
B) administer labetalol (Trandate)
C) administer nicardipine (Cardene)
D) administer nitroprusside (Nitropress)
E) administer nitroglycerin
Item 47
ANSWER: A
Current American Heart Association guidelines for blood pressure control in stroke patients advise
monitoring with no additional treatment for patients with a systolic blood pressure <220 mm Hg or a
diastolic blood pressure <120 mm Hg. The elevated blood pressure is thought to be a protective
mechanism that increases cerebral perfusion, and lowering the blood pressure may increase morbidity.
48. A 59-year-old white female has a blood pressure consistently at or above 140/90 mm Hg. Her
only other significant medical problem is diabetes mellitus, which is controlled by diet.
Which one of the following is the most clearly established advantage of angiotensin receptor
blockers (ARBs) when compared with ACE inhibitors in patients such as this?
A) Reduced risk of persistent cough
B) Reduced risk of headache
C) Reduced risk of heart failure
D) Improved control of blood pressure
E) Improved lipid profile
Item 48
ANSWER: A
In multiple studies, angiotensin receptor blockers (ARBs) have been shown to be less likely to cause a
chronic cough when compared with ACE inhibitors. Although this is not a life-threatening danger, it is
a side effect that can be persistent and lead to discontinuation of medication.
15
Angioedema, a more dangerous side effect, was thought to be ACE-inhibitor specific. However, it is rare
and there is not yet good evidence that ARBs are safer. There have been case reports of angioedema
associated with ARB use.
The incidence of headache is similar for the two drug classes. ARBs have not been proven superior to
ACE inhibitors in blood pressure control, effects on lipid profiles, or prevention of heart failure, and there
is substantially more data on ACE inhibitors for the prevention of heart failure and proteinuria.
49. Which one of the following is the most likely cause of chronic unilateral nasal obstruction in an
adult?
A) Nasal septal deviation
B) Foreign-body impaction
C) Allergic rhinitis
D) Adenoidal hypertrophy
Item 49
ANSWER: A
The most common cause of nasal obstruction in all age groups is the common cold, which is classified as
mucosal disease. Anatomic abnormalities, however, are the most frequent cause of constant unilateral
obstruction, with septal deviation being most common. Foreign-body impaction is an important, but
infrequent, cause of unilateral obstruction and purulent rhinorrhea. Mucosal disease is usually bilateral and
intermittent. Adenoidal hypertrophy is the most common tumor or growth to cause nasal obstruction,
followed by nasal polyps, but both are less frequent than true anatomic causes of constant obstruction.
50. Which one of the following is most consistent with a diagnosis of iron deficiency anemia?
A) Low iron-binding capacity
B) An elevated methylmalonic acid level
C) Increased serum ferritin
D) Reticulocytosis about 1 week after administration of iron
Item 50
ANSWER: D
In iron deficiency anemia, serum iron is low but iron-binding capacity is high. Serum ferritin is one-tenth
of normal. Bone marrow iron stores are depleted. Oral replacement, which is safer than parenteral
administration and more acceptable to patients, should raise the hemoglobin level by 0.2 g/dL/day. A
reticulocyte response should be seen in a week to 10 days unless factors such as a concomitant folic acid
deficiency prevent a full response to therapy.
Are self breast exams recommended
However, self-examination has been shown to be potentially harmful because it leads
to an increase in biopsies but does not decrease mortality; it is therefore no longer recommended.
Mammography is recommended every 1–2 years in women 40–50 years old,
Prolonged QTc what is abnl in males and females
QTc >460 msec in females and >440 msec in males
What works for knees
glucosamine
Which of the atypical antipschotics do you get the least amount of weight gain
abilify
How do you diangnose mulitple myeloma
bone marrow biopsy

The diagnosis is confirmed by a
bone marrow examination showing >10% plasma cells in the marrow. The serum level of monoclonal
immunoglobulin is typically >3 g/dL.
When do you place a chest tube for a pneumo
A chest tube is required only when the pneumothorax involves
>15% of lung volume.
Who should receive the 23-valent pneumococcal polysaccharide vac
cine. All persons between the ages of 19
and 64 who smoke should receive this vaccine. One-time revaccination after 5 years is recommended for
persons with chronic renal failure, asplenia (functional or anatomic), or other immunocompromising
conditions.
limitation of internal rotation when the hip is flexed to 90°.
Slipped capital femoral epiphysis
51. Which one of the following is the best INITIAL management for hypercalcemic crisis?
A) Intravenous furosemide
B) Intravenous pamidronate (Aredia)
C) Intravenous plicamycin (Mithramycin)
D) Intravenous saline
Item 51
ANSWER: D
The initial management of hypercalcemic crisis involves volume repletion and hydration. The combination
of inadequate fluid intake and the inability of hypercalcemic patients to conserve free water can lead to
calcium levels over 14–15 mg/dL. Because patients often have a fluid deficiency of 4–5 liters, delivering
1000 mL of normal saline during the first hour, followed by 250–300 mL/hour, may decrease the
hypercalcemia to less than critical levels (<13 mg/dL). If the clinical status is not satisfactory after
hydration alone, then renal excretion of calcium can be enhanced by saline diuresis using furosemide.

Intravenous pamidronate, a diphosphonate, reduces the hypercalcemia of malignancy and is best used in
the semi-acute setting, since calcium levels do not start to fall for 24 hours. The same is true for
intravenous plicamycin.
52. A healthy 24-year-old male presents with a sore throat of 2 days’ duration. He reports mild
congestion and a dry cough. On examination, his temperature is 37.2°C (99.0°F). His pharynx
is red without exudates, and there are no anterior cervical nodes. His tympanic membranes are
normal, and his chest is clear.
You would do which one of the following?
A) Treat with analgesics and supportive care
B) Treat with azithromycin (Zithromax)
C) Perform a throat culture and begin treatment with penicillin
D) Perform a rapid strep test
Item 52
ANSWER: A
The Centers for Disease Control and Prevention (CDC) assembled a panel of national health experts to
develop evidence-based guidelines for evaluating and treating adults with acute respiratory disease.
According to these guidelines, the most reliable clinical predictors of streptococcal pharyngitis are the
Centor criteria. These include tonsillar exudates, tender anterior cervical lymphadenopathy, absence of
cough, and history of fever. The presence of three or four of these criteria has a positive predictive value
of 40%–60%, and the absence of three or four of these criteria has a negative predictive value of 80%.
Patients with four positive criteria should be treated with antibiotics, those with three positive criteria
should be tested and treated if positive, and those with 0–1 positive criteria should be treated with
analgesics and supportive care only. This patient has only one of the Centor criteria, and according to the
panel should not be tested or treated with antibiotics.
53. A 28-year-old male is seen for follow-up of acute low back pain. He has a past history of
substance abuse. Ibuprofen and acetaminophen have helped some, but he is experiencing muscle
spasms.
It is best to avoid which one of the following when treating this patient’s problem?
A) Chlorzoxazone (Parafon Forte DSC)
B) Metaxalone (Skelaxin)
C) Cyclobenzaprine (Flexeril)
D) Methocarbamol (Robaxin)
E) Carisoprodol (Soma)
Item 53
ANSWER: E
There is limited data regarding the effectiveness of muscle relaxants in musculoskeletal conditions, but
strong evidence regarding their toxicity. Because the evidence for comparable effectiveness is weak, drug
selection should be based on patient preference, side-effect profile, drug interactions, and abuse potential.
Carisoprodol is metabolized to meprobamate, which is a class III controlled substance. It has been shown
to produce both physical and psychologic dependence.
54. A 2-year-old female is brought to the emergency department with a 2-day history of fever and
increasing redness on the left forearm. She is otherwise healthy. On examination her
temperature is 39.9°C (103.8°F), pulse rate 140 beats/min, and respiratory rate 42/min. She
is irritable, and the left forearm has a 4-cm erythematous, warm, tender area, with a fluctuant
area centrally. Her WBC count is 21,000/mm (N 4300–1 3 0,800), with 14% immature bands.
In addition to incision and drainage, which one of the following is the best initial treatment in
this patient?
A) Intravenous vancomycin
B) Intravenous ampicillin/sulbactam (Unasyn)
C) Intravenous nafcillin
D) Intravenous clindamycin (Cleocin)
E) No antibiotics
Item 54
ANSWER: A
This patient has systemic symptoms that suggest a severe underlying infection. Community-acquired
methicillin-resistant Staphylococcus aureus (CA-MRSA) should be considered the cause of this type of
infection until definitive cultures are obtained. CA-MRSA can cause aggressive infections in children,
especially in the skin and soft tissue. Incision and drainage of the abscess is necessary for treatment. In
a severe infection, vancomycin should be started initially until culture and sensitivities are available (SOR
B).
55. You test a patient’s muscles and find that his maximum performance consists of the ability to
move with gravity neutralized. This qualifies as which grade of muscle strength, on a scale of
5?
A) 0
B) 1
C) 2
D) 3
E) 4
Item 55
ANSWER: C
Muscle strength is scored on a scale of 0 to 5. The inability to contract a muscle is scored as 0.
Contraction without movement constitutes grade 1 strength. Movement with the effect of gravity
neutralized is grade 2 strength, while movement against gravity only is grade 3 strength. Movement
against gravity plus some additional resistance indicates grade 4 strength. Normal, or grade 5, strength
is demonstrated by movement against substantial resistance.
56. A 40-year-old male with acute pancreatitis has an alanine transaminase (ALT) level that is five
times normal. Which one of the following is the most likely diagnosis?
A) Gallstone pancreatitis
B) Pancreatic necrosis
C) Pancreatic pseudocyst
D) Hepatitis C
E) Alcohol-induced pancreatitis
Item 56
ANSWER: A
In this setting, a threefold or greater elevation of alanine transaminase has a positive predictive value of
95% for acute gallstone pancreatitis. High levels of C-reactive protein are associated with pancreatic
necrosis. Hepatitis C is identified by antibody detection or polymerase chain reaction testing. Other
markers are investigational.
57. You are asked to see a mentally challenged 45-year-old male from a nearby group home who
has groin pain. On examination you notice that he has large ears, a prominent jaw, and large
symmetric testicles.
These findings are consistent with
A) a variant form of Down syndrome
B) Asperger’s syndrome
C) Klinefelter’s syndrome
D) homocystinuria
E) fragile X syndrome
Item 57
ANSWER: E
Fragile X syndrome accounts for more cases of mental retardation in males than any other genetic disorder
except Down syndrome; about one in 4000–6000 males is affected. Down syndrome, Klinefelter’s
syndrome, and homocystinuria do not present with the described findings. Asperger’s syndrome is a
variant of autism in people of normal to high intelligence. Patients with Klinefelter’s syndrome usually
have small testicles.
58. According to the U.S. Preventive Services Task Force, multivitamin supplements in the geriatric
age group
A) are not recommended for prevention of any disorder
B) should be prescribed to reduce elevated homocysteine levels
C) decrease coronary atherosclerosis
D) decrease the incidence of lung cancer
E) decrease the incidence of colon cancer
Item 58
ANSWER: A
The U.S. Preventive Services Task Force makes no specific recommendations for vitamins or antioxidants
to prevent cancer or cardiovascular disease. Moreover, it makes no specific recommendations for vitamin
supplements for any condition.
59. Which one of the following antihypertensive drugs is most likely to cause ankle edema?
A) Hydrochlorothiazide
B) Amlodopine (Norvasc)
C) Lisinopril (Prinivil, Zestril)
D) Losartan (Cozar)
E) Atenolol (Tenormin)
Item 59
ANSWER: B
The most common side effects of calcium channel blockers, such as amlodipine, are due to vasodilation.
One result of this may be peripheral edema, but it can also cause dizziness, nausea, hypotension, cough,
and pulmonary edema. These problems may decrease with time, with reductions in dosage, or with the
addition of a diuretic or second calcium antagonist. Other classes of drugs are not associated with these
problems.
60. A 27-year-old Korean female consults you regarding several painful ulcers she has developed
in the vaginal area. Your examination reveals multiple 0.5-cm to 1.5-cm oval ulcers with
sharply defined borders and a yellowish-white membrane. She denies recent sexual activity.
Except for recurring aphthous ulcers of her mouth, her past history is unremarkable.
You obtain blood for a CBC and serology. A Tzanck smear and culture of her ulcer is negative
for herpes simplex virus. Two days later she returns to discuss her laboratory findings. She
draws your attention to a pustule with an erythematous margin at the site where the venipuncture
was done.
At this time the most likely diagnosis is
A) Reiter’s syndrome
B) Behçet’s syndrome
C) syphilis
D) mucocutaneous lymph node syndrome (Kawasaki disease)
E) AIDS
Item 60
ANSWER: B
The original description of Behçet’s syndrome included recurring genital and oral ulcerations and relapsing
uveitis. It is more common in Japan, Korea, and the Eastern Mediterranean area, and affects primarily
young adults. The cause is unknown. Two-thirds of patients will develop ocular involvement that may
progress to blindness. Patients may develop arthritis, vasculitis, intestinal manifestations, or neurologic
manifestations. This disease is also associated with cutaneous hypersensitivity; 60%–70% of patients will
develop a sterile pustule with an erythematous margin within 48 hours of an aseptic needle prick.
Reiter’s syndrome is not associated with genital ulcers. The ulcers of syphilis are characteristically
painless. Mucocutaneous lymph node syndrome (Kawasaki disease) primarily affects children under 6
years of age. While AIDS causes distinctive skin lesions, genital ulcers are not a common manifestation
of this disease.
61. A 40-year-old runner complains of gradually worsening pain on the lateral aspect of his foot.
He runs on asphalt, and has increased his mileage from 2 miles/day to 5 miles/day over the last
2 weeks. Palpation causes pain over the lateral 5th metatarsal. The pain is also reproduced
when he jumps on the affected leg. When you ask about his shoes he tells you he bought them
several years ago.
Which one of the following is the most likely diagnosis?
A) Ligamentous sprain of the arch
B) Stress fracture
C) Plantar fasciitis
D) Osteoarthritis of the metatarsal joint
Item 61
ANSWER: B
Running injuries are primarily caused by overuse due to training errors. Runners should be instructed to
increase their mileage gradually. A stress fracture causes localized tenderness and swelling in superficial
bones, and the pain can be reproduced by having the patient jump on the affected leg. Plantar fasciitis
causes burning pain in the heel and there is tenderness of the plantar fascia where it inserts onto the medial
tubercle of the calcaneus.
62. A 55-year-old male is brought to the emergency department because of confusion and seizures.
He has a history of hypertension and obstructive sleep apnea due to obesity. He is not conscious
and no other history is available. An examination shows no focal neurologic findings, but a
general examination is limited because of his size. Breath sounds are diminished, and heart
sounds are difficult to hear. He has venous insufficiency changes on his lower extremities, with
brawny-type edema. Laboratory testing reveals a sodium level of 116 mmol/L (N 135–145),
but normal renal and liver functions. A chest radiograph shows mild cardiomegaly. A BNP
level is pending, but immediate treatment is felt to be indicated.
Which one of the following is the treatment of choice for this patient?
A) Valsartan (Diovan)
B) Furosemide
C) Vasopressin (Pitressin)
D) Hypertonic saline
E) Conivaptan (Vaprisol)
Item 62
ANSWER: D
This patient has severe hyponatremia manifested by confusion and seizures, a life-threatening situation
warranting urgent treatment with hypertonic (3%) saline. The serum sodium level should be raised by only
1–2 mmol/L per hour, to prevent serious neurologic complications. Saline should be used only until the
seizures stop. Some authorities recommend concomitant use of furosemide, especially in patients who are
likely to be volume overloaded, as this patient is, but it should not be used alone.
The arginine vasopressin antagonist conivaptan is approved for the treatment of euvolemic or hypervolemic
hyponatremia, but not in patients who are obtunded or in a coma, or who are having seiz
63. A 29-year-old gravida 1 para 0 at 8 weeks gestation is concerned about Down syndrome. She
had a sibling with Down syndrome, and she and her spouse want to know what antenatal tests
are available to them.
Which one of the following has the best detection rate for Down syndrome in the first trimester
of pregnancy?
A) Serum $-hCG and pregnancy-associated plasma protein A (PAPP-A), with nuchal
translucency (combined screening)
B) Maternal serum levels of inhibin A, "-fetoprotein, unconjugated estriol, and $-hCG
(quadruple screening)
C) Ultrasonography
D) Chorionic villus sampling
E) Amniocentesis
Item 63
ANSWER: D
In today’s environment, there are multiple screening tools and tests to detect fetal aneuploidy. All pregnant
women, regardless of age, should be offered the opportunity to undergo some form or combination of
screening to detect fetal abnormalities (SOR B). Chorionic villus sampling can be offered at 10–13 weeks
gestation, and has a 97.8% detection rate for Down syndrome—the best detection rate of studies offered
in the first trimester (SOR C). Combined screening can be offered at 11–14 weeks gestation, and has a
78.7%–89% detection rate (SOR A). Although amniocentesis has the best detection rate of the options
listed (99.4%), it cannot be offered until 16–18 weeks gestation (SOR C). Quadruple screening is done
at 15–20 weeks gestation, and has a 67%–81% detection rate (SOR A); ultrasonography at 18–22 weeks
gestation has a 35%–79% detection rate (SOR C).
when is choroinic villi sampling done
Chorionic villus sampling can be offered at 10–13 weeks
gestation, and has a 97.8% detection rate for Down syndrome
When is combined screening done
Serum B-hCG and pregnancy-associated plasma protein A (PAPP-A), with nuchal
translucency (combined screening)

Combined screening can be offered at 11–14 weeks gestation, and has a
78.7%–89% detection rate
when is amnios done
Although amniocentesis has the best detection rate of the options
listed (99.4%), it cannot be offered until 16–18 weeks gestation
When is quad screening done
Quadruple screening is done
at 15–20 weeks gestation, and has a 67%–81% detection rate
64. A 72-year-old male presents to your clinic in atrial fibrillation with a rate of 132 beats/min. He
has hypertension, but no history of heart failure or structural heart disease. He is otherwise
healthy and active.
The best INITIAL approach to his atrial fibrillation would be
A) rhythm control with antiarrythmics and warfarin (Coumadin) only if he cannot be
consistently maintained in sinus rhythm
B) rhythm control with antiarrythmics and warfarin regardless of maintenance of sinus
rhythm
C) ventricular rate control with digoxin, and warfarin for anticoagulation
D) ventricular rate control with digoxin, and aspirin for anticoagulation
E) ventricular rate control with a calcium channel blocker or $-blocker, and warfarin for
anticoagulation
Item 64
ANSWER: E
Randomized, controlled trials have indicated that in most patients with atrial fibrillation, rate control is the
best initial management. Patients who were stratified to the rhythm control arm of these trials did not have
lower morbidity or mortality and were more likely to suffer from adverse drug effects and increased
hospitalizations. The most efficacious drugs for rate control are calcium channel blockers and -blockers.
Digoxin is less effective for rate control and its role should be limited to a possible additional drug for
those not controlled with a -blocker or calcium channel blocker, or for patients with significant left
ventricular systolic dysfunction.
In patients 65 years of age or older or with one or more risk factors for stroke, the best choice for
anticoagulation to prevent thromboembolic disease is warfarin. If rhythm control is successful and sinus
rhythm is maintained, the thromboembolic rate is equivalent to that seen with a rate control strategy. Thus,
the data suggests that patients managed with a rhythm control strategy should be maintained on
anticoagulation regardless of whether they are consistently in sinus rhythm.
65. A 7-year-old Hispanic female has a 3-day history of a fever of 40.0°C (104.0°F), muscle aches,
vomiting, anorexia, and headache. Over the past 12 hours she has developed a painless
maculopapular rash that includes her palms and soles but spares her face, lips, and mouth. She
has recently returned from a week at summer camp in Texas. Her pulse rate is 140 beats/min,
and her blood pressure is 80/50 mm Hg in the right arm while lying down.
Which one of the following is the most likely diagnosis?
A) Mucocutaneous lymph node syndrome
B) Leptospirosis
C) Rocky Mountain spotted fever
D) Scarlet fever
E) Toxic shock syndrome
Item 65
ANSWER: C
While all of the diagnoses listed are in the differential, the most likely is Rocky Mountain spotted fever
(RMSF) (SOR C). It occurs throughout the United States, but is primarily found in the South Atlantic and
south central states. It is most common in the summer and with exposure to tall vegetation (e.g., while
camping, hiking, or gardening), and is transmitted by ticks. The diagnosis is based on clinical criteria that
include fever, hypotension, rash, myalgia, vomiting, and headache (sometimes severe). The rash
associated with RMSF usually appears 2–4 days after the onset of fever and begins as small, pink,
blanching macules on the ankles, wrists, or forearms that evolve into maculopapules. It can occur
anywhere on the body, including the palms and soles, but the face is usually spared.
Mucocutaneous lymph node syndrome is a similar condition in children (usually <2 years old), but
symptoms include changes in the lips and oral cavity, such as strawberry tongue, redness and cracking of
the lips, and erythema of the oropharyngeal mucosa. Leptospirosis is usually accompanied by severe
cutaneous hyperesthesia. The patient with scarlet fever usually has prominent pharyngitis and a fine,
papular, erythematous rash. Toxic shock syndrome may present in a similar fashion, but usually in
postmenarchal females.
66. A 48-year-old male who weighs 159 kg (351 lb) is admitted to the hospital with a left leg deepvein
thrombosis and pulmonary embolism. Treatment is begun with enoxaparin (Lovenox).
Which one of the following would be most appropriate for monitoring the adequacy of
anticoagulation in this patient?
A) Anti-factor Xa levels
B) Activated partial thromboplastin time (aPTT)
C) Daily INRs
D) Daily factor VIII levels
Item 66
ANSWER: A
In severely obese patients (>330 lb) and those with renal failure, low molecular weight heparin therapy
should be monitored with anti-factor Xa levels obtained 4 hours after injection. Most other patients do not
need monitoring. The INR is used to monitor warfarin therapy, and the activated partial thromboplastin
time (aPTT) is used to monitor therapy with unfractionated heparin. Factor VIII levels are not used to
monitor anticoagulation therapy.
67. Under current guidelines, hospice programs are most likely to serve patients dying from
A) heart failure
B) COPD
C) severe dementia
D) multiple strokes
E) cancer
Item 67
ANSWER: E
The general requirement for enrolling an individual in hospice is that they have a terminal illness and an
estimated life expectancy of 6 months or less. Given these criteria, it is not surprising that over 40% of
hospice patients have a cancer diagnosis.

Cancer usually has a short period of obvious decline at the end and is predictable to a degree. Diseases
such as COPD, end-stage liver disease, and heart failure result in long-term disability with periodic
exacerbations, any one of which could result in death, but far less predictably. Those with severe dementia
or frailty often experience a dwindling course that is also difficult to predict.
68. A healthy 48-year-old bookkeeper who works in a medical office has a positive PPD on routine
yearly screening. Which one of the following would be most appropriate at this point?
A) A chest radiograph
B) A repeat PPD
C) Treatment with isoniazid and one other antituberculous drug for 12 months
D) Anergy testing
Item 68
ANSWER: A
Clinical evaluation and a chest radiograph are recommended in asymptomatic patients with a positive PPD
(SOR C). A two-step PPD is performed on those at high risk whose initial test is negative. Asymptomatic
patients with a positive PPD and an abnormal chest film should have a sputum culture for TB, but a culture
is not required if the chest film is negative. Persons with a PPD conversion should be encouraged to take
INH for 9 months with proper medical supervision. Patients with a negative PPD who are still at high risk
for TB, especially HIV-positive patients, could be evaluated for anergy, but it is not recommended at this
time.
69. Which one of the following decreases the absorption of orally administered calcium
supplements?
A) Taking calcium carbonate with meals
B) Taking calcium citrate with meals
C) Vitamin D supplementation
D) Proton pump inhibitors
Item 69
ANSWER: D
2 Long-term histamine H -blocker or proton pump inhibitor use is associated with decreased absorption of
calcium carbonate. Patients taking these medications who require calcium supplementation should use
calcium citrate to improve absorption. Calcium carbonate preparations should be given with a meal to
improve absorption. Vitamin D is important in calcium absorption.
70. A 16-year-old high-school football player plants his left foot to make a cut and feels his left leg
give way. He feels a pop in the knee, followed by acute pain. He is evaluated on the field, and
examination with the knee flexed 20° reveals that the tibia can be displaced farther anteriorly
than with the uninvolved knee.
Which one of the following conditions is most likely?
A) Patellar tendon rupture
B) Posterior cruciate ligament tear
C) Anterior cruciate ligament tear
D) Tibial plateau fracture
E) Patellar dislocation
Item 70
ANSWER: C
Anterior cruciate ligament (ACL) tears are the most common ligament injury requiring surgery. Females
have a significantly higher rate of ACL tears, with the majority of tears in both men and women occurring
without physical contact. In addition to the immediate problems, there is a significant increase in
premature osteoarthritis of the knee. Approximately 50% of patients with this injury develop osteoarthritis
in 10–20 years.
Findings that help make the diagnosis of ACL tear include a noncontact mechanism of injury, an audible
popping sound, early swelling of the joint, and the inability to participate in the game after the injury.
Many patients can walk normally and can perform such straight-plane activities as climbing stairs, biking,
or jogging

Physical examination using the Lachman test or pivot shift test can be used to further assess whether the
ligament is torn. MRI can be used to confirm the diagnosis, although it is not needed if the diagnosis is
clear from the history and examination.
The other conditions listed are also sports-related knee injuries, but have different mechanisms of injury
or physical findings. Patients with patellar tendon rupture are unable to fully extend their knee and
examination shows a palpable defect in the patellar ligament and a high-riding patella. While the
mechanism of injury in patients with posterior cruciate ligament tears may be similar to that of ACL injury,
the examination would show posterior rather than anterior displacement of the tibia when the knee is flexed
at 90° (the posterior drawer sign).
The mechanism of injury of tibial plateau fractures in a healthy young male generally involves a highenergy
collision causing a valgus force with axial loading. Patients with patellar dislocations have
symptoms similar to those of an ACL injury, including an audible crack or pop and the feeling of the knee
giving way after a twisting motion. Immediately following the injury, however, examination would show
an obvious deformity, but the patella may spontaneously relocate prior to the on-field exam. There would
be no instability on the Lachman maneuver.
71. The best available evidence supports which one of the following guidelines for discussing serious
illnesses?
A) Physicians should delay having a detailed discussion with the patient about the expected
prognosis of cancer until staging is completed
B) For patients who are ambivalent about knowing their prognosis, the discussion should
focus on optimal potential outcomes and providing hope, even if this is unrealistic
C) Physicians should delay discussions about palliative care until curative measures have
failed
D) Physicians should respect the family’s wishes regarding how much information to share
with the patient
Item 71
ANSWER: A
It is best to discuss prognosis after accurate cancer staging, when specific details about survival rates will
give a much clearer and more accurate picture. After assessing the patient’s readiness to receive
prognostic information, the physician should focus on communicating an accurate prognosis without giving
a false sense of hope. Using simultaneous-care models, physicians can provide palliative and curative care
at the same time. Physicians should initiate a discussion about the availability of coordinated, symptomdirected
services such as palliative care early in the disease process; as the disease progresses, patients
should transition from curative to palliative therapy. How much information to share with the patient
depends on the physician’s assessment of the patient’s level of understanding about the disease and how
much patients themselves want to know.
72. A 50-year-old male presents with a 1-day history of fever and chest pain. The chest pain is
worse when he is in a supine position and with deep inspiration, and improves when he leans
forward. He has no shortness of breath and has never had this problem before. His vital signs
are normal except for a temperature of 37.8°C (100.0°F). He has no other medical problems
or allergies, and takes no medications. An EKG reveals widespread ST-segment elevation,
upright T waves, and PR-segment depression. His troponin level is normal. An echocardiogram
is pending.
Which one of the following would be the most appropriate treatment for this patient?
A) Aspirin
B) Prednisone
C) Heparin
D) Enoxaparin (Lovenox)
aItem 72
ANSWER: A
This patient demonstrates classic clinical features of acute pericarditis. Although the EKG findings appear
specific for the early stages of pericarditis, myocardial infarction would also be included in the differential
diagnosis. However, unlike with acute pericarditis, the EKG in myocardial infarction typically
demonstrates ST elevation that is localized and convex, often has Q waves, and rarely shows PR-segment
depression. A friction rub can be heard in up to 85% of patients with acute pericarditis. An
echocardiogram is often performed to determine the type and amount of effusion.
23
Conventional therapy for acute pericarditis includes NSAIDs, such as aspirin and ibuprofen. Recent
studies demonstrate that adding colchicine to aspirin may be beneficial in reducing the persistence and
recurrence of symptoms.
73. An otherwise healthy 37-year-old male presents to your office with a 2-week history of redness
and slight irritation in his groin. On examination a tender erythematous plaque with mild scaling
is seen in his right crural fold. The area fluoresces coral-red under a Wood’s light.
Which one of the following would be the most appropriate treatment at this time?
A) Amoxicillin
B) Erythromycin
C) Ketoconazole
D) Nystatin (Mycostatin)
E) Triamcinolone (Kenalog)
aItem 73
ANSWER: B
The characteristics of this lesion, including coral-red fluorescence under a Wood’s light, suggests
Corynebacterium infection, which is associated with erythrasma. Tinea cruris caused by Microsporum
infection fluoresces green, while intertrigo and tinea cruris caused by Epidermophyton or Trichophyton
infections do not fluoresce. Erythromycin, either systemic or topical, is the treatment of choice.
74. An otherwise healthy 40-year-old male comes to your office for follow-up of elevated liver
enzymes on an insurance examination. He is 173 cm (68 in) tall and weighs 113 kg (250 lb)
(BMI 37.7 kg/m2). He says he drinks about two beers per week. Findings are normal on a
physical examination, except for a slightly enlarged liver. AST and ALT levels are twice the
upper limits of normal.
Which one of the following would be the most appropriate next step?
A) A liver biopsy
B) Ultrasonography of the liver
C) Colonoscopy
D) Testing for viral hepatitis
E) Repeat AST and ALT levels in 3 months
aItem 74
ANSWER: D
Nonalcoholic fatty liver disease is the most likely diagnosis in this patient, but hepatitis B and C should
be ruled out. The patient's alcohol consumption of less than two drinks per week makes alcoholic fatty
liver disease unlikely. A liver biopsy would not be appropriate at this time. Liver ultrasonography should
be considered after hepatitis B and C are ruled out. The patient is younger than the recommended
screening age for colonoscopy.
75. A 68-year-old African-American male with a history of hypertension and heart failure continues
to have shortness of breath and fatigue after walking only one block. He has normal breath
sounds, no murmur, and no edema on examination. His current medications include furosemide
(Lasix), 20 mg/day, and metoprolol extended-release (Toprol-XL), 50 mg/day. He previously
took lisinopril (Prinivil, Zestril), but it was discontinued because of angioedema. A recent
echocardiogram showed an ejection fraction of 35%.
Which one of the following would be most likely to improve both symptoms and survival in this
patient?
A) Valsartan (Diovan)
B) Metolazone (Zaroxolyn)
C) Digoxin
D) Verapamil (Calan, Isoptin)
E) Isosorbide/hydralazine (BiDil)
aItem 75
ANSWER: E
In patients with systolic heart failure, the usual management includes an ACE inhibitor and a -blocker.
Since this patient had angioedema with an ACE inhibitor, an angiotensin receptor blocker may cause this
side effect as well. Adding metolazone is generally not necessary unless the patient has volume overload
that does not respond to increased doses of furosemide. Digoxin may improve symptoms, but has not been
shown to increase survival. For patients who cannot tolerate an ACE inhibitor, especially African-
Americans, a combination of direct-acting vasodilators such as isorbide and hydralazine is preferred.
Verapamil has a negative inotropic effect and should not be used.
76. A 70-year-old white female comes to your office for an initial visit. She has taken levothyroxine
(Synthroid), 0.3 mg/day, for the last 20 years. Although a recent screening TSH was fully
suppressed at <0.1 :U/mL, she claims that she has felt “awful” when previous physicians have
attempted to lower her levothyroxine dosage.
You explain that a serious potential complication of her current thyroid medication is
A) adrenal insufficiency
B) carcinoma of the ovary
C) carcinoma of the thyroid
D) hip fracture
E) renal failure
aItem 76
ANSWER: D
Women older than 65 years of age who have low serum TSH levels, indicating physiologic
hyperthyroidism, are at increased risk for new hip and vertebral fractures. Use of thyroid hormone itself
does not increase the risk of fracture if TSH levels are normal.
77. A 34-year-old white male letter carrier has developed progressively worsening dysphagia for
liquids and solids over the past 3 months. He says that he has lost about 30 lb during that time.
On examination, you note that he is emaciated and appears ill. His pulse rate is 98 beats/min,
temperature 37.8°C (100.2°F), respiratory rate 24/min, and blood pressure 95/60 mm Hg. His
weight is 45 kg (99 lb) and his height is 170 cm (67 in). His dentition is poor, and there is
evidence of oral thrush. His mucous membranes are dry.
You palpate small posterior cervical and axillary nodes. The heart, lung, and abdominal
examinations are normal. You promptly consult a gastroenterologist, who performs upper
endoscopy, which reveals numerous small ulcers scattered throughout the esophagus with
otherwise normal mucosa.
As you continue to investigate, you take a more detailed history. Which one of the following
is most likely to be related to the patient’s problem?
A) Intravenous drug use
B) A family history of esophageal cancer
C) Chest pain relieved by nitroglycerin
D) Recent travel to Russia
aItem 77
ANSWER: A
A young man with weight loss, oral thrush, lymphadenopathy, and ulcerative esophagitis is likely to have
HIV infection. Intravenous drug use is responsible for over a quarter of HIV infections in the United
States. Esophageal disease develops in more than half of all patients with advanced infection during the
course of their illness. The most common pathogens causing esophageal ulceration in HIV-positive
patients include Candida, herpes simplex virus, and cytomegalovirus. Identifying the causative agent
through culture or tissue sampling is important for providing prompt and specific therapy.
77. A 34-year-old white male letter carrier has developed progressively worsening dysphagia for
liquids and solids over the past 3 months. He says that he has lost about 30 lb during that time.
On examination, you note that he is emaciated and appears ill. His pulse rate is 98 beats/min,
temperature 37.8°C (100.2°F), respiratory rate 24/min, and blood pressure 95/60 mm Hg. His
weight is 45 kg (99 lb) and his height is 170 cm (67 in). His dentition is poor, and there is
evidence of oral thrush. His mucous membranes are dry.
You palpate small posterior cervical and axillary nodes. The heart, lung, and abdominal
examinations are normal. You promptly consult a gastroenterologist, who performs upper
endoscopy, which reveals numerous small ulcers scattered throughout the esophagus with
otherwise normal mucosa.
As you continue to investigate, you take a more detailed history. Which one of the following
is most likely to be related to the patient’s problem?
A) Intravenous drug use
B) A family history of esophageal cancer
C) Chest pain relieved by nitroglycerin
D) Recent travel to Russia
aItem 78
ANSWER: D
The lesions described are nonbullous impetigo, due to either Staphylococcus aureus or Streptococcus
pyogenes. Topical antibiotics, such as mupirocin, but not compounds containing neomycin, are the
preferred first-line therapy for impetigo involving a limited area. Oral antibiotics are widely used, based
on expert opinion and traditional practice, but are usually reserved for patients with more extensive
impetigo or with systemic symptoms or signs. Penicillin V and hexachlorophene have both been shown
to be no more effective than placebo. Topical antibiotics have been shown to be as effective as
erythromycin, which has a common adverse effect of nausea.
79. A 25-year-old female at 31 weeks gestation presents to the labor wing with painful uterine
contractions every 3 minutes. On examination her cervix is 3 cm dilated and 50% effaced. Her
membranes are intact and fetal heart monitoring is reassuring. She is treated with tocolysis,
betamethasone, antibiotics, and intravenous hydration, and cultured for group B Streptococcus.
The neonatal intensive care unit is notified, but the contractions ease and eventually stop. After
2 days of observation, her cervix is unchanged and she is discharged home.
One week later, the patient presents with contractions for the last 8 hours. Her cervical findings
are unchanged. Her group B Streptococcus culture was negative.
Which one of the following would be the most appropriate next step in the management of this
patient?
A) Repeat tocolysis, betamethasone, antibiotics, and intravenous hydration
B) Betamethasone, antibiotics, and intravenous hydration only
C) Antibiotics and intravenous hydration only
D) Tocolysis only
E) Expectant management
aItem 79
ANSWER: E
The purpose of obstetric management of preterm labor before 34 weeks gestation is to allow time to
administer corticosteroids. Treatment does not substantially delay delivery beyond 1 week. Repeated
administration of corticosteroids does not confer more benefit than a single course. Antibiotics are
administered for prophylaxis of group B Streptococcus and are useful for delaying delivery if membranes
are ruptured. They do not add any benefit otherwise, even though subclinical amnionitis may be a
causative factor in many cases of preterm labor. Prolonged and repeated tocolysis is believed to be
harmful. Tocolysis would not be indicated in this patient because she has had no cervical change and is
therefore having preterm contractions, not preterm labor. Careful monitoring for fetal compromise,
25
consultation with obstetric colleagues, and neonatal intensive-care unit involvement should be part of
expectant management of preterm labor cases.
80. You have decided that in addition to the counseling she has been receiving for depression, a 12-
year-old female in your practice might benefit from an antidepressant medication. Which one
of the following has shown the most favorable risk-to-benefit ratio in children and adolescents?
A) Fluoxetine (Prozac)
B) Lithium
C) Amitriptyline
D) Venlafaxine (Effexor)
E) St. John’s wort
aItem 80
ANSWER: A
SSRIs have been shown to benefit children and adolescents with depression, but there are concerns
regarding their association with suicidal behavior. Fluoxetine seems to be the most favorable SSRI, and
is the only one recommended by the FDA for treatment of depression in children 8–17 years old. There
is limited or no evidence to support the use of lithium, venlafaxine, or St. John’s wort in children and
adolescents. Amitriptyline and other tricyclic antidepressants are ineffective in children and have limited
effectiveness in adolescents, and safety is an issue in both of these groups.
81. Fibromyalgia is characterized by tender trigger points
A) along the medial border of each scapula
B) bilaterally at the anatomic snuffbox
C) at the insertion of the Achilles tendon into the posterior heel
D) at the second and third web spaces on the plantar surface of the foot
aItem 81
ANSWER: A
The typical fibromyalgia trigger points lie along the medial scapula borders, as well as the posterior neck,
upper outer quadrants of the gluteal muscles, and medial fat pads of the knees. Tenderness of the anatomic
snuffbox, Achilles tendons, or web spaces of the toes would most likely be related to another diagnosis.
82. A 47-year-old female presents to your office with a complaint of hair loss. On examination she
has a localized 2-cm round area of complete hair loss on the top of her scalp. Further studies
do not reveal an underlying metabolic or infectious disorder.
Which one of the following is the most appropriate initial treatment?
A) Topical minoxidil (Rogaine)
B) Topical immunotherapy
C) Intralesional triamcinolone (Kenalog)
D) Oral finasteride (Proscar)
E) Oral spironolactone (Aldactone)
aItem 82
ANSWER: C
These findings are consistent with alopecia areata, which is thought to be caused by a localized
autoimmune reaction to hair follicles. It occasionally spreads to involve the entire scalp (alopecia totalis)
or the entire body (alopecia universalis). Spontaneous recovery usually occurs within 6–12 months,
although areas of regrowth may be pigmented differently. Recovery is less likely if the condition persists
for longer than a year, worsens, or begins before puberty.
The initial treatment of choice for patients older than 10 years of age, in cases where alopecia areata affects
less than 50% of the scalp, is intralesional corticosteroid injections. Minoxidil is an alternative for children
younger than 10 years of age or for patients in whom alopecia areata affects more than 50% of the scalp.
While topical immunotherapy is the most effective treatment for chronic severe alopecia areata, it has the
potential for severe side effects and should not be used as a first-line agent.
Finasteride inhibits 5-reductase type 2, resulting in a decrease in dihydrotestosterone levels, and is used
in the treatment of androgenic alopecia (male-pattern baldness). Similarly, spironolactone is sometimes
used for androgenic alopecia because it is an aldosterone antagonist with antiandrogenic effects.
83. The preferred antibiotic treatment for community-acquired pneumonia in a young adult in the
ambulatory setting is
A) trimethoprim/sulfamethoxazole (Bactrim, Septra)
B) cephalexin (Keflex)
C) azithromycin (Zithromax)
D) penicillin V
E) ciprofloxacin (Cipro)
aItem 83
ANSWER: C
In a young adult with community-acquired pneumonia who is not sick enough to be hospitalized, the
current recommendation is to empirically treat with a macrolide antibiotic such as azithromycin. This
covers the atypical organism Mycoplasma pneumoniae, which is one of the most common causes of
community-acquired pneumonia. Certain fluoroquinolones such as levofloxacin also cover atypical causes,
but ciprofloxacin does not. The other antibiotics listed are also ineffective against Mycoplasma.
84. Which one of the following is a risk factor for intermittent claudication?
A) Hyperthyroidism
B) Hypercalcemia
C) Diabetes mellitus
D) Hypogonadism
E) Elevated angiotensin-converting enzyme
aItem 84
ANSWER: C
Diabetes mellitus and cigarette smoking are significant risk factors for intermittent claudication, as are
hypertension and dyslipidemia.
Hyperthyroidism, hypercalcemia, and hypogonadism are not closely associated with intermittent
claudication. Elevation of angiotensin-converting enzyme occurs with sarcoidosis.
85. The most common presenting symptom of obstructive sleep apnea is
A) excessive daytime sleepiness
B) snoring
C) morning headache
D) gastroesophageal reflux
E) enuresis
aItem 85
ANSWER: A
The most common presenting symptom of obstructive sleep apnea is excessive daytime sleepiness (SOR
A). Other symptoms include snoring, unrefreshing or restless sleep, witnessed apneas and nocturnal
choking, morning headache, nocturia or enuresis, gastroesophageal reflux, and reduced libido.
86. Which one of the following is recommended in the treatment of all four stages of COPD, from
mild through very severe?
A) Scheduled oral mucolytics such as N-acetylcysteine (Mucomyst)
B) Scheduled inhaled corticosteroids such as fluticasone (Flovent HFA)
C) Scheduled long-acting inhaled bronchodilators such as salmeterol (Serevent)
D) Scheduled long-acting anticholinergics such as tiotropium (Spiriva)
2 E) Short-acting inhaled $ -agonists such as albuterol (Ventolin HFA), as needed for
dyspnea
aItem 86
ANSWER: E
Short-acting bronchodilators such as albuterol and ipratropium are recommended on an as-needed basis
for treatment of breathlessness in stage I (mild) COPD. They are also recommended for as-needed use
in stage II (moderate), stage III (severe), and stage IV (very severe) COPD. Long-acting bronchodilators
such as salmeterol or tiotropium are recommended for stages II, III, and IV. Inhaled corticosteroids are
recommended for stages III and IV. Mucolytics can be considered for stages III and IV.
87. A 50-year-old female complains of a 6-month history of the insidious onset of right shoulder
pain and decreased range of motion. She does not respond to consistent use of prescriptionstrength
anti-inflammatory medication. Radiographs are negative.
Treatment of this patient’s condition should include
A) physical therapy with home exercises
B) early surgical referral
C) a short course of oral methylprednisolone
D) corticosteroid injection of the acromioclavicular joint
aItem 87
ANSWER: A
This patient most likely has either adhesive capsulitis or a degenerative rotator cuff tendinopathy. It is
important to rule out osteoarthritis with radiographs. Treatment typically includes NSAIDs, subacromial
cortisone injections, and physical therapy. These problems take months to treat and should not be referred
quickly for surgical evaluation, unless the diagnosis is in question.
88. In a patient with symptoms of thyrotoxicosis and elevated free thyroxine (T ), the presence of
thyroid TSH receptor site antibodies would indicate which one of the following as the cause of
thyroid gland enlargement?
A) Toxic multinodular goiter
B) Toxic adenoma
C) Hashimoto’s (lymphadenoid) thyroiditis
D) Subacute (giant cell) thyroiditis
E) Graves’ disease
aItem 88
ANSWER: E
When there is a question about the etiology of goiter and thyrotoxicosis, the presence of thyroid TSH
receptor immunoglobulins would indicate the presence of Graves’ disease, which is considered an
autoimmune disease. The prevalence of specific forms of TSH receptor site antibodies can distinguish
Graves’ disease from Hashimoto’s disease. Both are autoimmune diseases, but in Graves’ disease there
is a predominance of TSH receptor antibodies. In Hashimoto’s disease TSH receptor–blocking antibodies
are more predominant. These immunoglobulins tend to disappear during therapy.
89. In assessing the nutritional status of an infant it is useful to know that birth weight is expected
to be regained within
A) 5 days
B) 14 days
C) 21 days
D) 28 days
aItem 89
ANSWER: B
A helpful guideline for assessing normal growth in the very young infant is that birth weight should be
regained within 14 days.
90. The advance directive specifications contained in an individual’s living will become effective
A) at the time it is signed and witnessed
B) when it is confirmed by the individual’s health care surrogate
C) at the time of admission to a health care facility such as a hospital
D) when the patient develops a terminal illness
E) when the individual becomes unable to communicate health care wishes
aItem 90
ANSWER: E
The living will, a written advance directive, allows a competent person to indicate his or her health care
preferences while cognitively and physically healthy. A living will may list medical interventions the
patient would prefer to have withheld or withdrawn when he or she becomes unable to communicate.
91. A 3-year-old male is carried into the office by his mother. Yesterday evening he began
complaining of pain around his right hip. Today he has a temperature of 37.6°C (99.7°F), cries
when bearing weight on his right leg, and will not allow the leg to be moved in any direction.
A radiograph of the hip is normal.
Which one of the following would be most appropriate at this time?
A) A CBC and an erythrocyte sedimentation rate
B) A serum antinuclear antibody level
C) Ultrasonography of the hip
D) MRI of the hip
E) In-office aspiration of the hip
aItem 91
ANSWER: A
This presentation is typical of either transient synovitis or septic arthritis of the hip. Because the conditions
have very different treatment regimens and outcomes, it is important to differentiate the two. It is
recommended that after plain films, the first studies to be performed should be a CBC and an erythrocyte
sedimentation rate (ESR). Studies have shown that septic arthritis should be considered highly likely in
a child who has a fever over 38.7°C (101.7°F), refuses to bear weight on the leg, has a WBC count
>12,000 cells/mm , and has an ESR >40 mm/hr. If several or all of these conditions exist, aspiration 3
of the hip guided by ultrasonography or fluoroscopy should be performed by an experienced practitioner.
MRI may be helpful in cases that are unclear based on standard data, or if other etiologies need to be
excluded.
92. A 55-year-old male is found to have three hyperplastic polyps on a routine screening
colonoscopy. He has no personal or family history of colon cancer.
This patient’s next colonoscopy should be in
A) 1 year
B) 3 years
C) 5 years
D) 10 years
aItem 92
ANSWER: D
Colonoscopy is the gold standard for screening for colon cancer. Because of differences in recommended
screening intervals, the American Cancer Society and the U.S. Multi-Society Task Force on Colorectal
Cancer issued recommendations for follow-up in 2006 to bring some uniformity to the guidelines.
Patients with hyperplastic polyps are considered to have normal colonoscopy findings and can be followed
up in 10 years, unless they have hyperplastic polyposis syndrome. Patients with one or two small
adenomas (<1 cm, with no- or low-grade dysplasia) are considered at low risk and can be followed up
in 5–10 years, depending on family history, previous colonoscopy findings, and patient and physician
preference.
Patients with three or more small adenomas, or one adenoma >1 cm in size should be followed up in 3
years if the adenomas are completely removed. Patients who have had a sessile adenoma removed
piecemeal should have repeat colonoscopy in 2–6 months to make sure that the polyp has been completely
removed. Other factors that influence the screening interval include the quality of the preparation and the
ability of the physician to see the entire colon.
Although this patient had three hyperplastic polyps removed, he is at low risk for colon cancer and should
have repeat screening at the normal 10-year interval.
93. The parents of a 7-year-old male ask you to evaluate him because of increasing concerns about
his temper tantrums over the past 9 months. He often becomes angry and hostile, argues with
them constantly, and refuses to follow rules or directions. A major source of difficulty is his
refusal to quit playing with his toys when he is asked to come to the dinner table. After the child
ignored repeated attempts to get him to come to the table a few nights ago, the father became
frustrated and told him he had lost his television privileges. In response, the child became
aggressive and destructive, breaking his toys and sweeping his dinner plate and glass of milk
onto the floor. The parents describe many similar scenarios at bedtime, bath time, and when
he is getting dressed. They believe that their son is deliberately behaving this way to annoy
them.
This history is most consistent with
A) attention-deficit/hyperactivity disorder
B) bipolar disorder
C) conduct disorder
D) oppositional defiant disorder
E) normal childhood individualization
aItem 93
ANSWER: D
This child meets the DSM-IV criteria for oppositional defiant disorder, defined as a pattern of negativistic,
hostile, and defiant behavior lasting at least 6 months. The child will often lose his or her temper, argue
with adults, actively defy or refuse to comply with adults’ requests or rules, deliberately annoy people,
blame others for his or her mistakes or misbehavior, be easily annoyed by others, appear angry and
29
resentful, or be spiteful or vindictive. At least four of these behaviors must be present to meet the criteria
for diagnosis. The disturbance in behavior must also cause clinically significant impairment in social,
academic, or occupational functioning, and the behaviors must not occur exclusively during the course of
a psychotic or mood disorder. Meeting the criteria for conduct disorder excludes the diagnosis of
oppositional defiant disorder. If the individual is 18 years of age or older and meets the criteria for
antisocial personality disorder, then oppositional defiant disorder is excluded.
94. In patients with type 2 diabetes mellitus, intensive glycemic control has not been shown to be
beneficial for which one of the following diabetic complications?
A) Peripheral neuropathy
B) Foot infections
C) Cardiovascular disease
D) Proliferative retinopathy
E) Nephropathy
aItem 94
ANSWER: C
Intensive management of hyperglycemia, with a goal of achieving nondiabetic glucose levels, helps reduce
microvascular complications such as retinopathy, nephropathy, and neuropathy. Foot infections are less
common in patients without neuropathy and in patients with good glycemic control. Intensive management
of hyperglycemia also has a beneficial effect on cardiovascular disease in patients with type 1 diabetes
mellitus but, unfortunately, not in patients with type 2 diabetes mellitus. In fact, there is data to suggest
1c that intensive glycemic control (hemoglobin A <6.5) may be detrimental in certain populations, such as
the elderly and those with cardiovascular disease.
95. A 52-year-old male requests “everything you’ve got” to help him stop smoking. You review
common barriers to quitting and the benefits of cessation with him, and develop a plan that
includes follow-up. He chooses to start varenicline (Chantix) to assist with his efforts, and asks
about also using nicotine replacement.
Which one of the following would be accurate advice?
A) Combining these medications has not proven to be beneficial
B) The addition of transdermal nicotine, but not nicotine gum, has proven benefits
C) The combination is highly efficacious
D) Nicotine replacement doses need to be doubled in a patient taking varenicline
E) The combination of nicotine and varenicline is potentially lethal
aItem 95
ANSWER: A
Varenicline works by binding to nicotine receptors in the brain, providing much lower stimulation than
nicotine itself would. This has the effect of reducing the reinforcement and reward that smoking provides
to the brain. However, this medication also blocks the benefit a patient would receive from nicotine
replacement products. Studies have shown that using nicotine replacement products concurrently with
varenicline leads to an increase in nausea, headaches, dizziness, and fatigue.
96. Which one of the following is more likely to occur with glipizide (Glucotrol) than with
metformin (Glucophage)?
A) Lactic acidosis
B) Hypoglycemia
C) Weight loss
D) Gastrointestinal distress
aItem 96
ANSWER: B
Metformin is a biguanide used as an oral antidiabetic agent. One of its main advantages over some other
oral agents is that it does not cause hypoglycemia. Lactic acidosis, while rare, can occur in patients with
renal impairment. In contrast to most other agents for the control of elevated glucose, which often cause
weight gain, metformin reduces insulin levels and more frequently has a weight-maintaining or even a
weight loss effect. Gastrointestinal distress is a common side effect of metformin, particularly early in
therapy.
97. Typically, a high-grade squamous intraepithelial lesion (HSIL) of the cervix is treated with
ablation or excision. In which one of the following can treatment be deferred?
A) Adolescents
B) Patients attempting to conceive
C) Patients with a history of three previous normal Papanicolaou smears
D) Patients with a negative DNA test for HPV
E) Patients over the age of 70
aItem 97
ANSWER: A
Patients attempting to conceive are not candidates for conservative management of cervical dysplasia,
because treatment of progressive disease during pregnancy may be harmful. When possible, the problem
should be resolved before conception. Patients who have had three normal Papanicolaou (Pap) smears in
succession are candidates for lengthened screening intervals according to some recommendations.
However, once a problem is found, they should be managed the same as other cases.
A negative test for HPV can be used to assess the risk of patients with atypical squamous cells of
undetermined significance (ASC-US) or a low-grade squamous intraepithelial lesion (LSIL); it does not
change the management of patients with a high-grade intraepithelial lesion (HSIL). HPV infection is
common and transient in most young women in their first few years of sexual activity. With careful
follow-up, they can be observed rather than treated for HSIL. Patients over 70 years of age no longer
require screening if they have a long history of normal Pap smears, but when an abnormality is found it
should be treated.
98. A 14-year-old female is brought to your office by her parents because of concerns regarding her
low food intake, excessive exercise, and weight loss. Her weight is less than 75% of ideal for
her height.
Which one of the following sets of additional findings would indicate that the patient suffers
from severe anorexia nervosa?
A) Hypertension, tachycardia, and hyperthermia
B) Hypertension, tachycardia, and hypothermia
C) Hypotension, tachycardia, and hypothermia
D) Hypotension, bradycardia, and hyperthermia
E) Hypotension, bradycardia, and hypothermia
aItem 98
ANSWER: E
Characteristic vital signs in patients with severe anorexia nervosa include hypotension, bradycardia, and
hypothermia. Criteria for hospital admission include a heart rate <40 beats/min, blood pressure <80/50
mm Hg, and temperature <36°C (97°F). Increased cardiac vagal hyperactivity is thought to cause the
bradycardia.
99. A 12-year-old male presents for a routine physical examination prior to entering middle school.
He has received all childhood vaccines at the recommended time. Other than influenza vaccine,
he has not received any vaccines since entering kindergarten at age 6.
Which one of the following is recommended at this time?
A) Tdap, quadrivalent meningococcal conjugate vaccine (MCV4), and quadrivalent HPV
vaccine
B) Td and MCV4
C) Tdap and MCV4
D) Tdap only
E) No vaccinations are recommended at this age
aItem 99
ANSWER: C
Three vaccines are recommended for adolescents ages 11–12 years: Tdap, quadrivalent meningococcal
conjugate vaccine (MCV4), and quadrivalent HPV. However, the HPV vaccine is recommended and FDA
approved only for females.
100. A 55-year-old female has severe symptoms of gastroesophageal reflux disease. Upper
endoscopy with a biopsy shows severe esophagitis and Barrett’s esophagus.
Which one of the following is true regarding this patient?
A) The severity of her symptoms is due to the presence of Barrett’s esophagus
B) Follow-up screening endoscopy will reduce her risk of death from esophageal cancer
C) Her risk of developing esophageal adenocarcinoma is >90%
D) Her risk of developing esophageal adenocarcinoma is <1%
aItem 100
ANSWER: D
The actual risk of adenocarcinoma from Barrett’s esophagus is less than 1%. Endoscopy does nothing to
reduce the risk of death. Patients with Barrett’s esophagus can have minimal symptoms.
101. A 45-year-old male presents with a complaint of recent headaches. He has had four headaches
this week, and his description indicates that they are moderate to severe, bilateral, frontal, and
nonthrobbing. There is no associated aura. He has had similar episodes of recurring headaches
in the past.
Based on this limited history, which one of the following headache types can be eliminated from
the differential diagnosis?
A) Tension-type headache
B) Sinus headache
C) Migraine headache
D) Cluster headache
E) Headache of intracranial neoplasm
aItem 101
ANSWER: D
Cluster headache can be removed from the differential because it is always unilateral, although the affected
side can vary. The remainder of these headache types can be bilateral, frontal, and nonthrobbing. Brain
tumor headaches may be similar in character to previous headaches, but are often more severe or frequent.
102. In which one of the following scenarios is a physician most likely to be protected by a Good
Samaritan statute?
A) Assisting flight attendants with the care of a fellow passenger who develops respiratory
distress while in flight over the United States
B) Attending to an unconscious player while acting as an unpaid volunteer physician at a
high-school football game
C) Attending to a bicyclist with heat exhaustion while volunteering at a first-aid station
during a fund-raising ride
D) Attending to the family member of a patient who slips and falls in the waiting room at
the physician’s office
E) Attending to a nurse’s aide who collapses while the physician is staffing the hospital
emergency department
aItem 102
ANSWER: A
Generally, Good Samaritan laws apply to situations in which the physician does not have a preexisting duty
to provide care to the patient. A physician who volunteers as a standby health care provider at an event
assumes a duty to care for illness or injury in the participants. Likewise, physicians have a duty to provide
emergency care to a person in need within a facility where they are working, such as a medical office or
an emergency department. On an airplane, there is no preexisting duty for a physician to attend to a fellow
passenger who becomes ill. In addition, a specific federal law, the Aviation Medical Assistance Act,
ensures that physicians have Good Samaritan protection if they provide medical assistance while in flight
over the United States.
103. A 69-year-old female sees you for an annual examination. She asks you to look at her toes, and
you note a fungal infection in five toenails. She says the condition is painful and limits her
ability to complete her morning walks. She asks for treatment that will allow her to resume her
daily walks as soon as possible. Her only other medical problem is allergic rhinitis which is well
controlled.
Which one of the following would be the most appropriate treatment for this patient?
A) Oral griseofulvin ultramicrosize (Gris-PEG) daily for 12 weeks
B) Oral terbinafine (Lamisil) daily for 12 weeks
C) Topical terbinafine (Lamisil AT) daily for 12 weeks
D) Topical ciclopirox (Penlac Nail Lacquer) daily for 12 weeks
E) Toenail removal
aItem 103
ANSWER: B
Continuous therapy with oral terbinafine for 12 weeks has the highest cure rate and best long-term
resolution rate of the therapies listed. Other agents and pulsed dosing regimens have lower cure rates.
Topical creams are not appropriate for onychomycosis because the infection resides in the cell of the
toenail. Antifungal nail lacquers have a lower cure rate than systemic therapy and should be used only
when oral agents would not be safe. Toenail removal is reserved for patients with an isolated infected nail
or in cases involving a dermatophytoma.
104. A 24-year-old female had been healthy with no significant medical illnesses until about 3 months
ago, when she was diagnosed with schizophrenia and treatment was initiated. She is now
concerned because she has gained 10 lb since beginning treatment. A comprehensive metabolic
panel is normal, with the exception of a fasting blood glucose level of 156 mg/dL.
Which one of the following medications would be most likely to cause these findings?
A) Clonazepam (Klonopin)
B) Thioridazine
C) Chlorpromazine
D) Aripiprazole (Abilify)
E) Olanzapine (Zyprexa)
aItem 104
ANSWER: E
Second-generation, or “atypical,” antipsychotics are associated with weight gain, elevated triglycerides,
and type 2 diabetes mellitus. Olanzapine and clozapine are associated with the highest risk. Clonazepam,
a benzodiazepine, does not share these risks. Thioridazine and chlorpromazine are first-generation
antipsychotics, and carry less risk of these side effects. Aripiprazole, although it is a second-generation
antipsychotic, has been found to cause weight gain and metabolic changes similar to those seen with
placebo.
105. A 40-year-old male with a 20-pack-year history of smoking is concerned about lung cancer. He
denies any constitutional symptoms, or breathing or weight changes.
You encourage him to quit smoking and order which one of the following?
A) No testing
B) A chest radiograph
C) Low-dose CT of the chest
D) Sputum cytology
aItem 105
ANSWER: A
This patient is at risk for lung cancer, even with no symptoms. He should be encouraged to stop smoking,
especially if he has concerns that may help motivate him to quit. No study has demonstrated that screening
with any of the tests listed improves survival, and no major organization endorses lung cancer screening.
106. A 40-year-old white female presents with pain on inspiration and dyspnea since this morning.
She has no chronic medical problems, takes no medications, has not traveled, and has no history
of trauma. On examination the patient is afebrile, has a heart rate of 90 beats/min and a
respiratory rate of 20/min, and her lungs are clear to auscultation. The pain is worse in the
supine position.
Which one of the following would you do initially?
A) Order a CBC with differential
B) Order a chest film and EKG
C) Prescribe ibuprofen
D) Prescribe omeprazole (Prilosec)
E) Prescribe a bronchodilator
aItem 106
ANSWER: B
This patient has pleuritic chest pain, and the fact that it is worse when supine and is accompanied by
dyspnea creates additional concern. Supine pain could be due to pericarditis, which may be evident on an
EKG. Dyspnea increases suspicion for pneumonia, pulmonary embolism, pneumothorax, and myocardial
infarction, and a chest film and EKG are recommended to evaluate these possibilities. The lack of any
significant medical history does not rule out any of these problems.
Once these problems have been ruled out, a diagnosis of pleurisy would be reasonable and can be treated
with an NSAID. A CBC would only indicate the possibility that infection or anemia is the cause of the
problem. Omeprazole or a bronchodilator would be inappropriate treatment, as asthma and reflux are not
likely in this patient.
107. An anxious 62-year-old white male comes to the emergency department complaining of extreme
shortness of breath and a cough producing blood-tinged sputum. The patient denies chest pain
and fever. On examination he is afebrile and has expiratory wheezes and a few rales throughout
3 the chest. The heart is normal except for a rapid rate and an S gallop. A chest radiograph
reveals a right pleural effusion with enlargement of the cardiac silhouette and redistribution of
blood flow to the upper lobes.
Which one of the following tests would be best for confirming the diagnosis?
A) Troponin I
B) BNP
C) D-dimer
D) CT angiography of the chest
E) Arterial blood gases
Item 107
ANSWER: B
3 This patient has heart failure with a bronchospastic component. The S gallop occurs with a dilated left
ventricle and a right-sided pleural effusion, which are common in heart failure. A BNP level is useful in
differentiating cardiac and pulmonary diseases, while a troponin I level is helpful in assessing for cardiac
ischemia. Arterial blood gasses are not useful in confirming the diagnosis. A CT angiogram of the chest
would be useful for diagnosing pulmonary embolism. A d-dimer test is helpful to rule out venous
thromboembolic disease.
108. A study finds that the positive predictive value of a new test for breast cancer is 75%, which
means
A) if 100 patients with known breast cancer have the test, 75 (75%) will have a positive test
result
B) if 100 patients with no breast cancer have the test, 75 (75%) will have a negative test
C) 75% of patients who test positive actually have breast cancer
D) 75% of patients who test negative do not have breast cancer
Item 108
ANSWER: C
Positive predictive value refers to the percentage of patients with a positive test for a disease who actually
have the disease. The negative predictive value of a test is the proportion of patients with negative test
results who do not have the disorder.
33
The percentage of patients with a disorder who have a positive test for that disorder is a test’s sensitivity.
The percentage of patients without a disorder who have a negative test for that disorder is a test’s
specificity.
109. A 49-year-old white female comes to your office complaining of painful, cold finger tips which
turn white when she is hanging out her laundry. While there is no approved treatment for this
condition at this time, which one of the following drugs has been shown to be useful?
A) Propranolol (Inderal)
B) Nifedipine (Procardia)
C) Ergotamine/caffeine (Cafergot)
D) Methysergide (Sansert)
tem 109
ANSWER: B
At present there is no approved treatment for Raynaud’s disease. However, patients with this disorder
reportedly experience subjective symptomatic improvement with calcium channel antagonists, with
nifedipine being the calcium channel blocker of choice. -Blockers can produce arterial insufficiency of
the Raynaud type, so propranolol and atenolol are contraindicated. Drugs such as ergotamine preparations
and methysergide can produce cold sensitivity, and should therefore be avoided in patients with Raynaud’s
disease.
110. A 54-year-old male comes to your office with a 2-day history of swelling, erythema, and pain
in his right first metatarsophalangeal joint. This is the third time this year he has had this
problem. He has treated previous episodes with over-the-counter pain medicines, ice packs, and
elevation. Your evaluation suggests gout as the diagnosis.
Which one of the following treatments for gout is most likely to worsen his current symptoms?
A) Allopurinol (Zyloprim)
B) Colchicine
C) Elastic compression bandages
D) Indomethacin
E) Prednisone
Item 110
ANSWER: A
All of the treatments listed are commonly used in the management of gout with good success. Allopurinol
decreases the production of uric acid and is effective in reducing the frequency of acute gouty flare-ups.
However, it should not be started during an acute attack since fluctuating levels of uric acid can actually
worsen inflammation and intensify the patient’s pain and swelling.
Colchicine inhibits white blood cells from enveloping urate crystals and is effective during acute attacks,
as are NSAIDs such as indomethacin. Corticosteroids such as prednisone are also considered a first-line
treatment for acute attacks. Compression as an adjunctive therapy may help control pain and swelling.
antidepressant recommended for children
one recommended by the FDA for treatment of depression in children 8–17 years old
Treatment of stage 1 COPD
Short-acting bronchodilators such as albuterol and ipratropium are recommended on an as-needed basis
for treatment of breathlessness in stage I (mild) COPD.
Difference in antibodies between Graves and Hashimotos thyroiditis
but in Graves’ disease there
is a predominance of TSH receptor antibodies. In Hashimoto’s disease TSH receptor–blocking antibodies
are more predominant. These immunoglobulins tend to disappear during therapy
Child with fever and pain in the hips
It is
recommended that after plain films, the first studies to be performed should be a CBC and an erythrocyte
sedimentation rate (ESR). Studies have shown that septic arthritis should be considered highly likely in
a child who has a fever over 38.7°C (101.7°F), refuses to bear weight on the leg, has a WBC count
>12,000 cells/mm , and has an ESR >40 mm/hr.
111. A 30-year-old female who had a deep venous thrombosis in her left leg during pregnancy has
an uneventful delivery. During the pregnancy she was treated with low molecular weight
heparin. Just after delivery her left leg is pain free and is not swollen. She plans to resume
normal activities soon.
Which one of the following would be most appropriate with regard to anticoagulation?
A) Discontinuing treatment, with no further evaluation
B) Discontinuing treatment if venous Doppler ultrasonography is negative for thrombus
C) Continuing low molecular weight heparin for 6 more weeks
D) Switching to low-dose unfractionated heparin for 6 weeks
E) Switching to aspirin for 6 weeks
aItem 111
ANSWER: C
The risk of pulmonary embolism continues in the postpartum period, and may actually increase during that
time. For patients who have had a deep-vein thrombosis during pregnancy, treatment should be continued
for 6 weeks after delivery, with either warfarin or low molecular weight heparin.
112. An overweight 11-year-old male with acanthosis nigricans is found to have a fasting plasma
glucose level of 175 mg/dL on two occasions. Over the next 6 months, despite reasonable
adherence to a diet and exercise regimen, he has preprandial and bedtime finger-stick blood
1c glucose levels that average 180 mg/dL. His hemoglobin A is 9.0%.
Which one of the following oral agents would be most appropriate at this time?
A) Metformin (Glucophage)
B) Glyburide (DiaBeta)
C) Sitagliptin (Januvia)
D) Pioglitazone (Actos)
E) Acarbose (Precose)
aItem 112
ANSWER: A
Metformin and insulin are the only agents approved for treatment of type 2 diabetes mellitus in children.
113. A 32-year-old female experiences an episode of unresponsiveness associated with jerking
movements of her arms and legs. Which one of the following presentations would make a
diagnosis of true seizure more likely?
A) Post-event confusion
B) Eye closure during the event
C) A history of fibromyalgia
D) A history of chronic back pain
E) A normal serum prolactin level after the event
aaItem 113
ANSWER: A
Up to 20% of patients diagnosed with epilepsy actually have pseudoseizures. Eye closure throughout the
event is uncommon in true seizures, and a history of fibromyalgia or chronic pain syndrome is predictive
of pseudoseizures. If obtained within 20 minutes of the event, a serum prolactin level may be useful in
differentiating a true seizure from a pseudoseizure. An elevated level has a sensitivity of 60% for
generalized tonic-clonic seizures and 46% for complex partial seizures. Other features suggestive of
seizure activity include tongue biting, the presence of an aura, postictal confusion, and focal neurologic
signs.
114. A patient dying of cancer is suffering from pain in spite of his narcotic regimen. You increase
his dosage of morphine, knowing it will probably hasten his death.
Which ethical principle are you following?
A) Distributive justice
B) Double effect
C) Death with dignity
D) Futility
E) Autonomy
Item 114
ANSWER: B
The concept of “double effect” dates back to the Middle Ages. It is used to justify medical treatment
designed to relieve suffering when death is an unintended but foreseeable consequence. It is based on two
basic presuppositions: first, that the doctor’s motivation is to alleviate suffering, and second, that the
treatment is appropriate to the illness.
Distributive justice relates to the allocations of resources. Death with dignity is a recently introduced
concept and is not a factor in the scenario described here. Futility refers to using a treatment for which
there is no rational justification. Autonomy refers to the patient’s ability to direct his or her own care,
which is not an issue in this case.
115. You are a member of a committee at your local hospital that has been asked to develop measures
to reduce the incidence of postoperative methicillin-resistant Staphylococcus aureus (MRSA)
infections. Which one of the following would be most effective for preventing these infections?
A) Give preoperative antibiotics to all surgical patients to eradicate bacteria
B) Screen all admitted patients for MRSA and use antibiotics pre- and postoperatively in
positive cases
C) Culture the nares of all hospital employees upon hiring and on a routine basis thereafter
D) Institute an intensive program of good hand washing for all employees
atem 115
ANSWER: D
Nosocomial infections are a significant factor in morbidity and cost in the health care field. Methicillinresistant
Staphylococcus aureus (MRSA) has rapidly increased in frequency, first being found only at
tertiary centers, then local hospitals, and now in the outpatient setting.
35
In 2004, an estimated 1.5% of U.S. residents carried MRSA in the anterior nares of the nose. Of those
who are found to be colonized, either at the time of hospitalization or later by a routine culture, 25% will
develop a MRSA infection. However, a recent study showed that of 93 patients who became infected with
the organism, 57% were not colonized at the time of infection. The study also attempted to screen all
patients for MRSA on admission, but found that even though 337 previously unknown carriers were found
(in addition to those already known to harbor the organism), there was not a significant decrease in the rate
of MRSA infections during the study.
Although MRSA infections can be serious, they comprise only 8% of nosocomial infections in the hospital,
and concentrating prevention efforts only on MRSA has little effect on that 8%, and no effect on the 92%
of infections caused by other organisms.
Iatrogenic complications arise from trying to treat MRSA carriers, including both drug reactions and the
development of other resistant organisms. Costs related to attempts at prophylaxis also go up.
Culturing all hospital employees has not been proven to be of value, as employees can pick up the
organism after screening, and also can spontaneously eradicate the organism without treatment. The best
way to prevent complications and postoperative infections is to aggressively advocate universal and
frequent hand washing and room cleaning, and use good isolation techniques and methods of preventing
infection, such as strict catheter and intravenous tubing protocols.
116. A 25-year-old male who came to your office for a pre-employment physical examination is found
to have 2+ protein on a dipstick urine test. You repeat the examination three times within the
next month and results are still positive. Results of a 24-hour urine collection show protein
excretion of <2 g/day and normal creatinine clearance. As part of his further evaluation you
obtain split urine collections with a 16-hour daytime specimen containing an increased
concentration of protein, and an 8-hour overnight specimen that is normal.
Additional appropriate evaluation for this man’s problem at this time includes which one of the
following?
A) Serum and urine protein electrophoresis
B) Antinuclear antibody
C) Serum albumin and lipid levels
D) Renal ultrasonography
E) No specific additional testing
aItem 116
ANSWER: E
Persons younger than 30 years of age who excrete less than 2 g of protein per day and who have a normal
creatinine clearance should be tested for orthostatic proteinuria. This benign condition occurs in about
3%–5% of adolescents and young adults. It is characterized by increased protein excretion in the upright
position, but normal protein excretion when the patient is supine. It is diagnosed using split urine
collections as described in the question. The daytime specimen has an increased concentration of protein,
while the nighttime specimen contains a normal concentration. Since this is a benign condition with normal
renal function, no further evaluation is necessary.
117. A 50-year-old male is brought to the emergency department because of a syncopal episode.
Prior to the episode, he felt bad for 30 minutes, then developed nausea followed by vomiting.
During a second bout of vomiting he blacked out and fell to the floor. His wife did not observe
any seizure activity, and he was unconscious only for a few seconds. His history is otherwise
negative, his past medical history is unremarkable, and he currently takes no medications. A
physical examination is normal.
Which one of the following would be the most helpful next step?
A) CT of the head
B) Carotid ultrasonography
C) A CBC and complete metabolic profile
D) Echocardiography
E) An EKG
aItem 117
ANSWER: E
The workup of patients with syncope begins with a history and a physical examination to identify those at
risk for a poor outcome. Patients who have a prodrome of 5 seconds or less may have a cardiac
arrhythmia. Patients with longer prodromes, nausea, or vomiting are likely to have vasovagal syncope,
which is a benign process. Patients who pass out after standing for 2 minutes are likely to have orthostatic
hypotension.
36
In most cases, the recommended test is an EKG. If the EKG is normal, dysrhythmias are not a likely
cause of the syncopal episode. Laboratory testing and advanced studies such as CT or echocardiography
are not necessary unless there are specific findings in either the history or the physical examination.
118. A 38-year-old female with seasonal allergies presents with a 10-day history of sinus pain and
purulent nasal drainage, along with temperature elevations up to 102°F (39°C). She has been
taking nonprescription loratidine (Claritin), but says it provides little relief. She asks you to
prescribe an antibiotic.
Which one of the following would be most appropriate at this point?
A) Continuation of symptomatic treatment only
B) In-office nasal irrigation
C) Amoxicillin
D) Azithromycin (Zithromax)
E) Imaging of the sinuses
aItem 118
ANSWER: C
The American Academy of Otolaryngology published guidelines for the diagnosis and management of
rhinosinusitis in adults in 2007. They cite reasonable evidence for initiating antibiotic treatment in patients
with symptoms persisting for 7–10 days that are not improving or worsening (SOR B). Amoxicillin should
be the first-line agent, with azithromycin or trimethroprim/sulfamethoxazole recommended for penicillinallergic
patients. Broader-spectrum antibiotics such as fluoroquinolones should be reserved for treatment
failures. Imaging is indicated only if other etiologies are being considered or if the problem is recurrent.
119. A 27-year-old male was roughhousing with his children when he was accidentally struck in the
left eye. He immediately felt pain in the eye, and over the next hour noted increased tearing,
pain with blinking, increasing headache, and a foreign-body sensation. You see the patient the
next day, and examination with flourescein dye and a cobalt-blue filtered light reveals a corneal
abrasion.
Appropriate management includes which one of the following?
A) An eye patch for 24–48 hours
B) Mydriasis with a short-acting agent, such as tropicamide ophthalmic (Mydriacyl)
C) A topical anesthetic instilled every 4 hours if no foreign body or infection is found
D) Topical corticosteroid drops
E) Referral to an ophthalmologist if the edge of the abrasion is white or gray
aItem 119
ANSWER: E
A white or gray appearance at the edge of a corneal abrasion may indicate infection, and referral to an
ophthalmologist is indicated. Mydriatic agents and eye patching are ineffective for corneal abrasions and
are not recommended (SOR A).
Progression to recurrent corneal erosion may occur years after a corneal abrasion. Symptoms mimic the
initial corneal abrasion, and tearing on awakening is common.
Topical anesthetics should be administered only in the office; if a patient uses the medication at home, it
can delay healing and mask complications.
120. When a screening test identifies a cancer earlier, thereby increasing the time between diagnosis
and death without prolonging life, this is called
A) length-time bias
B) lead-time bias
C) a false-positive screening test
D) increasing the positive predictive value of the screening test
E) attributable risk
aItem 120
ANSWER: B
Lead-time bias is when a screening test identifies a cancer earlier, thereby increasing the time between
diagnosis and death without actually prolonging life. Length-time bias is when a screening test finds a
disproportionate number of cases of slowly progressive disease and misses the aggressive cases, thereby
leading to an overestimate of the effectiveness of the screening. A false-positive test is one that suggests
cancer when no cancer exists. The positive predictive value is the proportion of positive test results that
are true positives. Attributable risk is the amount of difference in risk for a disease that can be accounted
for by a specific risk factor.
121. A 42-year-old white female presents to your office as a new patient. She states that she has an
8-year history of abdominal cramps and diarrhea. Her symptoms have not responded to the
usual treatments for irritable bowel syndrome. She has no rectal bleeding, anemia, weight loss,
or fever, and no family history of colon cancer. Her medical history and a review of symptoms
is otherwise negative, and a physical examination is normal.
Which one of the following would be the most appropriate next step in evaluating this patient?
A) A CBC
B) A TSH level
C) A complete metabolic panel
D) Serologic testing for celiac sprue
E) Stool testing for ova and parasites
aItem 121
ANSWER: D
In patients who have symptoms of irritable bowel syndrome (IBS), the differential diagnosis includes celiac
sprue, microscopic and collagenous colitis, atypical Crohn’s disease for patients with diarrhea-predominant
IBS, and chronic constipation (without pain) for those with constipation-predominant IBS. If there are no
warning signs, laboratory testing is warranted only if indicated by the history.
122. During a comprehensive health evaluation a 65-year-old African-American male reports mild,
very tolerable symptoms of benign prostatic hyperplasia, rated as a score of 7 on the American
Urological Association Symptom Index. He has never smoked, and his medical history is
otherwise unremarkable. Objective findings include an enlarged prostate that is firm and
nontender, with no nodules. A urinalysis is normal and his prostate-specific antigen level is 1.8
ng/mL.
Based on current evidence, which one of the following treatment options is most appropriate at
this time?
A) Observation, with repeat evaluation in 1 year
B) Saw palmetto
C) An "-receptor antagonist
D) A 5-"-reductase inhibitor
E) Urologic referral for transurethral resection of the prostate
aItem 122
ANSWER: A
Watchful waiting with annual follow-up is appropriate for men with mild benign prostatic hyperplasia
(BPH). Prostate-specific antigen (PSA) levels correlate with prostate volume, which may affect the
treatment of choice, if indicated (SOR C). PSA levels >2.0 ng/mL for men in their 60s correlate with
a prostatic volume >40 mL. This patient’s PSA falls below this level. In men with a prostatic volume
>40 mL, 5-reductase inhibitors should be considered for treatment (SOR A). -Blockers provide
symptomatic relief in men whose disease has progressed to the point that they have moderate to severe
BPH symptoms (SOR A).
A recent high-quality, randomized, controlled trial found no benefit from saw palmetto with regard to
symptom relief or urinary flow after 1 year of therapy. The American Urological Association does not
recommend the use of phytotherapy for BPH. Surgical consultation is appropriate when medical therapy
fails or the patient develops refractory urinary retention, persistent hematuria, or bladder stones.
123. A 47-year-old female presents with progressive difficulty hearing. She is employed as an office
worker, has no significant past medical history, and takes no medications. Physical examination
shows no gross abnormalities of her outer ears. The external ear canals are free of cerumen,
and the tympanic membranes move well to insufflation. Weber’s test and the Rinne test have
results that are compatible with a conductive hearing loss.
Which one of the following is the most likely cause of this patient’s hearing loss?
A) Noise-induced hearing loss
B) Meniere’s disease
C) Otosclerosis
D) Acoustic neuroma
E) Perilymphatic fistula
aItem 123
ANSWER: C
Otosclerosis typically presents between the third and fifth decades, and is more common in women. The
chief feature of otosclerosis is a progressive conductive hearing loss. Occasionally, when lesions impinge
on the stapes footplate, a sensorineural loss may occur. All of the other choices are exclusively
sensorineural in character. Meniere’s disease also causes fluctuating hearing loss. Noise-induced hearing
loss frequently and characteristically is accompanied by tinnitus. Perilymphatic fistula is associated with
sudden unilateral hearing loss with tinnitus and vertigo. Acoustic neuroma is associated with tinnitus and
gradual hearing impairment.
124. Risk factors for venous thromboembolism include which one of the following?
A) Anemia
B) The use of oral hypoglycemic agents
C) Being underweight
D) Young age
E) Spinal cord injury
aItem 124
ANSWER: E
There are many risk factors for thromboembolism, including polycythemia vera, oral contraceptive use,
obesity, advanced age, and spinal cord injury. Spinal cord injury induces immobility, as do obesity and
advanced age. Oral contraceptives make blood more coagulable, particularly in patients with clotting
factor abnormalities such as factor V Leiden. Polycythemia vera increases sludging of blood cells and
increases the risk of forming clots. Clot risk is not increased by oral hypoglycemic agents, low BMI,
youth, or anemia.
125. Women should be tested for human papillomavirus (HPV) DNA
A) every year, when a Papanicolaou (Pap) test is performed
B) every 2 years, when a Pap test is performed
C) prior to receiving HPV vaccine (Gardasil)
D) when a Pap smear shows ASC-US
E) when genital warts are noted
atem 125
ANSWER: D
Human papillomavirus (HPV) DNA testing is useful for determining whether colposcopy is needed in
patients whose Papanicolaou (Pap) test shows aytpical squamous cells of undetermined significance (ASCUS).
Women with a negative result for high-risk HPV can be safely followed without colposcopy. Some
recent studies suggest that screening HPV DNA testing for women in their thirties may be useful, most
likely as a replacement for Pap smears; however, this is in the preliminary stages of study. Younger
women would not benefit from screening for HPV DNA because it would identify the very large number
of young women who have a transient infection. The addition of HPV DNA testing to Pap smears at
currently recommended intervals would not change management.
The presence or absence of HPV DNA does not alter the indications for HPV vaccine, because patients
with or without HPV are candidates for vaccination. The vaccine protects against HPV strains that the
patient may not have contracted. Genital warts are caused by HPV, usually a low-risk strain, so testing
for HPV in patients with this problem does not provide useful information.
126. A 41-year-old male trips on a curb while running, sustaining an inversion ankle injury.
According to the Ottawa ankle rules, which one of the following would be an indication for
radiographic evaluation?
A) Tenderness at the anterior talofibular ligament
B) Point tenderness over the cuboid
C) Inability to take four steps either immediately after the injury or while in your office
D) Bony tenderness at the anterior aspect of the distal tibia
E) Point tenderness over the base of the fourth metatarsal
aItem 126
ANSWER: C
The Ottawa ankle rules have been designed and validated to reduce unnecessary radiographs. Radiographs
should be obtained for all patients with an acute ankle injury who meet any of the following criteria:
inability to take four steps, either immediately after the injury or when being evaluated; localized
tenderness of the navicular bone or the base of the fifth metatarsal; or localized tenderness at the posterior
edge or tip of either malleolus.
127. An anxious 30-year-old white female comes to the emergency department with shortness of
breath, circumoral paresthesia, and carpopedal spasms. Which one of the following sets of
blood gas values is most consistent with this clinical picture?
2 2 A) pH 7.25 (N 7.35–7.45), pCO 25 mm Hg (N 35–45), pO 100 mm Hg (N 80–100)
2 2 B) pH 7.25, pCO 50 mm Hg, pO 80 mm Hg
2 2 C) pH 7.50, pCO 25 mm Hg, pO 100 mm Hg
2 2 D) pH 7.55, pCO 50 mm Hg, pO 80 mm Hg
aItem 127
ANSWER: C
Anxiety, shortness of breath, paresthesia, and carpopedal spasm are characteristic of psychogenic
hyperventilation. Respiratory alkalosis secondary to hyperventilation is diagnosed when arterial pH is
2 elevated and pCO is depressed. Low pH is characteristic of acidosis, either respiratory or metabolic, and
2 elevated pH with elevated pCO is characteristic of metabolic alkalosis with respiratory compensation.
128. A 72-year-old female is admitted to the hospital after having surgery for a hip fracture. Her
previous medical history is significant for hypertension and type 2 diabetes mellitus. Two days
after admission the orthopedic surgeon consults with you because the patient has had several
hours of fever to 39°C (102°F); tachycardia, with a pulse rate of 120 beats/min; and systolic
blood pressures of 91–97 mm Hg (baseline 120–140 mm Hg with medication).
When you examine the patient she says she feels weak and chilled but she is alert. Her oxygen
saturation is excellent on room air, and a physical examination is normal except for the sinus
tachycardia and low blood pressure. A urinary catheter is in place, but there has been little
output over the last 4 hours. Her renal function was normal prior to her hospitalization.
A chest radiograph is normal. Her electrolyte levels are normal, but laboratory tests reveal the
following abnormal results:
WBCs. . . . . . . . . . . . . . . . . . . . . . . . . 2500/mm3 (N 5000–10,000)
BUN. . . . . . . . . . . . . . . . . . . . . . . . . . 50 mg/dL (N 10–15)
Creatinine. . . . . . . . . . . . . . . . . . . . . . . 2.3 mg/dL (N 0.6–1.0)
Bicarbonate. . . . . . . . . . . . . . . . . . . . . . 18 mmol/L (N 22–30)
Urinalysis
Specific gravity. . . . . . . . . . . . . . . . . . >1.030 (N 1.003–1.040)
WBCs. . . . . . . . . . . . . . . . . . . . . . . . >100/hpf
RBCs.. . . . . . . . . . . . . . . . . . . . . . . . 10–20/hpf
Epithelial cells. . . . . . . . . . . . . . . . . . . 3–5/hpf
Casts. . . . . . . . . . . . . . . . . . . . . . . . . few hyaline
In addition to antibiotics, which one of the following would be the most appropriate management
of this patient’s problem?
A) High-rate intravenous normal saline
B) Intravenous furosemide, 40 mg every 6 hours
C) Intravenous dopamine, 2–4 :g/kg/min
D) Intravenous sodium bicarbonate
E) Urgent nephrology consultation for dialysis
aItem 128
ANSWER: A
This patient appears to be experiencing sepsis syndrome due to urinary infection. The renal failure that
has resulted is almost certainly due to low perfusion of the kidneys (prerenal azotemia). This condition
requires aggressive intravenous fluids to halt and reverse the reduction in nephrologic function. At times,
this underperfusion can result in acute tubular necrosis (an intrinsic renal dysfunction) that may prevent
excretion of any excess fluid, so the patient’s fluid status should be monitored carefully. Metabolic
acidosis will likely reverse with appropriate hydration, and sodium bicarbonate should be reserved for
severe acidosis (<10–15 mmol/L) or for those with chronic kidney disease. Low-dose dopamine has been
proven to be ineffective in acute renal failure, and this patient does not have an indication for dialysis.
Intravenous furosemide is contraindicated.
129. A 30-year-old female comes to your office because she is concerned about irregular menses
(fewer than 9/year), acne, and hirsutism. Her BMI is 36.0 kg/m2. She has no other medical
problems and would like to have a baby. Her fasting blood glucose level is 140 mg/dL.
Which one of the following would be the most appropriate treatment for this patient’s condition
and concerns?
A) Lifestyle modification only
B) Lifestyle modification and pioglitazone (Actos)
C) Lifestyle modification and metformin (Glucophage)
D) Lifestyle modification and an oral contraceptive
E) Lifestyle modification and oral testosterone
aitem 129
ANSWER: C
This patient has classic features of polycystic ovary syndrome (PCOS). The diagnosis is based on the
presence of two of the following: oligomenorrhea or amenorrhea, clinical or biochemical
hyperandrogenism, or polycystic ovaries visible on ultrasonography.
Lifestyle modifications are necessary, but medications are also needed. First-line agents for the treatment
of hirsutism in patients with PCOS include spironolactone, metformin, and eflornithine (SOR A). Firstline
agents for ovulation induction and treatment of infertility in patients with PCOS include metformin and
clomiphene, alone or in combination with rosiglitazone (SOR A).
Metformin can also improve menstrual irregularities in patients with PCOS (SOR A), and is probably the
first-line agent for obese patients to promote weight reduction (SOR B). In addition, metformin improves
insulin resistance (diagnosed by elevated fasting blood glucose) in patients with PCOS, as do rosiglitazone
and pioglitazone.
40
Pioglitazone would not be appropriate for this patient because it causes weight gain. Oral contraceptives
would improve the patient’s menstrual irregularities and hirsutism, but she wishes to become pregnant.
Testosterone would worsen the hyperandrogenism and would not treat the PCOS.
130. Which one of the following drugs would be the most appropriate empiric therapy for nursinghome–
acquired pneumonia in a patient with no other underlying disease?
A) Cefazolin
B) Erythromycin
C) Ampicillin
D) Tobramycin (Nebcin)
E) Levofloxacin (Levaquin)
aItem 130
ANSWER: E
The major concern with regard to pneumonia in the nursing-home setting is the increased frequency of
oropharyngeal colonization by gram-negative organisms. In the absence of collectible or diagnostic sputum
Gram’s stains or cultures, empiric therapy must cover Streptococcus pneumoniae, Staphylococcus aureus,
Haemophilus influenzae, and gram-negative bacteria. Levofloxacin is the best single agent for providing
coverage against this spectrum of organisms.
tinnitus and
gradual hearing impairment.
Acoustic neuroma
sudden unilateral hearing loss with tinnitus and vertigo.
Perilymphatic fistula
hearing
loss frequently and characteristically is accompanied by tinnitus.
Noise-induced
typically presents between the third and fifth decades, and is more common in women. The
chief feature of otosclerosis is a progressive conductive hearing loss. Occasionally, when lesions impinge
on the stapes footplate, a sensorineural loss may occur
Otosclerosis
5 alpha-reductase inhibitors should be considered for treatment of BPH when PSa >
>2
131. A severely depressed 77-year-old male is hospitalized after an intentional drug overdose. He
was found by chance when his housekeeper returned to retrieve something she had left behind.
The patient has been severely depressed since he suffered a myocardial infarction 1 year ago,
and the recent death of his wife has increased his despondency.
He had left a note apologizing to his family and his physician, who has treated him with multiple
medications for depression over the past year. He has been treated with SSRIs, SNRIs, and
atypical antipsychotics in high doses and in various combinations without significant
improvement.
Which one of the following would be most likely to improve this patient’s depression at this
point?
A) Cognitive-behavioral therapy
B) Psychoanalysis
C) Electroconvulsive therapy
D) Goal-directed psychotherapy
E) Limbic stimulation
Item 131
ANSWER: C
Electroconvulsive therapy has been shown to be more effective than psychiatric therapy, pharmacologic
therapy, and other interventions in depressed older patients. It would be particularly appropriate in this
case given the patient’s age, his failure to respond to medications, and the need for rapid improvement to
decrease the risk of further suicide attempts.
132. A 55-year-old obese male with hypertension and daytime somnolence is found to have severe
obstructive sleep apnea, with an apnea-hypopnea index of 32 on an overnight polysomnogram.
Which one of the following is considered to be first-line therapy for this patient's condition?
A) Continuous positive airway pressure (CPAP)
B) An oral dental appliance
C) Uvulopalatopharyngoplasty
D) Sleep positioning therapy
E) Tracheostomy
Item 132
ANSWER: A
Patients with severe sleep apnea (apnea-hypopnea index >29) and concomitant cardiovascular disease
benefit the most from treatment for obstructive sleep apnea. Because it is relatively easy to implement and
has proven efficacy, continuous positive airway pressure (CPAP) is considered first-line therapy for severe
apnea.
133. A 72-year-old male is brought by ambulance to the emergency department with weakness and
numbness of his left side that began earlier this morning. While in the emergency department
he becomes comatose with infrequent, gasping breaths and is quickly intubated and placed on
a ventilator. A full evaluation shows an acute ischemic right-sided stroke. His wife states that
she wishes to have the ventilator stopped, as she believes this would be consistent with her
husband’s wishes in this circumstance. She understands that this would precipitate the patient’s
death. The wife presents a legally valid advance directive confirming her as the patient’s healthcare
proxy.
Which one of the following responses to the wife’s request is most ethically appropriate?
A) Withdraw the ventilator as requested
B) Contact the hospital ethics committee to initiate the legal requirements to process the
wife’s request
C) Inform the wife that all life-sustaining care should be given until the patient’s condition
has been determined to be irreversible
D) Inform the wife that intubation may have been avoided in the emergency department,
but once life-sustaining care has been initiated it should not be withdrawn
E) Promptly contact hospital security or the local law enforcement agency to report the
wife’s request
Item 133
ANSWER: A
Competent adult patients have the right to refuse any medical intervention, even if forgoing this treatment
may result in their death. Legally and ethically it does not matter whether the patient requests that care
be withheld before it is started or that it be withdrawn once it is begun. All states currently allow
competent patients to legally designate a health-care proxy to make these decisions for them if they become
unable to communicate or are no longer competent to decide for themselves. The patient in this example
has instituted such a legal advance directive and his proxy’s request should be respected as his own and
the care withdrawn. If there were no advance directive the decision in this case would become more
difficult, and might require a family conference or the involvement of an ethics committee. A patient’s
condition does not need to be terminal or irreversible to allow the removal of life-sustaining therapy. Legal
involvement is rarely required in situations where advance directives are already available and valid.
134. Which one of the following is most likely to be of benefit in patients with essential tremor of the
hand?
A) Isoniazid
B) Diazepam (Valium)
C) Topiramate (Topamax)
D) Clonidine (Catapres)
E) Gabapentin (Neurontin)
Item 134
ANSWER: C
Treatments likely to be beneficial for essential tremor of the hands include propranolol and topiramate.
Topiramate has been shown to improve tremor scores after 2 weeks of treatment, but is associated with
appetite suppression, weight loss, and paresthesias. Medications with unknown effectiveness include
benzodiazepines, -blockers other than propranolol, calcium channel blockers, clonidine, gabapentin, and
isoniazid.
135. A 24-year-old African-American male presents with a history of several weeks of dyspnea,
cough productive of bloody streaks, and malaise. His examination is normal except for bilateral
facial nerve palsy. A CBC and urinalysis are normal. A chest radiograph reveals bilateral
lymph node enlargement.
This presentation is most consistent with
A) polyarteritis nodosa
B) Goodpasture’s syndrome
C) sarcoidosis
D) pulmonary embolus
Item 135
ANSWER: C
Sarcoidosis, a disease of unknown etiology, affects young to middle-age adults (predominantly 20–29 years
old). In the U.S. it is more common in African-Americans. It is asymptomatic in 30%–50% of patients,
and is often diagnosed on a routine chest film. About one-third of cases will present with fever, malaise,
weight loss, cough, and dyspnea. The pulmonary system is the main organ system affected, and findings
may include bilateral hilar lymphadenopathy and discrete, noncaseating epithelial granulomas. Facial
nerve palsy is seen in <5% of patients, and usually occurs late in the process. Before Lyme disease was
recognized, bilateral facial nerve palsy was almost always due to sarcoidosis. Hemoptysis does not
generally occur until late in the course of sarcoidosis, and is usually related to Aspergillus infection or
cavitation. Renal involvement rarely results in significant proteinuria or hematuria.
Polyarteritis nodosa may involve the lungs. Although pneumonic episodes may be associated with
hemoptysis in a small percentage of patients, the chest radiograph is more likely to reveal granulomatous
lesions rather than patchy infiltrates. Goodpasture’s syndrome is characterized by pulmonary hemorrhage,
glomerulonephritis, and antiglomerular basement membrane antibodies. Hemoptysis, pulmonary alveolar
infiltrates, dyspnea, and iron-deficiency anemia are frequent presenting features. Within days or weeks,
the pulmonary findings are generally followed by hematuria, proteinuria, and the rapid loss of renal
42
function. Pulmonary embolus is an acute event, and would present with dyspnea and possibly hemoptysis,
but not hilar lymphadenopathy.
136. An elderly male patient takes aspirin, 81 mg daily, for prevention of a heart attack. He also
takes herbal supplements.
Which one of the following supplements can have a negative interaction with aspirin?
A) Kava
B) Yohimbine
C) Saw palmetto
D) Echinacea
E) Ginkgo biloba
Item 136
ANSWER: E
Herbal and dietary supplements can affect the absorption, metabolism, and disposition of other drugs.
Ginkgo biloba has been associated with serious intracerebral bleeding. In most of these patients,
concurrent anticoagulant drugs were being used. Ginkgo has been shown in vitro to inhibit platelet
aggregation and has been associated with case reports of spontaneous bleeding. Caution is recommended
when using this supplement with aspirin or other anticoagulants.
Kava is associated with gastrointestinal side effects and skin rashes. Yohimbine is associated with
hypertension. Saw palmetto and echinacea are not associated with bleeding.
137. An 8-year-old male presents to your office 2 days after returning from a trip to Mexico with his
family. He developed watery, nonbloody diarrhea on the day of departure. He has mild
abdominal cramping, but no fever or vomiting. His mother had similar symptoms, which were
milder and resolved with over-the-counter treatments.
Which one of the following would be most appropriate to treat this patient’s condition?
A) Metronidazole (Flagyl)
B) Ciprofloxacin (Cipro)
C) Azithromycin (Zithromax)
D) Mebendazole
E) Metoclopramide (Reglan)
Item 137
ANSWER: C
Traveler’s diarrhea commonly occurs in travelers to Mexico and developing countries. It is usually caused
by bacterial organisms such as Escherichia coli, Campylobacter, Shigella, and Salmonella. Viral and
parasitic organisms are less common causes, unless the diarrhea persists for 2 weeks. Appropriate
medications include antidiarrheal agents such as loperamide, bismuth subsalicylate, and antibiotics.
Fluoroquinolones are effective in adults, but should not be used in an 8-year-old. Azithromycin is
generally effective and safe in children. Metronidazole, mebendazole, and metoclopramide would not be
likely to successfully treat bacterial traveler’s diarrhera.
138. Secondary causes of osteoporosis in males include which one of the following?
A) Weekly consumption of 3–6 alcoholic drinks
B) Male hormone supplementation
C) Vitamin D excess
D) Obesity
E) Corticosteroid use
aItem 138
ANSWER: E
Corticosteroids are among the common secondary causes of osteoporosis in men. Other causes include
excessive alcohol use, hypogonadism, vitamin D deficiency, and decreased body mass index.
139. The preferred site for an emergency airway is
A) the thyrohyoid membrane
B) the cricothyroid membrane
C) immediately below the cricoid cartilage
D) through the first and second tracheal rings
E) at the level of the thyroid isthmus
Item 139
ANSWER: B
Fortunately, emergency tracheotomy is not often necessary, but should one be necessary the best site for
the incision is directly above the cricoid cartilage, through the cricothyroid membrane. Strictly speaking,
this is not a tracheotomy, because it is actually above the trachea. However, it is below the vocal cords
and bypasses any laryngeal obstruction. The thyrohyoid membrane lies well above the vocal cords,
making this an impractical site. The area directly below the cricoid cartilage—which includes the second,
third, and fourth tracheal rings, as well as the thyroid isthmus—is the preferred tracheotomy site under
controlled circumstances, but excessive bleeding and difficulty finding the trachea may significantly impede
the procedure in an emergency.
140. A 60-year-old male complains of multiple episodes of lightheadedness over the past 3 months,
saying he felt as if he might “pass out” while sitting at his desk. His past medical history and
a physical examination are unremarkable. An EKG shows right bundle branch block and left
anterior hemiblock.
Which one of the following would be the most appropriate next step?
A) Echocardiography
B) Cardiac event monitoring
C) Hospital admission for pacemaker insertion
D) Immediate initiation of aspirin and metoprolol (Lopressor)
Item 140
ANSWER: B
This patient’s EKG demonstrates a right bundle branch block, as well as a left anterior hemiblock. This
“trifascicular block” puts the patient at risk for tachyarrhythmias and bradyarrhythmias. Given the
patient’s complaint of near-syncope, a heart monitoring study would be most appropriate. An
echocardiogram may be helpful eventually to assess cardiac function. Although the patient is at risk for
heart block, immediate hospitalization is not indicated.
141. A 45-year-old female had myalgias, a sore throat, and a fever 2 weeks ago. She now has
anterior neck tenderness and swelling, with pain radiating up to her ears. An examination
reveals a tender goiter.
Which one of the following would support a diagnosis of subacute granulomatous thyroiditis?
A) Pretibial myxedema
B) Exophthalmos
C) A thyroid bruit
D) Low radioactive iodine uptake (<5%)
Item 141
ANSWER: D
4 Subacute granulomatous thyroiditis is the most common cause of thyroid pain. Free T is elevated early
4 in the disease, as it is in Graves’ disease; however, later in the disease T becomes depressed and then
returns to normal as the disease resolves. Pretibial myxedema, exophthalmos, and a thyroid thrill or bruit
can all be found in Graves’ disease, but not in subacute granulomatous thyroiditis. Patients with subacute
granulomatous thyroiditis will have a low radioactive iodine uptake (RAIU) at 24 hours, but patients with
Graves’ disease will have an elevated RAIU (SOR C).
142. You are examining a patient with a chronically painful shoulder. You forward flex the arm to
90° with the elbow bent to 90°. You then internally rotate the arm, which causes pain in the
shoulder.
This finding suggests
A) glenohumeral instability
B) anterior shoulder dislocation
C) impingement/rotator cuff disorder
D) acromioclavicular joint osteoarthritis
E) acromioclavicular joint separation
Item 142
ANSWER: C
The maneuver described is Hawkins’ impingement test. Pain with this maneuver may signify subacromial
impingement, including a rotator cuff tendinopathy or tear.
143. A 26-year-old female presents with symptoms of anhedonia and anxiousness. Your evaluation
leads to a diagnosis of major depressive disorder. The patient consents to medical treatment and
counseling, but she is engaged to be married in 2 months and is concerned that antidepressants
may lower her libido even further.
Which one of the following would be best for reducing the likelihood of sexual dysfunction?
A) Bupropion (Wellbutrin)
B) Paroxetine (Paxil)
C) Fluoxetine (Prozac)
D) Sertraline (Zoloft)
Item 143
ANSWER: A
Paroxetine has been found to cause higher rates of sexual dysfunction than bupropion, fluoxetine, and
sertraline. Bupropion has been found to have significantly lower rates of adverse effects on sexual function
than fluoxetine or sertraline.
144. In a patient with hypertriglyceridemia, the National Cholesterol Education Program recommends
that a reasonable goal for non-HDL cholesterol is no more than
A) 30 mg/dL above the LDL-cholesterol level
B) 40 mg/dL above the LDL-cholesterol level
C) 50 mg/dL above the LDL-cholesterol level
D) 60 mg/dL above the LDL-cholesterol level
E) 90 mg/dL above the LDL-cholesterol level
Item 144
ANSWER: A
The National Cholesterol Education Program’s Adult Treatment Panel III recommends a goal non–HDLcholesterol
level of no more than 30 mg/dL greater than the LDL-cholesterol level. This is based on a
“normal” very low density lipoprotein cholesterol level being defined as that present when triglycerides
are <150 mg/dL. This value typically is  30 mg/dL. Conversely, when triglyceride levels are >150
mg/dL, very low density lipoprotein is usually >30 mg/dL.
145. A 4-week-old white male is brought to your office with a 2-week history of increasing dyspnea,
cough, and poor feeding. The child appears nontoxic and is afebrile. On examination you note
conjunctivitis, and a chest examination reveals tachypnea and rales. A chest film shows
hyperinflation and diffuse interstitial infiltrates. A WBC count reveals eosinophilia.
What is the most likely etiologic agent?
A) Staphylococcus species
B) Chlamydia trachomatis
C) Respiratory syncytial virus
D) Parainfluenza virus
145
NSWER: B
Chlamydial pneumonia is usually seen in infants 3–16 weeks of age, and they frequently have been sick
for several weeks. The infant appears nontoxic and is afebrile, but is tachypneic with a prominent cough.
Physical examination reveals diffuse rales with few wheezes. Conjunctivitis is present in about 50% of
cases. The chest film shows hyperinflation and diffuse interstitial or patchy infiltrates.
Staphylococcal pneumonia has a sudden onset. The infant appears very ill and has a fever. At the time
of onset there may be an expiratory wheeze simulating bronchiolitis. Signs of abdominal distress,
tachypnea, dyspnea, and localized or diffuse bronchopneumonia or lobar disease may be present. The
WBC count shows a prominent leukocytosis.
Respiratory syncytial infections start with rhinorrhea and pharyngitis, followed in 1–3 days by cough and
wheezing. Auscultation reveals diffuse rhonchi, fine rales, and wheezes. The chest film is often normal.
If the illness progresses, cough and wheezing increase, air hunger and intercostal retractions develop, and
evidence of hyperexpansion of the chest is seen. In some infants, the course of the illness may be similar
to that of pneumonia. Rash or conjunctivitis may occur occasionally, and fever is an inconsistent sign.
The WBC count is normal or elevated, and the differential may be normal or shifted either to the right or
left. Chlamydial infections may be differentiated from respiratory syncytial infections by a history of
conjunctivitis and a subacute onset. Coughing is prominent, but wheezing is not. There may also be
eosinophilia. Fever is usually absent.
45
Parainfluenza virus infection presents with typical cold symptoms. Eight percent of infections affect the
upper respiratory tract. In children hospitalized for severe respiratory illness, parainfluenza viruses
account for about 50% of the cases of laryngotracheitis and about 15% each of the cases of bronchitis,
bronchiolitis, and pneumonia.
146. A 52-year-old female presents to the emergency department with a complaint of chest pain. The
symptoms began 2 hours ago while she was shopping. She describes the pain as a tightness on
the left side of her chest that radiates to her left shoulder. She has some shortness of breath with
the pain, but no nausea or diaphoresis. Her past medical history is significant for panic disorder.
Her vital signs and a physical examination are within normal limits.
Which one of the following would be the most appropriate next step in the management of this
patient?
A) Admit to a monitored bed for further evaluation
B) Obtain a CBC, a blood chemistry profile, liver function tests, and an EKG
C) Administer a short-acting benzodiazepine and observe for 60 minutes
D) Consult with a cardiologist for immediate heart catheterization
E) Obtain a troponin I measurement and an EKG
Item 146
ANSWER: E
This patient has symptoms that suggest acute coronary syndrome, which includes chest pain with activity
that radiates to the shoulder. An EKG is essential early in the evaluation of a patient with chest pain, and
the initial evaluation should also include a troponin I measurement. The patient should neither be admitted
nor given a benzodiazepine until the EKG is performed. The diagnosis of acute coronary syndrome should
be established prior to heart catheterization. Other laboratory tests may be appropriate, but they are not
the most important initial tests.
147. The most common identifiable cause of skin and soft-tissue infections presenting to metropolitan
emergency departments is
A) Staphylococcus epidermidis
B) Streptococcus pneumoniae
C) Pseudomonas aeruginosa
D) methicillin-resistant Staphylococcus aureus (MRSA)
E) Bacillus cereus
Item 147
ANSWER: D
Recent clinical experience has shown that methicillin-resistant Staphylococcus aureus (MRSA) is the most
common identifiable cause of skin and soft-tissue infections among patients presenting to emergency
departments in 11 U.S. cities. The other responses should be considered in evaluating these infections,
but they are not as common as MRSA infections.
148. A 16-year-old male accompanied by his mother presents to your outpatient clinic with concerns
about his short stature and “boyish” looks. He is a sophomore in high school but is frequently
mistaken for someone much younger. Radiographs reveal a bone age of 14.7 years.
Which one of the following would suggest the need for further evaluation?
A) A family history of delayed growth
B) Height below the fifth percentile for age
C) Weight below the fifth percentile for age
D) Prepubescent testicular size
Item 148
ANSWER: D
Most cases of short stature are due to constitutional growth delay, a term which implies that the child is
normal but delayed in his development. A hallmark of this condition is being below the fifth percentile
for height for most of childhood. Usually these children are thin and have a family history of delayed
development. Bone age would be expected to be at least 2.5 standard deviations below the mean for agematched
peers of the same chronologic age. However, most experts agree that if no signs of puberty are
seen by 14 years of age (no breast development in girls, no testicular enlargement in boys), then further
workup for a more serious condition should be sought. Other indications for evaluation would be no
menarche in a girl by 16 years of age and underdeveloped genitalia in a boy 5 years after his first pubertal
changes.
149. A 60-year-old African-American male is found to have type 2 diabetes mellitus. Which one of
the following should be ordered before initiating treatment with metformin (Glucophage)?
A) Serum electrolytes
B) A serum creatinine level
C) A CBC
D) A lipid panel
E) A thyroid panel
Item 149
ANSWER: B
Metformin is contraindicated in patients with renal dysfunction, because it is associated with an increased
incidence of lactic acidosis.
150. Which one of the following nutritional interventions should be recommended to accelerate
pressure ulcer healing in the elderly?
A) Supplemental arginine
B) Oral vitamin C and zinc
C) High-dose multivitamins
D) Adequate protein intake
Item 150
ANSWER: D
Very few nutritional interventions have been shown to accelerate pressure ulcer healing in the elderly.
Maintaining a protein intake of at least 1.2–1.5 g/kg/day is recommended, and some authorities
recommend 2 g/kg/day with stage III or IV ulcers. Increased caloric intake is also necessary to promote
healing. The role of vitamins and minerals in preventing and treating pressure ulcers is unclear.
151. According to the Health Insurance Portability and Accountability Act of 1996 (HIPAA), in
which situation would a physician be allowed to disclose personal information without the
patient’s written authorization?
A) The patient makes a verbal request to release information
B) The patient’s spouse requests information
C) The adult children of the patient request written information
D) A lawyer who claims to represent the patient requests information
E) Another physician involved in the patient’s care requests information
Item 151
ANSWER: E
HIPAA legislation states that a patient’s personal medical and financial information cannot be released
unless the patient authorizes such a release in writing. The exceptions to this standard are the following:
(1) coordination of care between providers and those involved in the patient’s case (i.e., caretakers, nurses,
consulting physicians); (2) arranging payment for medical services rendered; and (3) health-care operations
such as evaluating a provider or system’s competency or quality. The privacy rule allows some discretion
to a physician in “coordinating care,” even allowing a physician to speak with family members if that
physician “in his or her professional judgment” feels it is in the patient’s best interest. In such situations
it is advisable to ask the patient’s permission to do so if possible, and the information should be related on
a need-to-know basis.
152. The use of a corticosteroid inhaler in patients with stable chronic obstructive lung disease has
been shown to
A) increase the risk for osteoporotic fracture
B) increase the risk for pneumonia
C) produce no change in patients’ perceptions of quality of life
D) reduce overall mortality
Item 152
ANSWER: B
COPD is the fourth leading cause of death in the United States. Stopping smoking and the use of
continuous oxygen, when necessary, are the main interventions that have been shown to lessen overall
mortality in this illness.
The use of corticosteroid inhalers for COPD has received mixed reviews. Studies show an increase in the
incidence of pneumonia, which is directly related to the dosage. There are also concerns about the
potential for an increase in fractures; however, a meta-analysis of multiple studies has not shown this to
be the case. There has been no improvement in overall mortality with the use of the corticosteroid
inhalers; nevertheless, on questionnaires patients indicate an improvement in quality of life and fewer
bronchitis exacerbations.
153. Which one of the following is known to cause hyperthyroidism?
A) Propranolol (Inderal)
B) Amiodarone (Cordarone)
C) Methimazole (Tapazole)
D) Propylthiouracil
E) Methotrexate (Rheumatrex, Trexall)
Item 153
ANSWER: B
Amiodarone is 37% iodine and is the most common source of iodine excess in the United States.
Excessive iodine intake from dietary sources, radiographic contrast media, or amiodarone increases the
production and release of thyroid hormone in iodine-deficient individuals and in older persons with
multinodular goiter. Additionally, like other medications such as interferon and interleukin-2, amiodarone
can trigger thyroiditis in patients with normal thyroid glands. These characteristics combine to induce
hyperthyroidism in slightly over 10% of patients treated with amiodarone.
-Blockers such as propranolol may be useful in controlling the symptoms of hyperthyroidism.
Methimazole and propylthiouracil interfere with organification of iodine, thereby suppressing thyroid
hormone production; they are commonly used as antithyroid agents when treating hyperthyroidism.
Research is ongoing to determine if methotrexate plus prednisone is an effective treatment for the
ophthalmopathy associated with Graves’ hyperthyroidism.
154. A 20-year-old college wrestler is seen for an examination prior to the wrestling season. He tells
you that some friends have told him he should start taking dehydroepiandrosterone (DHEA), and
he asks for your advice.
Which one of the following is true about the effects of this drug?
A) It enhances performance but not muscle strength
B) It enhances muscle strength but not performance
C) It enhances both performance and muscle strength
D) It does not enhance either performance or muscle strength
Item 154
ANSWER: D
Dehydroepiandrosterone (DHEA) is illegal under the Anabolic Steroid Control Act of 2004, and is
prohibited by the NCAA and the International Olympic Committee. Like androstenedione, DHEA is a
precursor to testosterone, but neither of these substances has been shown to enhance either performance
or strength. In fact, they increase serum estrogen and luteinizing hormone levels
155. A 15-year-old white female who has had regular periods since age 12 comes to your office
because of secondary amenorrhea and a milky discharge from her breasts. A pregnancy test is
negative.
The best test for initial evaluation of the pituitary in this patient is
A) plasma antidiuretic hormone
B) plasma ACTH
C) serum prolactin
D) serum FSH and LH
E) fasting growth hormone
Item 155
ANSWER: C
Anterior pituitary hormone overproduction is suspected on clinical grounds and confirmed by appropriate
laboratory evaluation. The most common secretory pituitary adenomas are prolactinomas. They cause
galactorrhea and hypogonadism, including amenorrhea, infertility, and impotence. Growth
hormone–secreting tumors, which are the next most common secretory pituitary tumors, cause acromegaly
or gigantism. Next in frequency are corticotropic (ACTH-secreting) adenomas, which cause cortisol
excess (Cushing’s disease). Glycoprotein hormone-secreting pituitary adenomas (secreting TSH, LH, or
FSH) are the least common. TSH-secreting adenomas are a rare cause of hyperthyroidism. Paradoxically,
most patients with gonadotropin-secreting adenomas have hypogonadism.
156. A 36-year-old female consults you because of concerns about “fatigue.” After carefully
reviewing her history and performing a physical examination, which one of the following would
be LEAST valuable in assessing this patient?
A) A baseline serum cortisol level
B) An erythrocyte sedimentation rate
C) A complete metabolic panel
D) A TSH level
E) A pregnancy test
Item 156
ANSWER: A
In patients with fatigue, family physicians should complete an appropriate history and physical
examination. Laboratory studies should be considered, although the results affect management in only 5%
of patients. A baseline cortisol level would be valuable only in patients with significant findings of
Addison’s disease. In addition to an erythrocyte sedimentation rate, a complete metabolic panel, and a
TSH level, many physicians request a CBC and a urinalysis. A pregnancy test should be ordered for
women of childbearing age. No other tests have been shown to be useful unless a specific medical
condition is suspected.
157. A 45-year-old female presents with a complaint of pain and swelling in her right index finger
of 2 days’ duration. She reports that 5 days ago she had artificial nails applied, which she
removed yesterday due to the pain. She used hydrogen peroxide on the finger, but it did not
help. She denies any systemic symptoms or fever. On examination there is erythema and
swelling in the lateral nail fold of the right index finger, with purulent material noted.
Which one of the following would be the most appropriate treatment for this patient?
A) Removal of the proximal nail fold
B) Topical corticosteroids
C) Topical antibiotics
D) Topical antifungals
Item 157
ANSWER: C
This is a common presentation for acute paronychia, which typically is caused by local trauma to the nail
fold or cuticle, with resulting inoculation and infection. Topical antibiotics, with or without topical
corticosteroids, is one treatment option. Other options include warm compresses, oral antibiotics, and
incision and drainage; however, incision and drainage is not always necessary.
Removal of the proximal nail fold is used to treat chronic paronychia that is not responsive to other
treatments. Topical corticosteroids can be used alone for chronic paronychia, but if used for acute
paronychia, they should be combined with antibiotics since acute paronychia is typically caused by a
bacterial infection. Topical antifungals are a treatment option for chronic paronychia, which can be
associated with a fungal infection, but not for acute paronychia.
158. A 30-year-old male complains of the gradual onset of anterior right knee pain on climbing the
stairs. On examination there is no effusion, but there is tenderness over the medial retinaculum.
There is good ligament strength, and range of motion is normal. When the knee is extended
from 90° flexion to full extension, the patella deviates laterally.
Which one of the following would be the best initial treatment for this condition?
A) Bracing
B) Taping
C) NSAIDs
D) Arthroscopic surgery
E) Physical therapy
Item 158
ANSWER: E
This patient has patellofemoral stress syndrome. It is often called runner's knee or anterior knee pain.
The patellofemoral joint comprises the patella and femoral trochlea. The best initial treatment is physical
therapy. Bracing, taping, and medications are unlikely to have better outcomes. Arthroscopic surgery is
not indicated.
159. Which one of the following is appropriate at the routine postpartum visit?
A) A CBC
B) Screening for depression
C) Thyroid function tests
D) Glucose tolerance testing
E) A urine dipstick
Item 159
ANSWER: B
Screening for postpartum depression is recommended as part of the routine postpartum visit. The use of
a screening tool for depression is recommended, such as the Edinburgh Postnatal Depression Scale. This
scale has been shown to increase the identification of women at high risk for depression. A CBC or urine
dipstick is recommended only for patients who have an indication for them, and should not be routinely

ordered. Thyroid function tests and glucose tolerance testing are recommended for patients who are either
symptomatic or at high risk for disease.
160. The induction or inhibition of the cytochrome P450 (CYP) enzyme is responsible for many
adverse drug reactions. Which one of the following is an inducer of the cytochrome P450
enzyme?
A) Ciprofloxacin (Cipro)
B) Fluconazole (Diflucan)
C) Phenytoin (Dilantin)
D) Clarithromycin (Biaxin)
E) Grapefruit juice
Item 160
ANSWER: C
All of the drugs listed are inhibitors of the cytochrome P450 enzyme except phenytoin, which is a potent
inducer. Grapefruit juice is also a cytochrome P450 enzyme inhibitor.
161. A 72-year-old Hispanic female with moderately severe Alzheimer’s disease is hospitalized for
treatment of a fracture of the left humerus. The first night after admission she becomes confused
and agitated.
The most appropriate management at this point is which one of the following?
A) Physical restraints
B) Benzodiazepines for agitation
C) Meperidol (Demerol) for pain control
D) Moving the patient to the intensive-care unit for better monitoring
E) Having a bedsitter or family member stay in the room to reassure and orient the patient
Item 161
ANSWER: E
Delirium is a frequent complication of hospital admission in older patients, especially those with
preexisting dementia. Orientation and reassurance in a quiet environment will usually be effective in
treating the confusion and agitation, once serious causes of the delirium have been ruled out.
Benzodiazepines and meperidol have been reported to cause delirium. Physical restraints and restrictive
environments (e.g., intensive-care units) can predispose to delirium and are best avoided if possible.
162. A 75-year-old otherwise healthy white female states that she has passed out three times in the
last month while walking briskly during her daily walk with the local senior citizens mall
walkers’ club. This history would suggest which one of the following as the etiology of her
syncope?
A) Vasovagal syncope
B) Transient ischemic attack
C) Orthostatic hypotension
D) Atrial myxoma
E) Aortic stenosis
Item 162
ANSWER: E
Syncope with exercise is a manifestation of organic heart disease in which cardiac output is fixed and does
not rise (or even fall) with exertion. Syncope, commonly on exertion, is reported in up to 42% of patients
with severe aortic stenosis. Vasovagal syncope is associated with unpleasant stimuli or physiologic
conditions, including sights, sounds, smells, sudden pain, sustained upright posture, heat, hunger, and
acute blood loss. Transient ischemic attacks are not related to exertion. Orthostatic hypotension is
associated with changing from a sitting or lying position to an upright position. Atrial myxoma is
associated with syncope related to changes in position, such as bending, lying down from a seated position,
or turning over in bed.
163. Overweight and obesity in children should be determined by which one of the following?
A) Body weight
B) BMI percentile for age and gender
C) Individual BMI
D) Abdominal girth
E) Percentage of body fat
Item 163
ANSWER: B
In children, overweight and obesity is determined by the BMI percentile for age and gender. In adults,
BMI, body fat percentage, and abdominal girth are used to determine a patient’s classification
164. Which one of the following is the preferred treatment for scabies?
A) Topical benzoyl peroxide, 10%
B) Topical crotamiton (Eurax), 10%
C) Topical permethrin (Elimite), 5%
D) Topical lindane, 1%
E) Oral ivermectin (Stromectol), 200 mg
tem 164
ANSWER: C
Permethrin and lindane are the two most studied topical treatments for scabies. A Cochrane meta-analysis
of four randomized trials comparing these agents indicates that a single overnight application of permethrin
is more effective than lindane (odds ratio for clinical failure, 0.66; 95% confidence interval, 0.46–0.95).
The potential neurotoxicity of lindane, especially with repeated applications, has limited its use.
Other topical treatments include benzoyl benzoate and crotamiton. Crotamiton has significantly less
efficacy than permethrin at 4 weeks (61% versus 89%). Several controlled trials have assessed the efficacy
of a single dose of ivermectin (200 g/kg) for the treatment of scabies. In one placebo-controlled trial,
37 of 50 patients treated with ivermectin (74%) were cured.
165. A patient in the first trimester of pregnancy has just learned that her husband has acute hepatitis
B. She feels well, and her screening test for hepatitis B surface antigen (HBsAg) was negative
last month. She has not been immunized against hepatitis B.
Which one of the following would be the most appropriate management of this patient?
A) No further workup or immunization at this time, a repeat HBsAg test near term, and
treatment of the newborn if the test is positive
B) Use of condoms for the remainder of the pregnancy, and administration of immunization
after delivery
C) Testing for hepatitis B immunity (anti-HBs), and immunization if needed
D) Administration of hepatitis B immune globulin (HBIG) now and hepatitis B vaccine after
the first trimester
E) Administration of both HBIG and hepatitis B vaccine now
Item 165
ANSWER: E
Hepatitis B immune globulin (HBIG) should be administered as soon as possible to patients with known
exposure to hepatitis B. Hepatitis B vaccine is a killed-virus vaccine and can be used safely in pregnancy,
with no need to wait until after organogenesis. This patient has been exposed to sexual transmission for
at least 6 weeks, given that the incubation period is at least that long, so it is too late to use condoms to
prevent infection. The patient is unlikely to be previously immune to hepatitis B, given that she has no
history of hepatitis B infection, immunization, or carriage. Because the patient's HBsAg is negative, she
is not the source of her husband's infection. Full treatment for this patient has an efficacy of only 75%,
so follow-up testing is still needed.
166. A 55-year-old white male notices a nodular thickening over the flexor tendons in his medial
palm. He has no difficulty using his hand, and he is able to lay his palm flat on a tabletop. You
suspect Dupuytren's disease.
Which one of the following is true regarding this condition?
A) There is a strong association with diabetes mellitus
B) Surgical intervention is recommended at this point to prevent progression to contracture
C) Once a contracture develops, it is irreversible and no treatment is indicated
D) A single cortisone injection often leads to disease regression in mild to moderate cases
E) A search for an occult malignancy is indicated
Item 166
ANSWER: A
Dupuytren’s disease is characterized by shortening and thickening of the palmar fascia. It is initially
asymptomatic, but may progress and cause difficulty with function of the hand, and may eventually lead
to contracture. Early asymptomatic disease does not require treatment. A series of cortisone injections
over a period of months may lead to disease regression, and is useful in patients with mild to moderate

symptoms. Surgery is indicated if a metacarpal joint contracture reaches 30°, or with a proximal
interphalangeal joint contracture of any degree. If surgery is delayed, irreversible joint contracture may
occur.
There is a strong association between diabetes mellitus and Dupuytren’s disease, with up to a third of
diabetic patients having evidence of the disease. It is also associated with alcohol use and smoking.
Patients requiring surgery have an increased risk of dying from cancer, probably related to smoking,
alcohol use, or diabetes mellitus, but a search for cancer at the time of diagnosis is not indicated.
167. A 29-year-old white female is hospitalized following a right middle cerebral artery stroke
confirmed by MRI. Her past medical history is remarkable only for a history of an
uncomplicated tonsillectomy during childhood and a second-trimester miscarriage 3 years ago.
The only remarkable finding on physical examination is left hemiplegia.
The initial laboratory workup reveals normal hematocrit and hemoglobin levels, a normal
prothrombin time, and a platelet count of 200,000/mm3 (N 140,000–440,000). The activated
partial thromboplastin time is 95 sec (N 23.6–34.6), and it does not normalize when the patient’s
serum is mixed with normal plasma. A serum VDRL is positive, and a serum FTA-ABS is
nonreactive.
Which one of the following is the most likely diagnosis?
A) Hemophilia
B) Neurosyphilis
C) Antiphospholipid syndrome
D) Thrombotic thrombocytopenic purpura
E) Protein C deficiency
Item 167
ANSWER: C
The antiphospholipid syndrome is due to the appearance of a heterogeneous group of circulating antibodies
to negatively charged phospholipids, including most commonly a lupus anticoagulant and anticardiolipin
antibodies. The antibodies are usually detected by a false-positive serologic test for syphilis. Clinical
features include venous and arterial thrombosis, fetal wastage, thrombocytopenia, and the presence of an
activated partial thromboplastin time (aPTT) inhibitor. It is an important diagnostic consideration in all
patients with unexplained thrombosis or cerebral infarction, particularly in young patients.
Although hemophilia would also be associated with a prolonged aPTT, the PTT would normalize when
the patient’s serum was mixed with normal plasma. Neurosyphilis is excluded by the negative serum FTAABS
result. Thrombotic thrombocytopenic purpura is not associated with prolongation of the aPTT and
is associated with a hemolytic anemia. Although protein C deficiency is a hypercoagulable state that can
lead to stroke, none of the laboratory abnormalities suggests this diagnosis.
168. A 12-year-old male uses a short-acting bronchodilator three times per week to control his
asthma. Lately he has been waking up about twice a week due to his symptoms.
Which one of the following medications would be most appropriate?
A) Inhaled medium-dose corticosteroids
B) A scheduled short-acting bronchodilator
C) A scheduled long-acting bronchodilator
D) A leukotriene inhibitor
Item 168
ANSWER: A
This patient has moderate persistent asthma. Although many parents are concerned about corticosteroid
use in children with open growth plates, inhaled corticosteroids have not been proven to prematurely close
growth plates, and are the most effective treatment with the least side effects. Scheduled use of a shortacting
bronchodilator has been shown to cause tachyphylaxis, and is not recommended. The same is true
for long-acting bronchodilators. Leukotriene use may be beneficial, but compared to those using inhaled
corticosteroids, patients using leukotrienes are 65% more likely to have an exacerbation requiring systemic
corticosteroids.
169. In the United States, the number of deaths has increased in recent years for which one of the
following vaccine-preventable illnesses?
A) Tetanus
B) Hepatitis C
C) Rubella
D) Pertussis
E) West Nile virus
Item 169
ANSWER: D
In the United States, deaths from pertussis increased from 4 deaths in 1996 to 17 deaths in 2001, and a
total of 56 deaths from 2001 to 2003. Immunity has decreased in previously vaccinated adolescents and
adults, and now they are a reservoir for infection. Tdap vaccine is recommended as a single booster for
patients age 19–65, and those between the ages of 11 and 18 years should receive Tdap rather than a Td
booster. The Tdap vaccine protects against pertussis, in addition to tetanus and diphtheria. Tetanus and
rubella deaths are not increasing. There are no vaccines for hepatitis C or West Nile virus.
170. An 80-year-old female is seen for progressive weakness over the past 8 weeks. She says she
now has difficulty with normal activities such as getting out of a chair and brushing her teeth.
Her medical problems include hypertension, diabetes mellitus, and hyperlipidemia. Her
medications include glipizide (Glucotrol), simvastatin (Zocor), and lisinopril (Prinivil, Zestril).
Findings on examination are within normal limits except for diffuse proximal muscle weakness
and normal deep tendon reflexes. A CBC, urinalysis, erythrocyte sedimentation rate, TSH
level, and serum electrolyte levels are normal. Her blood glucose level is 155 mg/dL, and her
creatine kinase level is 1200 U/L (N 40–150).
Which one of the following is the most likely diagnosis?
A) Statin-induced myopathy
B) Polymyalgia rheumatica
C) Guillain-Barré syndrome
D) Diabetic ketoacidosis
Item 170
ANSWER: A
This patient is most likely suffering from a drug-induced myopathy caused by the simvastatin, which is
associated with elevated creatine kinase. Polymyalgia rheumatica is usually associated with an elevated
erythrocyte sedimentation rate. Guillain-Barré syndrome is associated with depressed deep tendon
reflexes. This case has no clinical features or laboratory findings that suggest ketoacidosis.
171. Studies indicate that patients most frequently want physicians to ask about their spiritual beliefs
in which one of the following situations?
A) When being treated for a potentially fatal illness
B) During the annual preventive visit
C) During the initial office visit with the physician
D) Only if specifically requested by the patient, a family member, their minister, or a
chaplain
E) When prayer is suggested by the patient or physician
Item 171
ANSWER: A
Patients often welcome spiritual discussion, depending on the situation. The percentage that welcome this
discussion increases with the severity of illness, and is greatest among those who are very seriously ill with
a potentially fatal disease. Spiritual inquiry during medical care should focus on understanding,
compassion, and hope, and should be directed toward individuals who suffer from serious illness.
172. Which one of the following has the best evidence of effectiveness for preventing fractures in
postmenopausal women with osteoporosis?
A) Home-hazard assessment
B) Daily supplementation with vitamin D
C) Treatment with calcitonin
D) Treatment with alendronate (Fosamax)
Item 172
ANSWER: D
Of the options listed, treatment with bisphosphonates to prevent osteoporotic hip and vertebral fractures
is the only one supported by consistent patient-oriented, high-quality clinical evidence (SOR A). While
each of the other recommendations has merit, the overall level of evidence for effectiveness is less
compelling for these treatments than for treatment with bisphosphonates (SOR B).
173. To be eligible for Medicare hospice benefits, a patient must
A) be enrolled in Medicare Part D
B) be referred to hospice by a physician
C) be debilitated and moribund
D) have a malignancy
E) have an estimated life expectancy of less than 6 months
Item 173
ANSWER: E
To be eligible for Medicare hospice benefits, a patient must be eligible for Medicare Part A (hospital
insurance). Although most hospice referrals come from physicians, nurses, and social workers, a patient’s
family members can also make a hospice referral. The patient must sign a statement choosing hospice,
and both the patient’s physician and the hospice medical director must certify that the patient has a terminal
illness with an estimated life expectancy of less than 6 months. There is no requirement that the patient
be debilitated or moribund.
174. A 39-year-old female presents with lower abdominal/pelvic pain. On examination, with the
patient in a supine position, you palpate the tender area of her abdomen. When you have her
raise both legs off the table while you palpate the abdomen, her pain intensifies.
Which one of the following is the most likely diagnosis?
A) Appendicitis
B) A hematoma within the abdominal wall musculature
C) Diverticulitis
D) Pelvic inflammatory disease
E) An ovarian cyst
Item 174
ANSWER: B
Carnett’s sign is the easing of the pain of abdominal palpation with tightening of the abdominal muscles.
If the cause is visceral, the taut abdominal muscles could guard the source of pain from the examining
hand. In contrast, intensification of pain with this maneuver points to a source of pain within the
abdominal wall itself.
175. A 14-year-old female sees you for follow-up after hypercalcemia is found on a chemistry profile
obtained during a 5-day episode of vomiting and diarrhea. She is now asymptomatic, but her
serum calcium level at this visit is 11.0 mg/dL (N 8.5–10.5). Her aunt underwent unsuccessful
parathyroid surgery for hypercalcemia a few years ago.
Which one of the following laboratory findings would suggest a diagnosis other than primary
hyperparathyroidism?
A) Low 24-hour urine calcium
B) Decreased serum phosphate
C) High-normal to increased serum chloride
D) Elevated alkaline phosphatase
E) Elevated parathyroid hormone
Item 175
ANSWER: A
Low urine 24-hour calcium levels or a low urine calcium to urine creatinine ratio is not characteristic of
hyperparathyroidism. This finding should suggest familial hypocalciuric hypercalcemia (SOR C).
Awareness of this condition is important to avoid unnecessary surgery. The parathyroid hormone level
may be mildly elevated.
Parathyroid hormone is elevated in hyperparathyroidism. Serum chloride tends to be high normal or
mildly elevated. Alkaline phosphatase may be elevated in more severe cases, while serum phosphate levels
tend to be low.
176. A 70-year-old African-American male who has been hospitalized for 2½ weeks for heart failure
develops severe, persistent diarrhea. For the past 3 days he has had abdominal cramps and
profuse, semi-formed stools without mucus or blood.
The patient’s current medications include captopril (Capoten), digoxin, furosemide (Lasix),
subcutaneous heparin, spironolactone (Aldactone), and loperamide (Imodium). He has coronary
artery disease, but has been relatively pain free since undergoing coronary artery bypass surgery
4 years ago. An appendectomy and cholecystectomy were performed in the past, and the patient
has since been free of gastrointestinal disease.
On physical examination his blood pressure is 100/80 mm Hg, pulse 100 beats/min and regular,
and temperature 37.0°C (98.6°F). He has mild jugular venous distention and crackles at both
lung bases. Examination of his heart is unremarkable, although there is 1+ dependent edema.
His abdomen is diffusely tender without masses or organomegaly. Findings on a rectal
examination are normal.
The results of routine laboratory tests, including a CBC, chemistry profile, EKG, and urinalysis,
are all normal. The stool examination shows numerous white blood cells.
Of the following, the most likely diagnosis is
A) viral gastroenteritis
B) Clostridium difficile colitis
C) ulcerative colitis
D) gluten-sensitive enteropathy (celiac sprue)
E) digoxin toxicity
Item 176
ANSWER: B
This patient most likely has Clostridium difficile colitis, suggested by semiformed rather than watery stool,
fecal leukocytes (not seen in viral gastroenteritis or sprue), and a hospital stay greater than 2 weeks. While
this disease has traditionally been associated with antibiotic use, it is posing an increasing threat to patients
54
in hospitals and chronic-care facilities who have not been given antibiotics. The primary sources for
infection in such cases have been toilets, bedpans, floors, and the hands of hospital personnel. Prompt
recognition and treatment is essential to prevent patient relapse and to minimize intramural epidemics. The
diarrhea of ulcerative colitis usually contains blood and occurs intermittently over a protracted course.
Digoxin toxicity is likely to be accompanied by electrocardiographic and laboratory abnormalities,
particularly hyper- or hypokalemia.
177. A 63-year-old female with type 2 diabetes mellitus presents to the emergency department with
unstable angina. Her blood pressure is 150/90 mm Hg, her pulse rate is 70 beats/min, and her
lungs are clear to auscultation.
The patient expresses a preference for conservative (i.e., noninvasive) therapy. In addition to
aspirin, which one of the following agents should be administered at this time?
A) Clopidogrel (Plavix) orally
B) Indomethacin (Indocin) orally
C) Nifedipine (Procardia) immediate-release, orally
D) Abciximab (ReoPro) intravenously
E) Enalaprilat intravenously
Item 177
ANSWER: A
An oral loading dose of clopidogrel should be administered as soon as possible in patients with unstable
angina/NSTEMI who are to be treated conservatively. The standard dosage should then be prescribed,
to be taken daily for at least 1 month along with aspirin (SOR B).
Immediate-release calcium channel antagonists such as nifedipine are not indicated. If -blockers are
contraindicated, verapamil or diltiazem would be the preferred agents. Intravenous ACE inhibitors may
induce shock and should be avoided in the first 24 hours.
Abciximab is used for patients who will undergo rapid catheterization with a significant chance of acute
coronary intervention. NSAIDs are contraindicated because they may weaken areas of damaged
myocardium and increase the risk of rupture, and may also increase the risk of infarction or extension.
They have been used in the past for treatment of associated pericarditis, which most frequently develops
a few days after presentation, but are now avoided.
178. Increasing patient copayments for prescription medications results in
A) an increase in the number of prescriptions filled by low-income medical-assistance
recipients
B) little demonstrable change in purchasing patterns
C) increased hospitalizations for patients with chronic illnesses
D) improved efficiency in the utilization of outpatient medical services
Item 178
ANSWER: C
Increasing prescription copayments results in a decrease in the number of prescriptions filled and
worsening clinical outcomes for patients with heart failure, diabetes mellitus, hyperlipidemia, and
schizophrenia. With each 10% increase in copayments, it is estimated that overall prescription spending
decreases 2%–6%. The cited study found that up to 25% of Medicaid recipients, faced with a copayment,
could not afford to fill at least one prescription in the previous year.
179. A 25-year-old female has an annular rash on the dorsal surface of both hands. The rash does
not respond to initial treatment with an antifungal medication, and a biopsy reveals granuloma
annulare.
Which one of the following would be the most appropriate advice for this patient?
A) Allow the rash to resolve without further treatment
B) Cover the rash because it is contagious
C) Treat the rash with systemic corticosteroids
D) Treat the rash with a stronger antifungal medication
Item 179
ANSWER: A
Granuloma annulare is a self-limited condition. It is not contagious, and therefore would not need to be
covered to prevent transmission. Treatments may include injected or topical corticosteroids, but oral
corticosteroids have not been specifically recommended. It may be necessary to refer the patient to a
dermatologist because many of the potential treatments can have serious side effects.
180. A 24-year-old female who works at a day-care facility presents to your office to discuss ways
to avoid getting “all the infections the kids get.” She plans to enroll her child in the facility.
She is specifically concerned about diarrheal illnesses, and a friend has suggested the use of
probiotics.
You tell her that probiotics
A) can lessen the severity and duration of infectious diarrhea
B) are recommended only for patients who are immunocompromised
C) have no known side effects
D) often interact with common prescription medications
E) are not appropriate for use in children
Item 180
ANSWER: A
Probiotics are microorganisms with likely health benefits, based on recent randomized, controlled trials.
Good evidence suggests that probiotics reduce the incidence, duration, and severity of antibiotic-associated
and infectious diarrhea. Common side effects include flatulence and abdominal pain. Contraindications
include short-gut syndromes and immunocompromised states. There are no known drug interactions, and
these agents appear safe for all ages (SOR A).
181. The recommended time to screen for gestational diabetes in asymptomatic women with no risk
factors for this condition is
A) in the first trimester
B) at 16–20 weeks gestation
C) at 24–28 weeks gestation
D) at 35–37 weeks gestation
Item 181
ANSWER: C
The recommended time to screen for gestational diabetes is 24–28 weeks gestation. The patient may be
given a 50-g oral glucose load followed by a glucose determination 1 hour later.
182. The 1990 Patient Self-Determination Act requires that
A) the process for advance directives be standardized for all 50 states
B) a living will be implemented for patients upon admission to the hospital
C) hospitals ask patients about advance directives
D) verbally expressed wishes be honored for individuals who do not have a written advance
directive
Item 182
ANSWER: C
The 1990 Patient Self-Determination Act (PSDA) requires hospitals, nursing homes, and health care
programs to ask patients about advance directives and then incorporate the information into medical
records. The living will, a written advance directive, allows a competent person to indicate his or her
health care preferences while cognitively and physically healthy. A living will may list medical
interventions the patient wishes to have withheld or withdrawn when he or she becomes unable to
communicate. Another type of advance directive, the durable power of attorney for health care, allows
persons to designate a proxy (or surrogate) to make decisions for them if they become incapacitated.
Although PSDA mandates that patients be asked about their advance directive status upon admission to the
hospital, it does not require hospitals or individual physicians to offer patients an opportunity to complete
an advance directive. The acceptance and precision of verbal preferences varies from state to state.
Although verbal discussions are binding in many states, five states require “clear and convincing evidence
of patient preferences.” In California, Delaware, Michigan, Missouri, and New York, advance directives
must include such evidence regarding a specific condition and/or treatment, even if a durable power of
attorney states prior general verbal preferences. Therefore, lack of an advance directive may result in
continued medical interventions to preserve life even if the patient may not want such treatment.
183. A 55-year-old male consults you because he wants to begin an exercise program. He is
asymptomatic, but because of his family history you determine that he should undergo a stress
test with echocardiography.
Which one of the following would be considered a normal ejection fraction in this patient?
A) 48%
B) 65%
C) 76%
D) 84%
E) 92%
Item 183
ANSWER: B
The ejection fraction value is an important measure of left ventricular function, especially with regard to
previous cardiac events, medications, exercise tolerance, and preoperative risk. The normal predicted
value is 55%–75% when measured by echocardiography in a healthy asymptomatic patient. There is no
gender difference, but there is a decline with age. It may be as low as 15% in patients with left ventricular
dysfunction. Ischemic and valvular heart disease may significantly reduce the ejection fraction.
184. A 42-year-old asymptomatic female presents for a routine evaluation. On examination her uterus
is irregularly enlarged to the size seen at approximately 8 weeks gestation. Pelvic
ultrasonography shows several uterine fibroid tumors measuring <5 cm. The patient does not
desire future fertility.
Which one of the following would be the most appropriate treatment option?
A) Laparoscopic myomectomy
B) Hysterectomy
C) A gonadotropin-releasing hormone (GnRH) agonist
D) An oral contraceptive
E) Observation
Item 184
ANSWER: E
Uterine fibroid tumors (leiomyomas) are the most common female reproductive tract tumors, with some
evidence suggesting that the cumulative incidence in women ages 25–45 is approximately 30%. Symptoms
related to fibroids can include menorrhagia, pelvic pain, obstructive symptoms, infertility, or pregnancy
loss. However, many fibroids are asymptomatic and are discovered incidentally, with expectant
management being the treatment of choice in this situation (SOR B). The risk of malignant
leiomyosarcoma is exceedingly small (0.23% in one study) and there is a risk of side effects or
complications from other treatment modalities.
For women who are symptomatic, the data is insufficient to allow conclusions to be made about the most
appropriate therapy. Surgical options include myomectomy, hysterectomy, uterine artery embolization,
and myolysis, but data to allow direct comparison is lacking. With the exception of trials of GnRH-agonist
therapy as an adjunct to surgery, there is not enough randomized trial data to support the use of medical
therapies (oral contraceptives, NSAIDs, progestins) in the treatment of women with symptomatic fibroids.
185. A 27-year-old male with a diagnosis of depression prefers to avoid pharmacologic treatment.
You agree to engage in a trial of therapy in your office. During the treatment process, you help
the patient realize that some of his perceptions and interpretations of reality may be false and
lead to negative thoughts. Next, you help him discover alternative thoughts that reflect reality
more closely, and to learn to discard his previous distorted thinking. By learning to substitute
healthy thoughts for negative thoughts, he finds his mood, behavior, and physical reaction to
different situations are improved.
Which one of the following best categorizes this type of therapy?
A) Psychoanalysis
B) Biofeedback
C) Cognitive therapy
D) Group psychotherapy
E) Hypnosis therapy
Item 185
ANSWER: C
This patient is engaged in cognitive therapy, which is a treatment process that helps patients correct false
self-beliefs that can lead to negative moods and behaviors. Cognitive therapy has been shown to
effectively treat patients with unipolar major depression, and is particularly useful in patients who do not
respond to medication or who prefer nonpharmacologic therapy.
Psychoanalysis is a process of free association where repressed memories are recovered. Biofeedback
involves instrumentation that gives feedback about a patient’s physiologic response to various situations
in order to bring the autonomic nervous system under voluntary control. Group psychotherapy is a form
of treatment in which people who are emotionally ill meet in a group guided by a trained therapist and help
one another effect personality change. Hypnosis involves helping a patient enter a state of heightened focal
concentration and receptivity that is typified by a feeling of involuntariness or an altered state of
consciousness.
186. A 24-year-old male sustains a boxer’s fracture of the fifth metacarpal. A radiograph shows no
rotational deformity and 25° of volar angulation. After an attempt at closed reduction the
angulation remains unchanged.
Which one of the following would be most appropriate at this time?
A) Open reduction
B) Placement of a pin to prevent further displacement
C) A short arm-thumb spica cast
D) An ulnar gutter splint
Item 186
ANSWER: D
Up to 40° of volar angulation is acceptable for fifth metacarpal fractures. For second and third metacarpal
fractures, less angulation is acceptable. Appropriate treatment is a gutter splint.
187. What is the recommended compression-to-breath ratio for basic life support with a single rescuer
for a 2-year-old child?
A) 10:2
B) 15:2
C) 20:2
D) 25:2
E) 30:2
Item 187
ANSWER: E
For a single rescuer performing CPR on a 2-year-old, the ratio is 30 compressions to 2 ventilations. The
compression rate should be approximately 100 beats/min, and the chest should be compressed one-third
to one-half its depth with each compression. Compressions can be accomplished with one hand, the heel
of one or both hands, or the heel of one hand with the second hand on top.
188. Which one of the following is associated with an increased risk of delirium in hospitalized
patients?
A) Not having family members present at the time of admission
B) Vision or hearing impairment
C) Repeated reorientation for patients with cognitive impairment
D) Early mobilization
Item 188
ANSWER: B
Delirium occurs in 11%–42% of hospitalized patients. Risk factors for delirium in hospitalized patients
include vision impairment, hearing impairment, dehydration, immobility, cognitive impairment, and sleep
deprivation. Repeated reorientation of patients with cognitive impairment, early mobilization, and
minimizing unnecessary noise or stimulation are all effective interventions for reducing the risk of delirium
in hospitalized patients. Not having family members available at the time of admission has no effect on
the incidence of delirium.
189. A 70-year-old female becomes psychotic and risperidone (Risperdal) is prescribed. Which one
of the following should be used to monitor the patient for adverse cardiac effects of this drug?
A) Serum sodium levels
B) Echocardiography
C) Nuclear stress testing
D) Lower-extremity venous duplex ultrasonography
E) Electrocardiography
Item 189
ANSWER: E
Both typical and atypical antipsychotics can cause prolongation of the QTc interval, resulting in torsades
de pointes, ventricular tachycardia, and sudden death. The best way of monitoring the QTc interval is
electrocardiography.
190. Which one of the following historical or audiographic findings in an elderly person would
indicate that hearing loss is due to something other than presbycusis?
A) Conductive hearing loss
B) Bilateral hearing loss
C) Symmetric hearing loss
D) Gradual hearing loss
E) High-frequency hearing loss
Item 190
ANSWER: A
Presbycusis, the hearing loss associated with aging, is gradual in onset, bilateral, symmetric, and
sensorineural.
191. Which one of the following sonographic measurements is most accurate for estimating
gestational age?
A) Amniotic sac size at 5 weeks of pregnancy
B) Crown-rump length at 10 weeks of pregnancy
C) Femur length at 16 weeks of pregnancy
D) Biparietal diameter at 20 weeks of pregnancy
E) Abdominal circumference at 24 weeks of pregnancy
Item 191
ANSWER: B
Estimation of gestational age by ultrasound is most accurate early in the first trimester and begins to
decline by 22 weeks gestation. Crown-rump length is typically used to estimate gestational age before 13
weeks gestation. After 11 weeks gestation, combinations of biparietal diameter, femur length, head
circumference, and abdominal circumference are used to estimate the gestational age. These factors are
used by the software that generates ultrasonography reports.
192. Which one of the following treatment regimens is most appropriate for an HIV-positive 42-yearold
who has latent tuberculosis infection?
A) Isoniazid daily for 9 months
B) Rifampin (Rifadin) daily for 4 months
C) Rifampin plus pyrazinamide daily for 2 months
D) Combination therapy with isoniazid, rifampin, pyrazinamide, and ethambutol
(Myambutol) for 2 months
Item 192
ANSWER: A
Latent tuberculosis infection carries a risk of progression to active disease, especially among patients who
are immunosuppressed. Isoniazid monotherapy is the treatment of choice for most patients with latent
tuberculosis infection. Rifampin is not recommended as monotherapy in patients with HIV infection
because of increased rates of resistance and drug interactions with many antiretrovirals. Rifampin plus
pyrazinamide is no longer recommended for treatment of latent tuberculosis infection because cases of
significant hepatotoxicity have occurred with preventive therapy. Combination drug therapy is reserved
for treatment of active tuberculosis in order to prevent drug resistance.
193. A 75-year-old female presents with a 2-month history of bilateral headache, diffuse myalgias,
and diplopia. On examination she has substantially diminished vision in her left eye, but no
other neurologic findings. A moderately tender, cordlike structure is palpable just anterior to
her ear and extending up to her lateral scalp. Blood tests show a markedly elevated erythrocyte
sedimentation rate.
Which one of the following would be most appropriate at this point?
A) Clopidogrel (Plavix)
B) High-dose corticosteroids
C) NSAIDs
D) Dipyridamole/aspirin (Aggrenox)
Item 193
ANSWER: B
The clinical findings in this patient are consistent with temporal arteritis: age over 50, new-onset headache,
abnormalities of the temporal artery, and an elevated erythrocyte sedimentation rate. A temporal artery
biopsy is needed to confirm the diagnosis, but when the findings are this compelling, corticosteroids should
be started even before a biopsy, to prevent further vision loss. Temporal arteritis is the most common
clinical pattern of giant cell arteritis, which can also involve other branches of the carotid artery.
194. A 25-year-old female presents with a maculopapular rash that has progressed to multiple areas
and exhibits target lesions. A cold sore appeared on her upper lip 2 days before the rash
appeared. She is not systemically ill and is on no medications.
Which one of the following is true concerning this problem?
A) Herpes simplex virus is a likely cause
B) A skin biopsy will confirm the diagnosis
C) The lesions usually disappear within 24 hours
D) The palms of the hands and soles of the feet are not involved
E) Scarring from the lesions is often seen after resolution
Item 194
ANSWER: A
Herpes simplex virus is the most common etiologic agent of erythema multiforme. Other infections,
particularly Mycoplasma pneumoniae infections and fungal infections, may also be associated with this
hypersensitivity reaction. Other causes include medications and vaccines. Skin biopsy findings are not
specific for erythema multiforme. As opposed to the lesions of urticaria, the lesions of erythema
multiforme usually are present and fixed for at least 1 week and may evolve into target lesions. The palms
of the hands and soles of the feet may be involved. The lesions of erythema multiforme usually resolve
spontaneously over 3–5 weeks without sequelae.
195. A 44-year-old male sees you for evaluation of an episode of pink-tinged urine last week. He
denies any flank or abdominal pain, as well as frequency, urgency, and dysuria. He has no
prior history of renal or other urologic disease, and no other significant medical problems. He
has a 24-pack-year smoking history.
A urinalysis today reveals 8–10 RBCs/hpf. You refer him to a urologist for cystoscopy.
Which one of the following would be the most appropriate additional evaluation?
A) KUB radiography
B) Transabdominal ultrasonography
C) Voiding cystourethrography
D) CT urography
E) Magnetic resonance urography
Item 195
ANSWER: D
CT urography or intravenous pyelography is recommended by the American College of Radiology as the
most appropriate imaging procedure for hematuria in all patients, with the exception of those with
generalized renal parenchymal disease, young women with hemorrhagic cystitis, children, and pregnant
females.
196. The test of choice for immediate evaluation of an acutely swollen scrotum is
A) a pelvic radiograph
B) radionuclide imaging
C) color Doppler ultrasonography
D) CT
E) MRI
Item 196
ANSWER: C
Any patient with a new scrotal mass should be evaluated immediately because of the risk of potential
emergencies, such as testicular torsion, or of life-threatening diseases such as testicular carcinoma. Color
Doppler ultrasonography is the test of choice for immediate evaluation of scrotal masses (SOR B) because
it can be done quickly and has a high sensitivity (86%–88%) and specificity (90%–100%) for detecting
testicular torsion, which is a surgical emergency.
197. A 34-year-old white female visits your office complaining of a sore throat. She takes
haloperidol, 2 mg after each meal, for schizophrenia, and you notice that she seems unable to
sit still and is extremely anxious.
The most likely diagnosis is
A) drug-induced parkinsonism
B) akathisia
C) tardive dyskinesia
D) hysteria
E) dystonia
Item 197
ANSWER: B
Motor side effects of the antipsychotic drugs can be separated into five general categories: dystonias,
parkinsonism, akathisia, withdrawal dyskinesias, and tardive dyskinesia. Akathisia is a syndrome marked
by motor restlessness. Affected patients commonly complain of being inexplicably anxious, of being
unable to sit still or concentrate, and of feeling comfortable only when moving. A diagnosis of hysteria
is inconsistent with the findings presented.
198. A 58-year-old male complains of leg claudication. Subsequent tests reveal that he has significant
bilateral peripheral arterial disease. His current medications include atenolol (Tenormin), 50
mg/day, and aspirin, 325 mg/day. His blood pressure is 128/68 mm Hg, and his pulse rate is
64 beats/min. His LDL-cholesterol level is 123 mg/dL.
The addition of which one of the following could reduce this patient’s symptoms?
A) Epoetin alfa (Epogen)
B) Nifedipine (Procardia)
C) Simvastatin (Zocor)
D) Testosterone supplementation
E) Warfarin (Coumadin) titrated to an INR of 2.0–3.0
Item 198
ANSWER: C
Peripheral arterial disease (PAD) is a common malady that has several proven treatments. The outcomes
of these treatments can be separated into two primary categories: reducing PAD symptoms and preventing
death due to systemic cardiovascular events (CVEs), especially myocardial infarction. Routine exercise
up to near-maximal pain on a regular basis has been shown to be one of the most effective treatments for
symptoms of PAD. Smoking cessation and aspirin are also standard recommendations, and can both
prevent CVEs and slow the rate of progression of PAD symptoms.
Statin drugs (specifically simvastatin and atorvastatin) have been shown to be beneficial for treatment of
PAD symptoms and prevention of CVEs through the reduction of cholesterol, but they also appear to have
other properties that help reduce leg pain in patients with PAD. Although lowering abnormally high blood
pressure is recommended in PAD patients, only ACE inhibitors have been shown to reduce symptoms of
PAD directly. Furthermore, the combination of atenolol and nifedipine has actually been shown to worsen
symptoms of PAD. The addition of warfarin to aspirin has no additional benefit in either reduction of
PAD symptoms or prevention of CVEs, but it may have a role in preventing clots in patients who have
undergone revascularization.
199. A 55-year-old female sees you because of a constant leakage of small amounts of urine. Her
obstetric/gynecologic history includes two pregnancies, with vaginal deliveries. Her current
medications include hydrochlorothiazide, metformin (Glucophage), and glyburide (DiaBeta).
On examination she has mild diabetic retinopathy, decreased sensation to monofilament testing
on her feet, and suprapubic fullness.
The most appropriate initial treatment for this problem would be
A) tolterodine (Detrol LA)
B) duloxetine (Cymbalta)
C) estrogen replacement therapy
D) bladder neck needle suspension
E) a set schedule for urination
Item 199
ANSWER: E
There are four types of urinary incontinence in women: functional incontinence, which occurs when the
patient's inability to ambulate or transfer results in loss of urine; urinary stress incontinence, which is a
result of pelvic relaxation and is manifested as involuntary loss of urine with increases in abdominal
pressure such as that which occurs with laughing, sneezing, or coughing; detrusor instability or overactive
bladder, which is when the urge to urinate is quickly followed by loss of urine, usually a large volume;
and neurogenic bladder, which is marked by constant leakage of small amounts of urine.
Neurogenic bladder can be caused by diabetes mellitus, multiple sclerosis, or spinal cord injury, and is
usually initially treated with a strict voluntary urination schedule, which may be coupled with Crede's
maneuver. It can be treated further by adding bethanechol to the regimen. Many patients have to be
taught intermittent self-catheterization of the bladder. Ultimately, the patient may require resection of the
internal sphincter of the bladder neck.
200. A 35-year-old white male presents to the emergency department with chest pain of 30 minutes
duration. He describes the pain as feeling like pressure on his chest, and says it radiates into
his left arm. It is accompanied by dyspnea, diaphoresis, anxiety, and palpitations. His past
medical history is unremarkable and he has no family history of premature heart disease. He
smokes 2 packs of cigarettes per day and admits to intranasal cocaine use 2 hours ago.
Vital signs include a blood pressure of 180/110 mm Hg, a pulse rate of 110 beats/min, a
respiratory rate of 24/min, and a temperature of 37.2°C (99.0°F). Other than the anxiety and
diaphoresis, the general examination is unremarkable. An EKG shows sinus tachycardia with
an early repolarization pattern. Aspirin and nitroglycerin have been administered, as well as
oxygen via nasal cannula.
Which one of the following would be most appropriate at this point?
A) Nifedipine (Procardia)
B) Enalaprilat intravenously
C) Lorazepam (Ativan) intravenously
D) Metoprolol (Toprol) intravenously
E) Thrombolytic therapy
Item 200
ANSWER: C
Treatment of cocaine-associated chest pain is similar to that of acute coronary syndrome, unstable angina,
or acute myocardial infarction, but there are exceptions. The hypertension, tachycardia, and chest pain
will often respond to intravenous benzodiazepines as early management. While -blockers are
recommended for acute myocardial infarction, they can exacerbate coronary artery spasm in cocaineassociated
chest pain. Fibrinolytic therapy should be given only to patients who clearly have an STsegment
elevation myocardial infarction and cannot receive immediate direct percutaneous coronary
intervention. Calcium channel blocker use in the setting of cocaine-induced ischemia has not been studied,
but may be considered if there is no response to benzodiazepines and nitroglycerin. There are no
recommendations regarding the use of ACE inhibitors, but these agents would not address the tachycardia.
201. According to both the Centers for Disease Control and the American College of Sports
Medicine, in order to burn fat stores obese patients should exercise
A) a minimum of 30 minutes 3 days/week
B) a minimum of 30 minutes at least 5 days/week
C) a minimum of 20 minutes every day
D) a minimum of 10 minutes at least 3 times daily
Item 201
ANSWER: B
Because glycogen is the primary energy source for muscles during the first 20 minutes of exercise, at least
30 minutes of exercise is necessary to begin burning fat stores. The CDC and the American College of
Sports Medicine recommend a minimum of 30 minutes of exercise 5 days per week.
Twenty minutes of exercise daily or three 10-minute sessions daily does improve cardiovascular fitness,
but does not cause significant weight loss. Walking, on land or in water, and stationary biking are
equivalent in benefit.
202. In which one of the following scenarios would additional consent from a child’s parent or
guardian be necessary prior to treatment?
A) A 6-year-old female with divorced parents who lives primarily with her mother is
brought to the clinic by her father to discuss his concerns of possible abuse
B) An 8-year-old unconscious male is brought to the emergency department by a neighbor
after falling out of a tree and striking his head
C) A 13-year-old male is brought to the clinic by a babysitter with a note giving permission
to treat signed by a parent
D) A 15-year-old female who is considered emancipated under state law comes to your
office to discuss family planning
E) A 16-year-old female who has driven herself to her clinic appointment reports a 2-day
history of ear pain; she says her mother made this appointment for her
Item 202
ANSWER: E
Informed consent to treat is considered an important ethical and legal part of caring for children and
adolescents. Some situations can become confusing when trying to balance the need for treatment, a
child’s assent, and a parent or guardian’s permission. In most states, 18 is the age when legal decisions
can be made; however, in some states it is 21.
Children under the age of majority must have proof of permission to treat from a parent or guardian for
non-emergent care. This does not apply to emergency situations in which a delay in care could result in
serious harm. Another exception to parental consent is when a child is considered emancipated under state
law. This can happen with a court order, or (in some states) if the child is married, is a parent, is in the
military, or is living independently. Either biologic parent can consent to treatment unless one of them
is explicitly denied guardianship. If a child presents with a non-emergent condition and does not have
evidence of permission from a parent or guardian, permission should be sought before the physician
interaction takes place.
203. A 50-year-old female presents with a 2-day history of four vesicles on her upper eyelid, but no
pain or swelling. She has not experienced any eye trauma, has had no vision changes, and has
no other skin changes.
Which one of the following would be the most appropriate next step in treating this patient?
A) Referral to an ophthalmologist
B) A methylprednisolone (Medrol) dose pack
C) A topical corticosteroid
D) Topical mupirocin (Bactroban)
E) Topical metronidazole (MetroGel)
Item 203
ANSWER: A
This patient likely has herpes zoster ophthalmicus. In addition to treatment with a systemic antiviral agent,
it is important that the patient see an ophthalmologist to be evaluated for corneal disease and iritis, as vision
can be lost. This is a viral infection, so corticosteroids could worsen the infection. Mupirocin or
metronidazole would not resolve the infection.
204. A 70-year-old white male with hypertension has several abnormal liver function tests on routine
testing. He says he does not drink alcohol, and the prescription medications he is taking are
unlikely to cause hepatotoxicity. However, during more extensive history taking, he tells you
that he does use some over-the-counter medications.
Which one of these is most likely responsible for the abnormal laboratory findings?
A) Aspirin, used occasionally for headache
B) A fiber supplement taken to promote regular bowel habits
C) One long-acting niacin tablet per day
D) One 250-mg vitamin C tablet daily
E) Chewable simethicone after meals, almost daily
Item 204
ANSWER: C
Hepatotoxicity resulting from timed-release formulations of niacin has been reported in elderly individuals.
Patients may be taking this supplement without their physician’s knowledge, feeling it is safe because it
is a vitamin. Aspirin and vitamin C can result in gastrointestinal iron loss and anemia. The other
medications listed, if used in moderation, would not be expected to alter laboratory findings.
205. A 65-year-old male asks you about an advertisement for vascular disease screening tests that will
be performed at a local retail outlet this Saturday. The cost is minimal, but will be out of
pocket. The patient is asymptomatic, but has a 45-pack-year smoking history.
Based on U.S. Preventive Services Task Force guidelines, which one of the following screening
tests would you recommend for this patient?
A) Duplex ultrasonography of both carotid arteries
B) Ultrasonography of the thoracic aorta
C) Ultrasonography of the abdominal aorta
D) An ankle-brachial pressure index
Item 205
ANSWER: C
Although screening recommendations for abdominal aortic aneurysms vary, a 65-year-old male with a
history of smoking should be screened by ultrasonography. The other tests are not indicated in
asymptomatic patients.
206. A 35-year-old white male who has had diabetes mellitus for 20 years begins having episodes of
hypoglycemia. He was previously stable and well controlled and has not recently changed his
diet or insulin regimen.
Which one of the following is the most likely cause of the hypoglycemia?
A) Spontaneous improvement of $-cell function
B) Renal disease
C) Reduced physical activity
D) Insulin antibodies
Item 206
ANSWER: B
The most common cause of hypoglycemia in a previously stable, well-controlled diabetic patient who has
not changed his or her diet or insulin dosage is diabetic renal disease. A reduction in physical activity or
the appearance of insulin antibodies (unlikely after 20 years of therapy) would increase insulin
requirements and produce hyperglycemia. Spontaneous improvement of -cell function after 20 years
would be very rare.
207. A 54-year-old male presents to the emergency department with an acute onset of chest pain. His
cardiac risk factors include hypertension, hyperlipidemia, and a positive family history. His
temperature is 37.0°C (98.6°F), pulse rate 80 beats/min, blood pressure 155/86 mm Hg, and
respiratory rate 22/min. His oxygen saturation is 95% on room air. An EKG shows rare
unifocal PVCs and nonspecific ST-T–wave changes. Initial cardiac markers are negative.
Which one of the following would be most appropriate at this point?
A) Helical (spiral) CT of the chest
B) Echocardiography
C) PA and lateral chest films
D) A ventilation-perfusion scan
E) Magnetic resonance angiography
Item 207
ANSWER: C
PA and lateral chest radiographs are still valuable in the early evaluation of patients with chest pain. While
they do not confirm or rule out the presence of myocardial ischemia, other causes of chest pain may be
evident, such as pneumothorax, pneumonia, or heart failure. The chest film may also provide clues about
other possible diagnoses, such as pulmonary embolism, aortic disease, or neoplasia. The other tests listed
often have a role in the evaluation of chest pain, but none has supplanted the plain chest film as the best
initial imaging study.
208. A 19-year-old college freshman consults you at the request of her cross-country coach because
she has not had a period in 2 of the last 3 months. She notes that her current training regimen
is much more intense than in high school last year. She has an appropriate body image and
denies caloric restriction. A pregnancy test at the student health center was negative. On
examination she is lean and highly trained. Her examination is otherwise normal.
Which one of the following would be the most appropriate recommendation for this patient?
A) Estrogen supplementation
B) Cyclic oral contraceptive pills
C) Increased caloric intake
D) Bisphosphonate therapy
E) Discontinuation of elite-level athletics
Item 208
ANSWER: C
This patient has exercise-related oligomenorrhea, but does not have the eating disorder that characterizes
the female athlete triad. Menstrual problems in athletes do correlate with bone density loss and impaired
recovery from exercise. Additionally, menstrual irregularity of varying severity is extremely common in
female distance runners, perhaps affecting as many as 60%. Hormonal manipulation has not been shown
to affect bone density, though it may produce withdrawal bleeding. Bisphosphonate therapy has been
shown to be ineffective, and is not recommended in women of child-bearing age.
209. At a routine visit in October, a 17-year-old primigravida at 10 weeks gestation asks whether she
should get influenza vaccine. Her mother recommended it, but she is concerned about the
needle stick and potential harm to the fetus.
Which one of the following would you do?
A) Recommend intramuscular vaccine and tell her that evidence indicates some protection
for the baby up to 6 months of age
B) Recommend nasal vaccine because the patient is under age 50 and needle-averse
C) Recommend vaccine only if the patient has a coexistent chronic illness
D) Recommend that vaccination be delayed until the second trimester to reduce fetal risk
E) Recommend immunization of household contacts to reduce maternal risk, but no
immunization of the patient
Item 209
ANSWER: A
Women who will be pregnant during the influenza season should receive the inactivated vaccine (SOR C).
The live nasal vaccine is not approved for use in pregnancy. The vaccine can be given in any trimester.
Coexistent illness is not required for this indication. There appears to be some protective effect for the
infant up to the age of 6 months.
Immunization of family members is sometimes recommended for immunocompromised patients. In the
absence of other indications, however, it has not been recommended for family members of pregnant
patients.
210. A 50-year-old male comes to your office for a “doctor’s excuse” for days of work he missed last
week. He attended a picnic where he and other guests developed nausea and vomiting 2 hours
after eating. Within 48 hours, the symptoms had resolved.
The most likely etiology of the illness is which one of the following?
A) Staphylococcus
B) Clostridium botulinum
C) Clostridium perfringens
D) Clostridium difficile
E) Actinomycosis
aItem 210
ANSWER: A
This is a typical presentation of staphylococcal food poisoning. The symptoms usually begin 1–6 hours
after ingestion and resolve within 24–48 hours. Foodborne botulism is most commonly found in homecanned
foods, and symptoms begin 18–36 hours after ingestion. Clostridium perfringens is transmitted
in feces and water, and symptoms begin 6–24 hours after ingestion. Clostridium difficile is associated with
antibiotic use. Actinomycosis causes local abscesses, not gastroenteritis.
211. A 2-year-old Hispanic male with a 3-day history of nasal congestion presents with a barking
cough and hoarseness. He is afebrile. The examination reveals tachypnea, inspiratory and
expiratory stridor, noticeable intercostal retractions, and good color.
Which one of the following is indicated?
A) Albuterol syrup and the use of a humidifier
B) Inhaled albuterol (Proventil, Ventolin)
C) Aerosolized epinephrine and intramuscular dexamethasone
D) Visualization of the epiglottis, and ceftriaxone (Rocephin)
Item 211
ANSWER: C
This child has a history and physical findings typical of viral laryngotracheobronchitis, or croup. In rare
instances, this illness can be complicated by critical upper airway obstruction. The symptoms of cough,
respiratory stridor, and distress result from edema of the subglottic portion of the upper airway.
Humidification of inspired air is sometimes beneficial, but the child should not be sent home until
improvement is demonstrated. Because this child has stridor and intercostal retractions, aerosolized
epinephrine is indicated, along with intramuscular dexamethasone, and hospitalization may be required for
observation and continued treatment. Antibiotics do not have a role in the treatment of viral croup, and
attempted visualization of the epiglottis is not indicated since it will increase the child’s anxiety and worsen
the symptoms.
212. The most effective daily doses of vitamin D and calcium for hip fracture prevention in
postmenopausal women are
A) 800 IU vitamin D and 500 mg calcium
B) 400 IU vitamin D and 500 mg calcium
C) 400 IU vitamin D and 1000 mg calcium
D) 800 IU vitamin D and 1200 mg calcium
Item 212
ANSWER: D
The most effective daily dose of vitamin D for hip fracture prevention in postmenopausal women is 800
IU, and the recommended daily dose of calcium is 1200 mg.
213. The physician counseling a 4-year-old child about the death of a loved one should keep in mind
that children in this age group
A) often feel no sense of loss
B) often believe they are somehow responsible for the death
C) should not attend a funeral
D) should usually be told the loved one is having a long sleep
E) usually accept the finality of death with little question
Item 213
ANSWER: B
Children from the ages of 2 to 6 often believe they are somehow responsible for the death of a loved one.
The emotional pain may be so intense that the child may react by denying the death, or may somehow feel
that the death is reversible. If children wish to attend a funeral, or if their parents want them to, they
should be accompanied by an adult who can provide comfort and support. Telling a child the loved one
is asleep or that he or she “went away” usually creates false hopes for return, or it may foster a sleep
phobia.
214. A 35-year-old white male presents with dyspepsia. He has had no symptoms that suggest
gastroesophageal reflux or bleeding, but a test for Helicobacter pylori is positive. After 2 weeks
of treatment with omeprazole (Prilosec), amoxicillin, and clarithromycin (Biaxin), he is
asymptomatic.
Which one of the following is recommended to test for the eradication of H. pylori in this
patient?
A) Immunoglobulin G serology
B) A urea breath test
C) Upper endoscopy with a biopsy
D) An upper gastrointestinal series
tem 214
ANSWER: B
There is strong evidence that eradication of H. pylori improves healing and reduces the risk of recurrence
or rebleeding in patients with duodenal or gastric ulcer. A test-and-treat approach is recommended for
most patients with undifferentiated dyspepsia. This strategy reduces the need for antisecretory
medications, as well as the number of endoscopies. The currently recommended test for eradication of
H. pylori in this clinical setting is either the urea breath test or H. pylori stool antigen.
Serology remains positive for months after eradication and may give misleading information. Although
upper endoscopy, with a biopsy for histology, urease activity, or culture, can be used to test for
eradication, it is an invasive procedure with a higher cost and the potential for more morbidity compared
to the urea breath test or the H. pylori stool antigen test. Rather than recommending endoscopy for all
patients, most national guidelines suggest a test-and-treat strategy unless the patient is over 45 years old
or has red flags for malignancy or a complicated ulcer. Although an upper gastrointestinal series might
provide information about gross pathology, it will not provide information about the eradication of H.
pylori following treatment.
215. Which one of the following is recommended for the treatment of patients with obsessivecompulsive
disorder?
A) Cognitive-behavioral therapy
B) Psychoanalytic therapy
C) Family therapy
D) Psychodynamic psychotherapy
E) Motivational interviewing
aItem 215
ANSWER: A
Cognitive-behavioral therapy is the recommended treatment for obsessive-compulsive disorder (OCD).
Psychoanalytic therapy has not been shown to help treat OCD. Family therapy can help reduce family
tensions that result from the disease. Psychodynamic psychotherapy and motivational interviewing may
help patients overcome their resistance to treatment.
216. A 73-year-old male sees you for evaluation of a tremor. Based on the history and examination,
you suspect Parkinson’s disease.
Which one of the following would be most helpful for confirming the diagnosis?
A) CT of the brain
B) MRI of the brain
C) A positive response to levodopa
D) Confirming that the tremor occurs with movement
E) Confirming that the tremor had a symmetric onset
aItem 216
ANSWER: C
Patients with Parkinson's disease should respond to an adequate therapeutic challenge of levodopa or a
dopamine agonist. The diagnosis of idiopathic Parkinson's disease is clinical, not radiographic. Cardinal
signs of Parkinson's disease include an asymmetric tremor onset and a distal resting tremor of 3–6 Hz.
217. Surgical management for an acute midshaft clavicle fracture would be appropriate in which one
of the following?
A) An 11-year-old male with a comminuted fracture
B) A 15-year-old female with a ½-cm displaced fracture
C) A 30-year-old male with a ½-cm displaced fracture
D) A 40-year-old male with a nondisplaced fracture
E) A 50-year-old female with a comminuted fracture
aItem 217
ANSWER: E
Midshaft clavicle fractures are usually treated nonoperatively, but have a higher risk of nonunion. Risk
factors for nonunion include female gender, fracture comminution or displacement, clavicle shortening,
advanced age, and greater extent of initial trauma. These fractures in children heal extremely well, even
if displaced or comminuted, because of periosteal regenerative potential.
218. A 32-year-old primipara is ready to be discharged after a full-term vaginal delivery that was
complicated by a prolonged second stage of labor. She required a second-degree posterior
vaginal repair, but had no periurethral trauma. A transurethral catheter was removed a few
hours after delivery, but 48 hours later she complained of abdominal pain and a persistent need
to urinate. The catheter was replaced and yielded approximately 2000 cc of straw-colored urine.
Urinary symptoms quickly resolved, but the patient continues to be unable to void on her own.
A perineal examination is normal, as is a urinalysis.
Which one of the following would be the most appropriate management at this time?
A) Oxybutynin (Ditropan), 10 mg daily
B) Prednisone, starting with 60 mg/day and tapering quickly over 7 days
C) Urgent vaginal ultrasonography
D) Urology consultation for cystoscopy
E) Discharge with a catheter in place and close follow-up
aItem 218
ANSWER: E
This patient suffers from postpartum urinary retention (PUR). PUR is often defined as a post-void bladder
residual of at least 150 cc that is present 6 hours or more after delivery. This condition is more likely to
occur in patients who are primiparous, have a prolonged first or second stage of labor, have instrumented
vaginal deliveries, or require a cesarean section for failure to progress. The question of whether epidural
anesthesia promotes the condition is still debated. Most cases of PUR will resolve 2–6 days after delivery,
but some can take up to several weeks. The use of intermittent self-catheterization or a transurethral
catheter is recommended until the patient’s ability to spontaneously micturate returns. Imaging studies and
referrals to a specialist are rarely necessary, and no medication has been proven helpful.
219. A 5-month-old infant has had several episodes of wheezing, not clearly related to colds. The
pregnancy and delivery were normal; the infant received phototherapy for 1 day for
hyperbilirubinemia. He had an episode of otitis media 1 month ago. There is no chronic runny
nose or strong family history of asthma. He spits up small amounts of formula several times a
day, but otherwise appears well. His growth curve is normal. An examination is unremarkable
except for mild wheezing.
Which one of the following is the most likely diagnosis?
A) Benign reactive airway disease of infancy
B) Gastroesophageal reflux
C) Unresolved respiratory syncytial virus infection
D) Early asthma
E) Cystic fibrosis
aItem 219
ANSWER: B
Gastroesophageal reflux is a common cause of wheezing in infants. At 5 months of age, most infants no
longer spit up several times a day, and this is a major clue that this child’s wheezing may be from the
reflux. In addition, there is no family history of asthma and the wheezing is not related to infections.
Cystic fibrosis is more likely to present with recurrent infections and failure to thrive than with intermittent
wheezing.
220. A patient with end-stage metastatic cancer is having continued significant pain despite regular
use of 60 mg of long-acting morphine sulfate every 12 hours. What is the maximum 24-hour
dose of morphine sulfate that you may safely titrate up to in order to relieve this patient’s pain?
A) 240 mg
B) 360 mg
C) 480 mg
D) 600 mg
E) No limit
Item 220
ANSWER: E
Because there is no therapeutic ceiling for morphine, extremely large dosages can be used safely and
effectively if the drug is titrated properly.
221. On examination a 2-year-old child is found to have otalgia, a temperature of 39.0°C (102.2°F),
and a bulging, red tympanic membrane. She weighs 17 kg (35 lb).
Which one of the following would be the appropriate dosage of amoxicillin (Amoxil) for this
child?
A) 375 mg/day
B) 500 mg/day
C) 750 mg/day
D) 1000 mg/day
E) 1500 mg/day
aItem 221
ANSWER: E
For treating acute otitis media in this patient, the current recommended dosage of amoxicillin is 80–90
mg/kg/day.
222. Which one of the following surgical procedures is associated with the highest risk for
perioperative myocardial ischemia?
A) Femoropopliteal bypass
B) Pulmonary lobectomy
C) Hip arthroplasty
D) Transurethral resection of the prostate
E) Mastectomy
aItem 222
ANSWER: A
When deciding whether or not to recommend preoperative noninvasive cardiac testing, both patient risk
factors and surgical risk factors should be taken into account. Surgical procedures associated with a high
(>5%) risk of perioperative myocardial ischemia include aortic and peripheral vascular surgery and
emergent major operations, especially in patients over 75 years of age. Head and neck surgery,
intraperitoneal and intrathoracic surgery, orthopedic surgery, and prostate surgery carry an intermediate
risk (1%–5%). Endoscopic procedures and cataract and breast surgeries are considered low-risk (<1%)
procedures.
223. A 60-year-old white female with type 1 diabetes mellitus presents with early satiety, nausea,
bloating, and postprandial fullness. Laboratory tests are normal, as are upper endoscopy and
biliary ultrasonography.
Which one of the following would help confirm the most likely diagnosis?
A) Pelvic ultrasonography
B) An exercise stress test
C) Psychiatric consultation
D) Gastric emptying scintigraphy
E) Colonoscopy
aItem 223
ANSWER: D
This patient has typical findings of gastroparesis, an autonomic neuropathy more commonly seen in type
1 diabetics and in women. The initial evaluation should include a patient history and examination, a CBC
to rule out infection, a metabolic panel, endoscopy, and a biliary tract evaluation, but the diagnosis is best
confirmed by scintigraphy. Pelvic ultrasonography and colonoscopy are not indicated because the patient’s
symptoms are upper intestinal. Cardiac evaluation and psychiatric consultation are not warranted with
these symptoms.
224. During a preparticipation examination of a 5-year-old male for summer soccer camp, his mother
states that he frequently awakens during the night with complaints of cramping pain in both legs,
and that he seems to experience this after a day of heavy physical activity. She says that he
appears to drag his legs at times, but she has never noticed a definite limp. A physical
examination of the hips, knees, ankles, and leg musculature is entirely normal.
Which one of the following would be the most appropriate next step in the evaluation and
management of this patient?
A) Plain films of both hips and knees
B) Serum electrolyte levels
C) Recommending that he not participate in running sports
D) Reassurance, with no activity restrictions or treatment
E) Referral to a pediatric orthopedist
aItem 224
ANSWER: D
Benign nocturnal limb pains of childhood (growing pains) occur in as many as one-third of children, most
often between 4 and 6 years of age. The etiology is unknown, but the course does not parallel pubescent
growth, as would be expected if bone growth was the source of pain. The pain often awakens the child
within hours of falling asleep following an active day. The pain is generally localized around the knees,
most often in the shins and calves, but also may affect the thighs and the upper extremities. A
characteristic history coupled with a normal physical examination will confirm the diagnosis. Reassurance
that no additional tests or treatments are necessary and that the condition is self-limiting is the most
appropriate response.
225. During a routine prenatal visit, a patient at 28 weeks gestation describes a worsening pain in her
lower back and pelvic area. She is averse to analgesics but is eager to try exercise to relieve the
pain. Additional patient history and an examination confirm that the pain is not due to
underlying medical problems.
Which one of the following would be the most appropriate exercise prescription for this patient?
A) Isometric exercise
B) Concentric exercise
C) Core stability exercise
D) Closed kinetic chain exercise
E) Isotonic exercise
aItem 225
ANSWER: C
Low back pain and pelvic pain are commonly encountered in pregnancy, a time when medication or
physical modality use may prove undesirable or difficult. A properly prescribed exercise program is a
generally safe and effective method to treat this pain. The most appropriate exercises for pregnancyrelated
pelvic pain and low back pain target the low back, trunk, and abdominal muscles to increase core
stability. Examples of such exercises include Pilates, back extension exercises, and abdominal crunches.
Isometric and isotonic exercises work muscle groups against either an external force or opposing muscle
groups, and are best suited for the development of muscle tone, strength, and conditioning in the
extremities. Likewise, concentric and closed kinetic chain exercises involve working muscles against
resistance, and are best suited for rehabilitating and strengthening the extremities.
226. You have just received test results confirming that a 78-year-old patient has metastatic lung
cancer. She informs you she does not want to know the results of the tests and is “leaving it in
God’s hands.” You know that additional issues need to be explored, such as her desire for
chemotherapy and hospice care.
Which one of the following is the most appropriate strategy for determining her wishes?
A) Acknowledge her concerns, but proceed with a discussion of her diagnosis and
prognosis
B) Ask family members to gently break the news to your patient and tell them you will
return later to discuss the details and answer questions
C) Have a hospice representative visit and discuss the diagnosis and options for care
D) Ask the patient to designate someone with whom you can discuss the results and
prognosis
aItem 226
ANSWER: D
In a patient-centered approach to communication regarding end-of-life care, a patient’s wishes to not know
about a diagnosis or prognosis should be respected. However, it is reasonable to ask the patient to name
a proxy with whom you may discuss the issues. The other options listed do not respect the patient’s desire
to not know her diagnosis or prognosis.
227. You see a 30-year-old white male for the first time for a routine evaluation. He says that he has
been bothered by multiple skin lesions on the neck and axillae. On examination you note
numerous skin tags.
The presence of these lesions indicates an increased risk for
A) diabetes mellitus
B) squamous cell skin cancer
C) melanoma
D) glioblastoma multiforme
E) AIDS
aItem 227
ANSWER: A
Skin tags, or acrochordons, are associated with diabetes mellitus and obesity. The onset often occurs in
early adulthood, and the most common locations are the neck and axillae. These skin lesions are not
associated with any significant cancer risk, and have not been associated with HIV infection.
228. Which one of the following is considered first-line therapy for migraine prophylaxis in adults?
A) Gabapentin (Neurontin)
B) Propranolol (Inderal)
C) Fluoxetine (Prozac)
2 D) Vitamin B (riboflavin)
E) Naproxen (Naprosyn)
aItem 228
ANSWER: B
Propranolol is a first-line therapy for migraine prophylaxis in adults (SOR A). In a review of 26 placebocontrolled
trials using data pooled from nine studies, the calculated responder ratio (comparable to relative
risk) was 1.9 (95% confidence interval 1.6–2.35). Other first-line agents include timolol, amitriptyline,
divalproex sodium, sodium valproate, and topiramate.
2 Gabapentin, fluoxetine, vitamin B , and naproxen are considered second-line therapies for migraine
prophylaxis in adults (SOR B), and should be used when no first-line agent or combination is effective or
tolerable.
229. A 52-year-old white male is being considered for pharmacologic treatment of hyperlipidemia
because of an LDL-cholesterol level of 180 mg/dL. Before beginning medication for his
hyperlipidemia, he should be screened for
A) hyperthyroidism
B) hypothyroidism
C) Addison’s disease
D) Cushing’s disease
E) pernicious anemia
aItem 229
ANSWER: B
According to the National Cholesterol Education Program Adult Treatment Panel III Report of 2001, any
person with elevated LDL cholesterol or any other form of hyperlipidemia should undergo clinical or
laboratory assessment to rule out secondary dyslipidemia before initiation of lipid-lowering therapy.
Causes of secondary dyslipidemia include diabetes mellitus, hypothyroidism, obstructive liver disease,
chronic renal failure, and some medications.
230. A 36-year-old female presents with a several-week history of polyuria and intense thirst. She
currently takes no medications. On examination her blood pressure and pulse rate are normal,
and she is clinically euvolemic. Laboratory tests, including serum electrolyte levels, renal
function tests, and plasma glucose, are all normal. A urinalysis is significant only for low
specific gravity. Her 24-hour urine output is >5 L with low urine osmolality.
The most likely cause of this patient’s condition is a deficiency of
A) angiotensin II
B) aldosterone
C) renin
D) insulin
E) arginine vasopressin
aItem 230
ANSWER: E
This patient has diabetes insipidus, which is caused by a deficiency in the secretion or renal action of
arginine vasopressin (AVP). AVP, also known as antidiuretic hormone, is produced in the posterior
pituitary gland and the route of secretion is generally regulated by the osmolality of body fluid stores,
including intravascular volume. Its chief action is the concentration of urine in the distal tubules of the
kidney. Both low secretion of AVP from the pituitary and reduced antidiuretic action on the kidney can
be primary or secondary, and the causes are numerous.
Patients with diabetes insipidus present with profound urinary volume, frequency of urination, and thirst.
The urine is very dilute, with osmolality <300 mOsm/L. Further workup will help determine the specific
type of diabetes insipidus and its cause, which is necessary in order to implement appropriate treatment.
Low levels of aldosterone, plasma renin activity, or angiotensin would cause abnormal blood pressure,
electrolyte levels, and/or renal function. Insulin deficiency results in diabetes mellitus.
231. Sympathomimetic decongestants such as pseudoephedrine and phenylephrine can be problematic
in elderly patients because they can
A) decrease blood pressure
B) cause bradycardia
C) worsen existing urinary obstruction
D) enhance the anticholinergic effects of other medications
E) enhance the sedative effects of other medications
aItem 231
ANSWER: C
Sympathomimetic agents can elevate blood pressure and intraocular pressure, may worsen existing urinary
obstruction, and adversely interact with alpha-blockers, methyldopa, tricyclic antidepressants, and oral
hypoglycemic agents and MAOIs. They also speed up the heart rate. First-generation nonprescription
antihistamines can enhance the anticholinergic and sedative effects of other medications.
232. An 86-year-old female presents to your office with a complaint of increasing cough, especially
at night, over the past 2–3 weeks. On examination you hear some crackles at the bases of both
lungs.
The chest radiograph shown in Figure 1 is consistent with which one of the following causes of
this patient’s cough?
A) Bilateral pneumonia
B) Asbestosis
C) Tuberculosis
D) Heart failure
E) Emphysema
aItem 232
ANSWER: D
The chest radiograph is consistent with heart failure. It shows cardiomegaly, with a cardiothoracic ratio
>6.50, as well as some enlargement of pulmonary veins due to pulmonary venous hypertension.
The radiograph does not show an infiltrate, as would be expected with community-acquired pneumonia.
Pleural plaques would be expected with asbestosis, and upper-lobe involvement or cavitary lesions with
tuberculosis. With emphysema, there is typically a small vertical heart and evidence of hyperexpansion.
233. A 70-year-old white male presents with fatigue, weakness, and foot paresthesias. His
hemoglobin level is 10.5 g/dL (N 12.6–17.4). His peripheral smear is shown in Figure 2.
Which one of the following is the most likely diagnosis?
A) Iron deficiency anemia
12 B) Vitamin B deficiency anemia
C) Hemolytic anemia
D) Acute myelogenous leukemia
E) Chronic myelogenous leukemia
aItem 233
ANSWER: B
The blood smear shows a hypersegmented polymorphonuclear (PMN) white blood cell, typical of vitamin
12 B deficiency with pernicious anemia. The anemia is of the macrocytic type (MCV >100 m ). There 3
is no evidence of hemolysis or leukemia. While iron deficiency anemia can be a coexisting problem, the
12 hypersegmented PMN is classic for vitamin B deficiency.
12 It is important to note that elderly patients with vitamin B deficiency may have neurologic signs and
symptoms before developing hematologic abnormalities.
234. A 3-year-old female is brought to your office with a 3-hour history of skin lesions that are
prominent, warm, papular, and serpiginous (see Figure 3). Which one of the following is the
most likely cause of these lesions?
A) Heredity
B) Physical abuse
C) Infection
D) A topical agent
E) An oral medication
aItem 234
ANSWER: E
Acute urticaria occurs when an allergen activates mast cells in the skin, and is commonly caused by oral
and parenteral drugs, food, and, less frequently, infections. Topical agents and physical abuse are unlikely
to present in this manner, and hereditary angioedema is more a systemic illness than a skin disorder.
235. An 84-year-old male is walking across the street and has to hurry to avoid oncoming traffic. He
suddenly develops extreme pain in his knee and falls to the street, and has to be carried to the
sidewalk.
The following day he comes to the emergency department. He is comfortable when placed in
a knee immobilizer, but is very tender just above the patella. He can bend his knee, but when
he tries to straighten his leg it is so weak that he cannot move it at all. Radiographs of the knee
are shown in Figure 4.
What is the most likely diagnosis?
A) Patellar tendon rupture
B) Quadriceps tendon rupture
C) Tibial plateau fracture
D) Patellar subluxation
E) Lumbar radiculopathy
aItem 235
ANSWER: B
Quadriceps tendon rupture can be partial or complete. When complete, as in this case, the patient has no
ability to straighten the leg actively. A similar pattern is seen with patellar tendon rupture, but in this
situation the patella is retracted superiorly by the quadriceps. Quadriceps rupture often produces a sulcus
sign, a painful indentation just above the patella. If the patient is not seen until some time has passed since
the injury, the gap in the quadriceps can fill with blood, so that it is no longer palpable. The clinical
examination is usually diagnostic for this condition, but this patient’s radiograph shows some interesting
findings, especially on the lateral view. A small shard of the patella has been pulled off and has migrated
superiorly with the quadriceps. The hematoma filling the gap in the quadriceps is the same density as the
muscle, but wrinkling of the fascia over the distal quadriceps provides a clue that it is no longer attached
to the superior margin of the patella.
Tibial plateau fractures are intra-articular, so they produce a large hemarthrosis. They are evident on a
radiograph in almost all cases. Pain inhibits movement of the knee, but the extreme weakness evident in
this case would not be seen.
Patellar subluxation is obvious acutely, when the patella is displaced laterally. More often, the patient
comes in after the patella has relocated. Findings then include tenderness along the medial retinaculum,
sometimes a joint effusion, and a positive apprehension sign when the patella is pushed gently laterally.

Lumbar radiculopathy can cause weakness of the quadriceps if it involves the third lumbar root, but
complete paralysis would not occur. Other findings would include lumbar pain radiating to the leg,
possibly with paresthesias and fasciculations if there were significant neurologic impairment.
236. A 25-year-old female is in active labor at term and is dilated to 7 cm. An electronic fetal
monitoring tracing is shown in Figure 5.
Which one of the following is a possible etiology for this fetal heart rate pattern?
A) Normal progress of labor
B) Maternal fever
C) Effects of epidural anesthesia
D) Post-dates pregnancy
E) Umbilical cord prolapse
aItem 236
ANSWER: B
This tracing shows fetal tachycardia, defined as a baseline fetal heart rate >160 beats/min for at least 15
minutes. This is considered a nonreassuring pattern. Causes of fetal tachycardia include maternal fever,
fetal hypoxia, hyperthyroidism, maternal or fetal anemia, medication effects of parasympatholytic or
sympathomimetic drugs, chorioamnionitis, fetal tachyarrhythmia, and prematurity. Fetal tachycardia is
not a sign of normal progression of labor. Epidural anesthesia, post-dates pregnancy, and umbilical cord
prolapse would all be causes of fetal bradycardia.
237. A hospitalized 55-year-old male has developed the tachyarrhythmia shown in Figure 6. He is
alert and denies chest pain, although he complains of palpitations and is mildly dyspneic since
the onset of this sustained dysrhythmia. His blood pressure is 116/76 mm Hg and pulse
oximetry shows 93% saturation on 2L of oxygen.
Which one of the following would be the best treatment for this patient?
A) Synchronized DC cardioversion
B) Metoprolol (Lopressor)
C) Amiodarone (Cordarone)
D) Adenosine (Adenocard)
E) Atropine
aItem 237
ANSWER: D
The patient has paroxysmal supraventricular tachycardia (PSVT) with a heart rate of approximately 170
beats/min. Intravenous adenosine is the treatment of choice for PSVT. Because the patient is
hemodynamically stable, DC cardioversion is not indicated. Metoprolol may slow the heart rate but likely
will not convert it to sinus rhythm. Amiodarone is indicated for hemodynamically stable ventricular
tachycardia. Atropine is contraindicated in this or any other tachyarrhythmia.
238. A 12-year-old male who lives on a farm presents with lesions on his toes (shown in Figure 7).
Which one of the following items from the patient’s history is relevant to the diagnosis?
A) Recent tooth extraction and gingival surgery
B) A family history of systemic lupus erythematosus
C) Recurrent fevers for the past 2 weeks
D) Exposure to cold temperatures
E) Vaccination of the sheep he is raising for a 4-H project
aItem 238
ANSWER: D
This patient has pernio, or chilblains, which is a localized inflammatory lesion of the skin, usually found
in the extremities following exposure to nonfreezing cold temperatures. It is generally a benign condition,
and is not associated with any systemic diseases. These lesions are red-purple plaques with deep swelling,
and are accompanied by itching or burning. They are not associated with infections or connective tissue
disease.
239. A 90-year-old female nursing-home patient has a 1.5×2.0-cm lesion on her face (shown in
Figure 8). She states that the “spot” has been present for years and that it doesn’t bother her.
Closer examination reveals a flat maculopapular lesion with varying colors and an irregular
border.
Which one of the following is the most likely diagnosis?
A) Actinic keratosis
B) Metastatic breast carcinoma
C) Seborrheic keratosis
D) Lentigo maligna melanoma
E) Basal cell carcinoma
aItem 239
ANSWER: D
This patient has a malignant melanoma, often called lentigo maligna melanoma. These lesions typically
appear during the seventh or eighth decade of life, and are most often located on the face. This patient's
age, health status, and wishes must be considered in any treatment plans. The other skin lesions listed can
be seen in this age group, but they are easily distinguished from this malignant lesion.
240. A 5-year-old male fell while playing and complained that his wrist hurt. The next day he is
brought to your office because he refuses to use his arm.
Which one of the following best describes the condition seen in the radiographs shown in Figure
9?
A) A normal appearance
B) A radial fracture
C) An ulnar fracture
D) A radioulnar fracture
E) Indeterminate result
aItem 240
ANSWER: B
Even though they are the most common fracture in this age group, radial fractures can be missed by
clinicians. The bend in the cortex of the distal radius indicates the fracture. Sometimes referred to as a
buckle or torus fracture, it will heal with almost any choice of treatment. Most clinicians opt for casting
to reduce the chance of reinjury during the first few weeks of healing, but the parents’ preferences in this
regard are important. Some pediatric long-bone fractures involve growth plates, and the results can be
indeterminate, requiring either more advanced imaging or comparison views of the opposite limb.
2 tests that can be used to confirm eradicattion of H pylori
The currently recommended test for eradication of
H. pylori in this clinical setting is either the urea breath test or H. pylori stool antigen.
How often should we exercise
30 minutes a day 5 times a week
Difference on radio iodine uptake scan between graves and subacute granulomatou thyroiditis
Patients with subacute
granulomatous thyroiditis will have a low radioactive iodine uptake (RAIU) at 24 hours, but patients with
Graves’ disease will have an elevated RAIU (SOR C).
Travelers diarrhea in a kid and a adult tratment?
Kid- azithro
Adult- cipro
treament of essential tremor
topamax or propanolol
a girl should have a work up if no menses by age
16
What type of dietary intervention in pt with a pressure ulcer
Maintaining a protein intake of at least 1.2–1.5 g/kg/day is recommended, and some authorities
recommend 2 g/kg/day with stage III or IV ulcers. Increased caloric intake is also necessary to promote
syncope related to changes in position, such as bending, lying down from a seated position,
or turning over in bed.
Atrial myxoma
syncope is associated with unpleasant stimuli or physiologic
conditions, including sights, sounds, smells, sudden pain, sustained upright posture, heat, hunger, and
acute blood loss.
Vasovagal
syncope with exercise
aortic stenosisi
what side effect watch out for typical and atypical antipsychotics
prolonged QTC
Conductive Hearing Loss
If a hearing loss is caused by problems with the ear "conducting" the vibrations to the cochlea, then it is called a conductive loss.

The problem may be with the ear drum or with the ossicles (three tiny bones).

Conductive losses aren't the most common types of hearing loss. Nearly 90% of hearing loss cases are sensorineural losses, where the problem is with the cochlea or the auditory nerve Some forms of hearing loss are caused by problems with these bones. When the hearing loss is caused by these bones, it's referred to as a conductive loss

A conductive loss is often a flat loss, affecting all frequencies. Hearing aids can be especially effective in helping someone with a conductive loss hear better.

Many times a conductive loss can be fixed or at least improved with surgery.
Akathisia
is a syndrome marked
by motor restlessness. Affected patients commonly complain of being inexplicably anxious, of being
unable to sit still or concentrate, and of feeling comfortable only when moving

can be caused by antipsychotics
Pts with claudication should also be on a
Statin